Está en la página 1de 291

Banco de preguntas grupo 11

Entre las medidas no farmacológicas de nefro protección encontramos:


A. Dieta para el control metabólico y restricción de sal.
B. Control de exceso de peso.
C. Restricción de proteínas en caso de ERC Estadio II, III, IV.
D. Todas las anteriores
ANSWER: D
El tratamiento de mantenimiento con IECA Aumenta el aclaramiento de insulina y el FSR, a la vez que
disminuye la proteinuria:
A. Verdadero
B. Falso
C. Aumenta el aclaramiento de insulina y disminuye el FSR
D. Disminuye el FSR y la proteinuria.
ANSWER: A
En la necrosis tubular aguda por nefrotoxicidad se dice que es el efecto del fármaco sobre el:
A. Intersticio renal
B. Epitelio tubular renal
C. Todas las anteriores
D. Ninguna de las anteriores
ANSWER: B
En la nefritis túbulo-intersticial por nefrotoxicidad se dice que es el efecto del fármaco sobre el:
A. Intersticio renal
B. Epitelio tubular renal
C. Todas las anteriores
D. Ninguna de las anteriores
ANSWER: A
La nefritis túbulo-intersticial por nefrotoxicidad es causada por fármacos como Ciclosporina, Litio,
AINES:
A. Verdadero
B. Falso
C. Es producida solo por AINES.
D. Es producida solo por Litio y Ciclosporina
ANSWER: A
Banco de preguntas grupo 11

La obstrucción intratubular es causada por fármacos como: Metrotexato, Aciclovir, Sulfonamidas,


Indinavir:
A. Verdadero
B. Falso
C. Causada por Metrotexato y Aciclovir.
D. Causada solo por Sulfonamidas.
ANSWER: A
¿De los siguientes ítems, Cual No corresponde a los objetivos de la Nefrotección?
A. HbA1C ≤ 6,5%
B. PA < 130/80 mmHg
C. Proteinuria > 0,5gr/24h
D. Cese de hábito tabáquico.
ANSWER: C
¿Cuál de los siguientes medicamentos son considerados primera elección para la nefroprotección en
ERC?
A. Diuréticos y Antagonistas de calcio.
B. ARA II y Diureticos.
C. IECA y B bloqueantes.
D. IECA y ARA II
ANSEWR: D
¿Cuál de los siguientes NO corresponde a efectos renales de las IECA?
A. Bloqueo de efectos directos de la angiotensina II
B. Bloqueo del efecto vasoconstrictor de las catecolaminas.
C. Contribuye a mantener la TPG y la Volemia.
D. Disminuye la degradación de cininas.
ANSWER: C
¿Cuál de los siguientes NO corresponde a efectos renales de los ARA II?
A. Vasoconstricción de arterias preglomerulares.
B. Disminución del flujo medular.
C. Contracción de células mesangiales.
D. Actúa sobre la perfusión renal limitando la vasoconstricción y mejora la perfusión y aumenta la
formación de prostaglandinas.
ANSWER: D
Banco de preguntas grupo 11

El síndrome hepatorrenal (SHR) es una forma característica de insuficiencia renal que ocurre en los
pacientes con cirrosis hepàtica, de carácter funcional que aparece como consecuencia de una intensa:
A. vasoconstricción de la circulación renal secundaria a la disfunción circulatoria sistémica
B. vasodilatacionn de la circulación renal secundaria a la disfunción circulatoria sistémica
C. entidad que aparece en fases avanzadas de la cirrosis
D. alteración de la función renal
ANSWER :A
Cuál es el incremento de Creatinina sérica para que se manifieste como (SHR) tipo 1
A. >1.5 mg/dl
B. >1.5 mg /dl
C. >2.5mg/dl
D. >3mg /dl
ANSWER :C
Hay dos tipos de SHR: Como se manifiesta el SHR tipo 1 :
A. Inicio rápido, evolución progresiva y severa, con una supervivencia media de 15 días sin
tratamiento
B. Menos grave y progresivo, con una supervivencia media de 6 meses.
C. Evolución progresiva con una supervivencia media de 4 semanas
D. Evolución menos progresiva con una supervivencia media de 1 año
ANSWER :A
¿Como subdividió el club internacional de ascitis los criterios diagnósticos?
A. 2 tipos: de acuerdo a su velocidad y severidad de instauración
B. 3 tipos: de acuerdo a su velocidad, severidad de instauración y edad
C. Solo es de 1 tipo: de acuerdo a su velocidad
D. Solo es de 1 tipo: severidad de instauración
ANSWER: A
¿En qué año el Club Internacional de Ascitis estableció los criterios diagnósticos?
A.1994
B.1995
C.1996
D.1993
ANSWER: A
Banco de preguntas grupo 11

¿Cuál es el tratamiento farmacológico para el Hepatorrenal?


A. Vasoconstrictores y albumina
B. Trasplante hepático
C.Derivacion percutánea portosistémica intrahepática
D. DIALISIS
ANSWER: A
Señale. En el Síndrome Hepatorrenal. ¿Cuál es el primer mecanismo de resistencia vascular sobre la
presión arterial, a comienzos de la cirrosis en el paciente?
A. Disminución de la presión oncótica en el riñón
B. Aumento del gasto cardíaco
C. Aumento de endocannabinoides en el hígado
D. Disminución de la creatinina sérica
ANSWER: B
Señale. En el Síndrome Hepatorrenal. ¿Cuál es el primer mecanismo vasoconstrictor que se activa
después de fallar el aumento del gasto cardíaco en el paciente con cirrosis?
A. Hipersecreción de vasopresina
B. Aumento del gasto cardíaco
C. Sistema nervioso simpático
D. Sistema Renina – Angiotensina - Aldosterona
ANSWER: D
Señale. En el Síndrome Hepatorrenal. ¿Cuál es el objetivo a mantener en el paciente cirrótico, cuando
ya se han gastado todos sus mecanismos vasoconstrictores de compensación?
A. Mantener el volumen y la presión arterial constantes
B. Aumentar el gasto cardíaco
C. Disminuir la hipertensión portal
D. Evitar una complicación, como la peritonitis
ANSWER: A
Señale. En el Síndrome Hepatorrenal. ¿Cuál es la triada patológica que se ocasiona por retener sodio y
liberar agua a causa de la vasoconstricción renal?
A. Insuficiencia renal, diaforesis, alza térmica
B. Taquicardia, ascitis, fatiga
C. Ascitis, Edema, Hiponatremia dilucional
D. Edema, Shock séptico, Cirrosis
ANSWER: C
Banco de preguntas grupo 11

1. ¿Cuál es el agente etiológico más prevalente en un ITU de ámbito comunitario?


a) Proteus mirabilis
b) Escherichia coli
c) Proteus mirabilis
d) Klebsiella spp.
ANSWER: B
2. ¿Cuál es el agente etiológico más prevalente en un ITU en varones ancianos mayores?
a) Enteroccocus fecalis
b) Proteus mirabilis
c) Escherichia coli
d) Klebsiella spp.
ANSWER: A
3. Seleccione lo correcto. ¿Cuál es la vía MAS COMÚN de los gérmenes en una ITU?
a) Vía hematógena
b) Vía linfática
c) Vía ascendente
d) Vía oral
ANSWER: C
4. Seleccione la opción correcta. En la bacteriuria asintomática en mujeres gestantes ¿cuál es la
mejor opción terapéutica?
a) Al ser una colonización y no una infección, no requiere antibiótico terapéutico
b) No se debe tratar por que provocaría complicaciones fetales (RN de bajo peso y prematuridad)
c) Se recomienda Nitrofurantoína en el primer trimestre del embarazo
d) Se recomienda un tratamiento corto con uso seguros de betalactámicos según el germen
aislado y su antibiograma
ANSWER: D
5. Uno de los microorganismos que usualmente produce una pielonefritis por diseminación por
vía hematógena es:
a) S. aureus
b) K. pneumoniae
c) E. coli
d) P. mirabilis
ANSWER: A
Banco de preguntas grupo 11

6. Se define como infección recurrente del tracto urinario a la presencia de:


a) 2 o más ITU sintomáticas en 6 meses
b) 3 o más ITU sintomáticas en 12 meses
c) 2 ITU sintomáticas en un periodo de 12 meses
d) a y b son correctas
ANSWER: D
7. El microorganismo predominante que produce pielonefritis por vía ascendente es:
a) K. pneumoniae
b) P. mirabilis
c) E. coli
d) S. aureus
ANSWER: C
8. Seleccione los factores predisponentes de ITUs complicadas excepto
a) Diabetes
b) Obstrucción urinaria
c) Hepatitis
d) Reflujo vesicouteral
ANSWER: C
9. Como tratamiento empírico de ITUS complicadas se debe evitar:
a) Clavulánico
b) Amoxicilina
c) fluroquinaloas
d) todas las anteriores
ANSWER: D
10. Seleccione los enteropatógenos más frecuentes en ITUs
a) enterobacterias
b) E COLI
c) Proteus klebsiella
d) Todas las anteriores
ANSWER: D
Banco de preguntas grupo 11

En la ITU complicada se presentan los siguientes signos y síntomas, excepto


a) Disuria
b) Urgencia miccional
c) Polidipsia
d) Polipiuria
ANSWER: C
¿Cuál de los siguientes no corresponde a un cambio a nivel renal experimentado en el embarazo?
A. Aumento del tamaño del riñón
B. Hidronefrosis
C. Aumento del filtrado glomerular
D. Disminución de la actividad del SRAA.
ANSWER: D
Señale la incorrecta respecto a la hidronefrosis del embarazo:
A. Es más prominente del lado derecho
B. Se cree que es debido a la peristalsis disminuida por la progesterona
C. Se puede visualizar en el segundo trimestre del embarazo
D. Se resuelve en la primera semana después del parto.
ANSWER: D
¿Por qué se produce el aumento del gasto cardíaco durante la gestación? Señale la incorrecta:
A. Aumento de la masa eritrocitaria
B. Aumento del volumen sistólico
C. Reducción de la resistencia periférica
D. Aumento de la frecuencia cardiaca
ANSWER: A
Respecto a la anemia fisiológica del embarazo señale la incorrecta:
A. Se produce debido a un aumento del volumen plasmático sin un aumento de la masa
eritrocitaria
B. El aumento del volumen plasmático favorece las hemorragias postparto
C. El aumento del volumen plasmático provee un mayor flujo renal y placentario
D. Facilita la llegada de nutrientes al feto
ANSWER: B
Banco de preguntas grupo 11

Señale el enunciado incorrecto respecto a preeclampsia/eclampsia:


A. La más frecuente de las enfermedades hipertensivas del embarazo es la
preeclampsia/eclampsia.
B. La preeclampsia es la hipertensión que aparece después de las 20 semanas de gestación
acompañada de proteinuria significativa.
C. Se denomina eclampsia cuando la hipertensión se acompaña de convulsiones y/o coma.
D. Es una enfermedad exclusiva del embarazo humano, sin predisposición familiar.
ANSWER: D
De acuerdo con los signos y síntomas de preeclampsia/eclampsia, señale lo incorrecto:
A. La preeclampsia puede ser asintomática o causar edema o un aumento importante del peso.
B. El edema en los sitios no declive, como el rostro o las manos (la paciente no puede quitarse los
anillos de los dedos), es más específico que el edema en las regiones declive.
C. La actividad refleja puede estar aumentada, lo que indica irritabilidad neuromuscular, que
puede progresar en convulsiones (eclampsia).
D. No hay petequias y otros signos de coagulopatía.
ANSWER: D
En ausencia de proteinuria, la preeclampsia también se diagnostica si las mujeres embarazadas tienen
hipertensión de reciente comienzo junto con aparición de cualquiera de los siguientes, excepto:
A. Trombocitopenia (plaquetas < 100.000/microL)
B. Insuficiencia renal (creatinina sérica > 1,1 mg/dL o duplicación de la creatinina en suero en
mujeres sin enfermedad renal)
C. Deterioro de la función hepática (transaminasas > 2 veces el valor normal)
D. Sin síntomas cerebrales o visuales.
ANSWER: D
¿Cuál de estas no es una complicación del síndrome HELLP?
A. Isquemia hacia los órganos
B. Convulsiones.
C. Anemia
D. Trombosis venosa
ANSWER: D
Banco de preguntas grupo 11

Señale cuál de las siguientes manifestaciones no corresponde al síndrome de HELLP:


A. Hipertensión
B. Proteinuria
C. Elevación de enzimas hepáticas
D. Hiponatremia
ANSWER: D
En el 25% de los casos de síndrome de HELLP puede encontrarse:
A. Hiponatremia
B. Trastornos de visión y audición
C. Epigastralgia
D. Hipertensión
ANSWER: B
Defina. Qué es la poliquistosis renal
A: Es una enfermedad genética común que consiste en la aparición progresiva de lesiones quísticas en
los riñones
B: Los quistes remplazan el parénquima renal
C: Se produce una enfermedad renal crónica en estadio V
D: Todas son correctas
ANSWER: D
Cuáles son los patrones de herencia que se presenta en la poliquistosis renal
A: Autosómica Dominante (PKD1 y PKD2)
B: Autosómica recesiva (PKHD1).
D: Autosómica dominante y recesiva PKDY2 y PKHD3
E: Solo a y b son correctas
ANSWER: E
Cuáles son los cromosomas involucrados en la poliquistosis renal
A: el gen PKD1, localizado en el cromosoma 16
B: En el gen PKD2, localizado en el cromosoma 4
C: Tercer gen, PKD3, que aún no ha sido identificado
E: Todas son correctas
ANSWER: E
Banco de preguntas grupo 11

Prevalencia de la Poliquistosis Renal en el Mundo


A: 1 de cada 1000 personas
B: 1 de cada 500 personas
C: 10 de cada 1000 personas
D: 10 de cada 500 Personas
ANSWER: A
¿En la etiología de la Poliquistosis Renal, cual es el Gen que causa mayor severidad en la presentación
clínica?
A: PKD2
B: PKD1
C: PKD3
D: PKD4
ANSWER: B
¿En pacientes que tienen mutación del Gen PKD1 sus riñones se encuentran?
A: Aumentados de Tamaño
B: Disminuidos de Tamaño
C: Presenta una función renal disminuida
D: Presentan quistes
ANSWER: A
¿CUÁL es el gen y la proteína afectados en la PQRAR?
A: PKHD1 Fibrocistina
B: PKD 1/PKD 2 - Policistina 1/ Policistina 2
C: NPHP1 - Nefrocistina
D: HNF-1b - TCF2
E: No es una enfermedad genética
ANSWER: A
¿QUÉ debemos vigilar en la displasia multiquísica renal?
A: Tensión arterial
B: Malformaciones genitourinarias contralaterales
C: Crecimiento compensador del riñón contralateral
D: Todas las anteriores
ANSWER: D
Banco de preguntas grupo 11

¿Cuál es la EDAD más frecuente de presentación de la PQRAD?


A: 20-30 años, que son la 3ª -4ª década de la vida.
B: Intraútero.
C: Periodo de lactante (< 2 años).
D: Tercera edad
E: Infancia (2-12 años).
ANSWER: A
¿Cuál es la EDAD más frecuente de presentación de la PQRAD?
A: 20-30 años, que son la 3ª -4ª década de la vida.
B: Intraútero.
C: Periodo de lactante (< 2 años).
D: Tercera edad
E: Infancia (2-12 años).
ANSWER: A
1.) ¿El síndrome cardiorrenal tipo 1 a que hace referencia?
a.) Deterioro crónico de la función renal como consecuencia de un deterioro crónico de la
función cardiaca.
b.) Disfunción cardiaca aguda induce lesión o disfunción renal aguda.
c.) Enfermedad cardiovascular que acompaña a la ERC.
d.) Lesión concomitante renal y cardiaca aguda o crónica secundaria a una enfermedad
sistémica aguda o crónica.
2.) Señale el enunciado correcto.
a.) El deterioro agudo de la función cardiaca produce alteraciones hemodinámicas, entre las
que se incluyen reducción del volumen sistólico y del gasto cardiaco.
b.) El deterioro agudo de la función cardiaca produce alteraciones hemodinámicas, entre las
que se incluyen disminución de la presión en aurícula derecha y congestión venosa, que
conducen a un descenso del filtrado glomerular.
c.) El deterioro agudo de la función cardiaca produce alteraciones hemodinámicas, entre las
que se incluyen aumento del volumen sistólico y del gasto cardiaco.
d.) Ninguna.
Banco de preguntas grupo 11

¿Qué fármaco se utiliza como primera línea en el tratamiento del síndrome cardiorrenal tipo 1?
a.) IECAs.
b.) ARA II.
c.) Diuréticos.
d.) Beta-bloqueantes.
3.) De acuerdo a los tipos de Síndrome Cardiorenal, indique cuál pertenece al tipo 4:
a. Se caracteriza por afectación cardiovascular en pacientes con enfermedad renal crónica
b. Se caracteriza por la presencia simultánea de disfunción renal y cardiaca, asociada a
trastornos sistémicos agudos o crónicos
c. Deterioro crónico de la función renal como consecuencia de un deterioro crónico de la
función cardiaca
d. Disfunción cardiaca aguda induce lesión o disfunción renal aguda
4.) Señale lo correcto de acuerdo con el síndrome cardiorrenal secundario o Tipo 5, se caracteriza
por:
a. Choque séptico y el hemorrágico de origen traumático
b. Las quemaduras múltiples, el lupus eritematoso sistémico, la diabetes mellitus
c. activación del sistema nervioso simpático conlleva una estimulación del sistema renina-
angiotensina-aldosterona que contribuye a la liberación de angiotensina II
d. A y B son correctas
5.) De acuerdo con las Estrategias terapéuticas en el manejo del síndrome cardiorrenal, indique cual
es el blanco terapéutico en relación a la anemia y alteración del hierro
a. Hemodiálisis de alto flujo
b. Inhibidores de la enzima convertidora de angiotensina
c. Carnitina
d. Inhibidores de renina
6.) De acuerdo con los criterios de diagnósticos de la insuficiencia cardiaca y renal. Indique cuales se
presentan en la insuficiencia cardiaca crónica:
a. Vértigo, síncope, dolor torácico
b. Fiebre y malestar, deshidratación, edema
c. Hipertermia, hipotermia, fiebre y escalosfrios
d. Ortopnea, disnea de esfuerzo, ritmo de galope, hepatomegalia
Banco de preguntas grupo 11

8). EL SINDROME CARDIORRENAL TIPO 1 SE DA :


a) activación neural , alteraciones hemodinámicas y congestión venosa , reacción del eje hipotálamo -
hipofisis , inflamacion de señalización
b) activacion neumoral , alteraciones hemodinámicas y congestión venosa , reacción del eje hipotálamo
-hipófisis , inflamación de señalización
c) activación numeral , alteraciones dinámicas y congestion arteriosa , reacción del eje hipotálamo -
hipófisis , inflamación de señalización
d) ninguna de las anteriores
9). SINDROME CARDIORRENAL TIPO 2 EL TRATAMIENTO SE BASA
a) Se basa en una combinación de dieta, ejercicio y tratamiento de las enfermedades subyacentes, Los
betabloqueantes , Los IECAs
b)los vasodilatadores es disminuir las resistencias periféricas con el fin de reducir la regurgitación mitral,
aumentar el flujo sanguíneo y disminuir las presiones de llenado
c)diuréticos de asa y vasodilatadores, junto con el tratamiento de las arritmias y la suspensión de los
agentes nefrotóxicos.
D)Ninguno de los anteriores
10. EN LA FISIOPATOLOGÍA CARDIORRENAL TIPO 3 SE DA
a) El daño renal agudo se asocia a activación neuroendocrina y del sistema inmunitario. La activación
del sistema nervioso simpático es característica tanto en el FRA como en la insuficiencia cardiaca aguda
b) El deterioro agudo de la función cardiaca produce alteraciones hemodinámicas, incluyen
reducción del volumen sistólico y del gasto cardiaco, elevación de la presión en aurícula
derecha
c) Las anormalidades neurohumorales, con excesiva producción de mediadores
vasoconstrictores (epinefrina, angiotensina y endotelina) y alteración de la liberación de
vasodilatadores endógenos
d) Ninguna de los anteriores
La definición de síndrome nefrítico consiste en un cuadro caracterizado por las siguientes
características, excepto...
A. Hematuria
B. HTA
C. Polaquiuria
D. Edema
ANSWER; C
Banco de preguntas grupo 11

Todas son causas primarias de glomerulonefritis, excepto...


A. Glomerulonefritis postinfecciosa
B. Membrano proliferativa
C. Mesangial IgA
D. Goodpasture
ANSWER; D
En la endocarditis bacteriana aguda, están presente los siguientes depósitos inmunitarios, excepto…
A. IgA
B. IgM
C. Fracción C3 del complemento
D. IgG
ANSWER; A
Señale lo INCORRECTO, los criterios diagnósticos de sx nefrítico son:
A. Edema
B. Elevación creatinina y urea por días o meses
C. Hematuria menos de 5 hematíes por campo
D. Hipertensión
ANSWER; C
Las Glomerulonefritis asociadas a ANCA – C son:
A. Granulomatosis de Wegener
B. Granulomatosis de Wegener y Sx Goodpasture
C. Granulomatosis de Wegener y Poliangeitis microscópica
D. Granulomatosis de Wegener y Sx Churg Strauss
ANSWER; D
El tratamiento de elección para las sobrecargas leves o moderadas en el contexto de una
glomerulonefritis es:
A. Furosemida, 2-5 mg/kg/día por vía oral
B. Furosemida, 0,5-2 mg/kg/día por vía oral
C. Furosemida, 3 - 8 mg/kg/día por vía oral
D. Furosemida, 0,3 – 0,8 mg/kg/día por vía oral
ANSWER; B
Banco de preguntas grupo 11

En la glomerulonefritis, si a pesar de la restricción hidrosalina y el uso de diuréticos no se consigue


controlar la presión arterial, ¿qué fármacos se pueden asociar?
A. IECA
B. Hidralazina
C. Hidralazina o Nifedipino
D. IECA o Hidralazina
ANSWER; C
Porque en la glomerulonefritis se produce reducción de los niveles séricos de complemento.
a) porque se depositan cristales
b) por aumento de los inmunocomplejos
c) deposito en la membrana basal de inmunocomplejos
d) ninguno es correcto
ANSWER: C
Cuando existe un aumento en el diámetro de los poros de la membrana basal en el síndrome nefrítico
que manifestaciones presenta en la orina.
a) hematuria- oliguria- hiperproteinuria
b) hematuria-proteinuria cilindros de GR
c) hematuria- anuria – proteinuria
d) ninguna es correcta
ANSWER: B
En el síndrome nefrítico cuando disminuye la superficie de filtración glomerular presenta
a) disminuye la tasa de filtración- oliguria- azoemia- edema
b) aumenta la tasa de filtración- anuria- proteinuria- edema
c) disminuye la tasa de filtración- poliuria- azoemia- sin edema
d) ninguna es correcta
ANSWER: A
1. ¿Cuál de las siguientes afirmaciones es correcta respecto a los criterios de diabetes?
a. Glucemia basal >= 126 mg/dl
b. Glucemia >= 100mg/dl dos horas tras la SOG con 75 g de glucosa
c. HbA1C >= 65%
d. Paciente sintomático con glucemia de 100 mg/dl
ANSWER: A
Banco de preguntas grupo 11

2. ¿En qué pacientes usted pensaría una diabetes de tipo MODY?


a. Adulto de 40 – 60 años, anticuerpos negativos
b. Adulto joven de 25 – 40 años, anticuerpos negativos
c. Joven de 18 – 25 años, anticuerpos negativos
d. Niños de 0 – 18 años, anticuerpos negativos
ANSWER: B
3. ¿Cuál es el diagnostico apropiado para nefropatía diabética?
a) Pedir microalbuminuria a los 5 años del diagnóstico de DM1
b) Pedir microalbuminuria inmediatamente después del diagnostico de DM2
c) Pedir microalbuminuria a los 5 años del diagnóstico de DM2
d) A Y B son correctas
ANSWER: D
4. ¿Cuál es el valor normal de la albuminuria?
a) > 30 mg/g
b) <30 mg/g
c) 60 – 100 mg/g
d) 90 mg/g
ANSWER: B
5. Valor normal de la tasa de filtrado glomerular (TFG)
a) Mayor o igual a 90 ml/min/1,73m2
b) 60 ml/min/1,73m2
c) 30 – 60 ml/min/1,73m2
d) 80 ml/min/1,73m2
ANSWER: A
6. El primer cambio funcional en la nefropatía diabética és:
a) Hiperfiltración
b) Albuminuria
c) Proteinuria
d) Disminución de la TFG
ANSWER: A
Banco de preguntas grupo 11

7. ¿Cuál de los siguientes no es un cambio estructural en una afección renal por diabetes?
a) Aumento del ovillo glomerular
b) Engrosamiento de la membrana basal glomerular
c) Glomeruloesclerosis
d) Aumento del tamaño tubular
ANSWER: D
8. ¿La fase de nefropatía incipiente se caracteriza por?
a. Hiperfiltración
b. Albuminuria
c. Proteinuria
d. Disminución de la TFG
ANSWER: B
9. ¿En el manejo del diabético tipo 2 con insuficiencia renal moderada (creatinina sérica > 2
mg/dl) cuál de las siguientes afirmaciones es falsa?
a) Es más importante el uso, en sí mismo, de un nefroprotector como IECAs que el control
estricto de la PA.
b) Es bastante frecuente que la causa de la insuficiencia renal no sea la nefropatía
diabética específica.
c) Habitualmente se requiere tratamiento farmacológico combinado para el control
estricto de la PA.
d) Las necesidades de insulina suelen disminuir porque, aunque se incrementa la
resistencia insulínica predomina la acumulación insulínica por el fallo renal.
ANSWER: A
10. ¿En el manejo del paciente diabético tipo 2 con nefropatía incipiente (microalbuminuria) cuál
de las siguientes afirmaciones es cierta?
a) Con el uso de IECAs a dosis bajas no es preciso hacer un control precoz de creatinina y
potasio séricos.
b) El abandono del tabaco probablemente influye favorablemente sobre la nefropatía del
diabético tipo 2.
c) En el diabético tipo 2 con microalbuminuria el objetivo de control glucémico debe ser
HbA1c <8,5%.
d) Hay que evitar el uso de tiazidas en el diabético tipo 2 con microalbuminuria.
ANSWER: C
Banco de preguntas grupo 11

Elija la correcta definición de diálisis peritoneal:


A. terapia de reemplazo renal basada en la infusión de una solución estéril en la cavidad peritoneal
a través de un catéter y permite la eliminación de solutos y agua utilizando la membrana
peritoneal como superficie de intercambio.
B. Convencional, de alta eficacia, de alto flujo, tradicional
C. Tratamiento inmunosupresor debe ser individualizado buscando la sinergia inmunosupresora y
el mejor perfil de seguridad, y debe adaptarse a las diferentes etapas.
D. Extracción de un riñón de un individuo previamente sano (cadáver o de vivo) y su colocación en
un enfermo con insuficiencia renal crónica.
ANSWER: A
Señale cuál de las siguientes opciones contiene contraindicaciones relativas de la diálisis peritoneal:
A. Limitación severa de capacidad respiratoria, imposibilidad de autodiálisis sin apoyo, enfermedad
inflamatoria intestinal activa
B. Negativa del paciente, múltiples adherencias quirúrgicas
C. Múltiples hernias/fugas de repetición
D. Enfermedad psiquiátrica grave
ANSWER: A
Señale cuál de las siguientes alternativas corresponde a las complicaciones de diálisis peritoneal:
A. Esclerosis peritoneal encapsulante, disfunción del catéter, edema y falla de ultrafiltración
B. Enfermedad psiquiátrica grave
C. Limitación severa de capacidad respiratoria, imposibilidad de autodiálisis sin apoyo, enfermedad
inflamatoria intestinal activa
D. Negativa del paciente, múltiples adherencias quirúrgicas
ANSWERS: A
¿Cuál de las siguientes es una indicación inmediata de hemodiálisis de urgencia en un paciente con
insuficiencia renal?
A. Sodio urinario menor a 120 mEq/L
B. Creatinina plasmática mayor a 6 mg/dl
C. Hiperkalemia grave que no responde a tratamiento médico
D. Oliguria menor a 500 ml/día
ANSWER: C
Banco de preguntas grupo 11

En cuanto a la hemodiálisis convencional nos referimos a:


A. dializadores de baja permeabilidad y baja superficie (1,2 – 1,6 m2) flujo sanguíneo de 200-300
ml/min, flujo de líquido de diálisis 500ml/min
B. mayor aclaramiento de solutos. Suoerficie de 1,8-2,2 m2, flujo sanguíneo de 300-400 ml/min,
flujo de líquido de diálisis 700-1000ml/min. Requiere control preciso de ultrafiltración
C. empleo de lializadones de alta permeabilidad (mayor aclaramiento). Requiere membranas
biocompatibles. Riesgo de bacteremia
D. dializadores de baja permeabilidad y baja superficie (2,5 – 5,6 m2) flujo sanguíneo de 100-200
ml/min, flujo de líquido de diálisis 800ml/min
ANSWER: A
Los tipos de hemodiálisis se clasifican en los siguientes excepto
A. convencional
B. de alta eficacia
C. de alto flujo
D. tradicional
ANSWER: D
¿Cuánto es el flujo sanguíneo en HD convencional?
A. 100-200ml/min
B. 200-300 ml/min
C. 300-400ml/min
D. 700-1000mi/min
ANSWER: B
¿De los criterios de exclusión de donantes vivos, cual no es considerada como contraindicación
absoluta?
A. Proteinuria > 300mg/24 h
B. FG anormal para la edad
C. Hematuria
D. Infección por VIH
ANSWER: D
Banco de preguntas grupo 11

De las siguientes son contraindicaciones relativas de criterios de donantes vivos, excepto:


A. Trastornos psiquiátricos
B. Infección activa crónica (Hepatitis B/C)
C. Obesidad
D. Tumor maligno creciente.
ANSWER: D
Los siguientes son criterios de AMSTERDAM de aceptación de donante vivo, excepto:
A. IMC < 35 kg/m2
B. Albuminuria > 300 mg/24h
C. Filtrado glomerular > 80ml/min/1.73m2
D. Ausencia de antecedente de DM
ANSWER: B
¿CUÁL DE LOS SIGUIENTES NO ES UN CRITERIO CLINICO Y BIOQUÍMINO DEL SIÍDROME NEFRÓTICO?
A. Anasarca
B. Hipoproteinemia
C. Anemia ferropénica
D. Cociente proteinuria/creatinuria
E. Hipercolesterolemia
ANSWER: C
¿CUÁL DE LOS SIGUIENTES SI ES UN METODO DE MEDICIÓN DE PROTEINURIA EN EL SÍNDROME
NEFRÓTICO?
A. Eco doppler
B. Colirométrico
C. Coagulación
D. Iones en biometría
E. Recuento leucocitario
ANSWER: B
Banco de preguntas grupo 11

¿CUÁL DE LOS SIGUIENTES NO ES UN CRITERIO PARA REALIZAR UNA BIOPSIA RENAL EN NIÑOS EN
CASO DE SOSPECHA DEL SÍNDROME NEFRÓTICO?
A. Insuficiencia renal
B. Nefritis familiar
C. Cortico resistencia
D. Macro/micro hematuria
E. Hemoglobina < 8
ANSWER: E
¿CUÁL DE LOS SIGUIENTES NO ES UN DIAGNOSTICO DIFERENCIAL DEL SÍNDROME NEFRÓTICO?
A. Enteropatía perdedora de proteínas
B. Insuficiencia hepática
C. Glomerulonefritis aguda o crónica
D. Malnutrición protéica
E. Escoliosis
ANSWER: E
SELECCIONE LA INCORRECTA: ¿EN CUÁL DE LOS SIGUIENTES CASOS SE DEBE PROCEDER A LA
HOSPITALIZACION EN CASO DE SOSPECHA DE SINDROME NEFRÓTICO?
A. Presencia de edema severo
B. Sobreinfección
C. Oliguria
D. Diabetes diagnosticada hace más de 5 años
E. Azoemia
ANSWER: D
Todo lo que sigue acerca del síndrome nefrótico en la infancia, es cierto EXCEPTO:
A. Colesterol sérico elevado.
B. El 85% experimenta cambios mínimos de la enfermedad.
C. Reabsorción reducida de sodio por el riñón.
D. Triglicéridos séricos elevados.
E. La hipoalbuminemia es la causa de la hipoproteinemia.
ANSWER:C
Banco de preguntas grupo 11

Señala cuál entre las siguientes nefropatías primarias NO se presenta clínicamente cómo síndrome
nefrótico más que de modo excepcional:
A. La glomerulopatía membranosa.
B. La glomeruloesclerosis focal y segmentaria.
C. La enfermedad de cambios mínimos.
D. La nefropatía por IgA (enfermedad de Berger).
E. La nefropatía asociada con el VIH, sin o con colapso glomerular.
ANSWER:D
En un paciente con síndrome nefrótico, las medidas generales no específicas, para corregir la proteinuria
incluye uno de los siguientes procedimientos:
A. Dieta hiperproteica.
B. Diuréticos y/o b-bloqueantes.
C. Inhibidores del enzima conversor de la Angiotensina (IECAs).
D. Calcioantagonistas no dihidropiridínicos.
E. Corticoides por vía sistémica.
ANSWER:C
Todas las nefropatías primitivas enumeradas a continuación se manifiestan típicamente como síndrome
nefrótico, EXCEPTO una. Señálela:
A. Glomerulonefritis de mínimos cambios.
B. Glomerulonefritis segmentaria y focal.
C. Glomerulonefritis proliferativa focal.
D. Glomerulonefritis membranosa.
E. Glomerulonefritis membranoproliferativa.
ANSWER: C
¿Cúal enunciado no entra en el criterio diagnóstico de nefroesclerosis hipertensiva?
A. Mayores de 55 años de edad
B. Hipertensión arterial de larga evolución
C. Insuficiencia renal de evolución rápida
D. Retinopatía, hipertrofia ventricular izquierda
ANSWER: C
Banco de preguntas grupo 11

2.-Señale el enunciado correcto respecto a nefroescleorosis benigna


A. Arteriola con deposito hialino que afecta a todo el perímetro del vaso
B. Necrosis fibrinoide de las arteriolas aferentes
C. El diagnóstico está íntimamente ligado a los resultados del examen del fondo de ojo
D. Se detectan hemorragias y exudados, se habla de HTA acelerada.
ANSWER : A
3.- Patología relacionada al endurecimiento renal y es el resultado final de la sustitución del
parénquima renal normal por un tejido más denso con abundante componente colágeno producto de
la HTA crónica
A. Litiasis renal
B. Nefropatía hipertensiva
C. Hidronefrosis
D. Hipertensión renovascular
ANSWER: B
4.- Cuando existe aumento pasajero de 20mmHg en la presión sistólica que sucede en el consultorio
médico. Se denomina como:
A. HTA RESISTENTE
B. HTA SISTOLICA ASILADA
C. HTA DE BATA BLANCA
D. HTA ENMASCARADA
ANSWER: C
5.- Cuando existe una presión arterial ˃ 140/90 a pesar del uso 3 de medicamentos antihipertensivos a
dosis máxima e incluido un diurético. Se denomina:
A. HTA RESISTENTE
B. HTA SISTOLICA ASILADA
C. HTA DE BATA BLANCA
D. HTA ENMASCARADA
ANSWER: A
6.- Si existe un aumento de la PA marcado que provoca daño a órganos diana que requieren tto
inmediato. Se trata de:
A. HTA RESISTENTE
B. HTA SISTOLICA ASILADA
C. EMERGENCIA HIPERTENSIVA
D. HTA ENMASCARADA
Banco de preguntas grupo 11

ANSWER: C
7.- Cuando existe una PA normal en el consultorio, pero por medición ambulatoria de la PA se
encuentra elevada en más de 20 mmHg.
A. HTA RESISTENTE
B. HTA SISTOLICA ASILADA
C. EMERGENCIA HIPERTENSIVA
D. HTA ENMASCARADA
ANSWER: D
8.- Cual de las siguientes no es una causa secundaria de hipertensión arterial
A. QUISTES RENALES
B. UROPATIA OBSTRUCTIVA
C. MICOSIS
D. ALDOSTERONISMO PRIMARIO
ANSWER: C
9.- Características de los pacientes que deben suscitar la sospecha de HTA secundaria son las
siguientes, excepto:
A. PACIENTES JÓVENES MENORES A 40 AÑOS, CON HTA DE GRADO 2
B. CARACTERÍSTICAS CLÍNICAS DE APNEA OBSTRUCTIVA DEL SUEÑO
C. HTA RESISTENTE
D. EDEMA DE MIEMBROS SUPERIORES
ANSWER: D
10.- Cual de los siguientes métodos de diagnóstico no se usa para la hipertensión renovascular
A. RAYOS X
B. GAMMAGRAFIA CON CAPTOPRIL
C. ECO-DOPPLER
D. RM ANGIOGRAFICA
ANSWER: A
Cuáles son las Indicaciones orientativas del empleo de bloqueantes del SRAA en renoproteccion de
ERC:
A. ERC proteinurica (> 30 mg/24 hs)
B. ERC no proteinúrica si HTA o diabetes mellitus
C. ERC proteinurica (> 30 mg/24 hs)
D. A Y B
Banco de preguntas grupo 11

ANSWER: D
Cardio protectores de primera linea:
A. β bloqueantes
B. α bloqueantes
C. Los inhibidores de la renina
D. Todos
ANSWER: A
En la ERCA deben emplearse preferentemente:
A. Diuréticos tiazídicos
B. Diuréticos de asa
C. Diuréticos antialdosterónicos
D. Manitol
ANSWER: B
La asociación de alteraciones metabólicas más característica que podemos encontrar en presencia de
ERC en estadios avanzados es:
A. Hiperpotasemia, hipofosfatemia, hipercalcemia, acidosis metabólica
B. Hipopotasemia, hipofosfatemia, hipocalcemia, acidosis metabólica
C. Hiperpotasemia, hiperfosfatemia, hipocalcemia, acidosis metabólica
D. Hiperpotasemia, hiperfosfatemia, hipocalcemia, alcalosis metabólica
ANSWER:C
Cual es la causa más frecuente de ERC.
A. Diabetes mellitus
B. Hipertensión arterial
C. Glomerulonefritis
D. Riñón quístico
E. Infecciones urinarias de repetición
ANSWER: A
¿EN EL ANCIANO LA ERC SE DIAGNOSTICA POR?
e) CIFRAS NORMALES DE UREA
f) CIFRAS BAJAS DE UREA
g) CIFRAS BAJAS DE CRATINICA
h) CIFRAS ELEVADAS DE UREA Y CRATININA
ANSWER: D
Banco de preguntas grupo 11

¿CUÁNDO LA TFG ES MENOR O IGUAL A 15ML/MIN/1.73M2?


A. SE PRESENTA PROTEINURIA
B. SE PRODUCE LA UREMIA
C. PRODUCE HIPERTENCION
D. PRODUCE ORINA DILUIDA
ANSWER: B
LA ANEMIA EN LA ERC ES DE TIPO
e) NORMOCRONICA Y NORMOCITICA
f) HIPERCROMICA
g) SOLO NORMOCRONICA
h) SOLO NORMOCITICA
ANSWER: A
La ERC en el adulto se define como
A. La presencia de una alteración estructural o funcional renal (sedimento, imagen, histología) que
persiste más de 6 semnas, con o sin deterioro de la función renal; o un filtrado glomerular (FG) < 60
ml/min/1,73 m2 sin otros signos de enfermedad renal.
B. La presencia de una alteración estructural o funcional renal (sedimento, imagen, histología) que
persiste más de 3 meses, con o sin deterioro de la función renal; o un filtrado glomerular (FG) < 90
ml/min/1,73 m2 sin otros signos de enfermedad renal.
C. La presencia de una alteración estructural sin deterioro de la función renal; o un filtrado glomerular
(FG)  60 ml/min/1,73 m2 conn otros signos de enfermedad renal.
D. La presencia de una alteración estructural o funcional renal (sedimento, imagen, histología) que
persiste más de 3 meses, con o sin deterioro de la función renal; o un filtrado glomerular (FG) < 60
ml/min/1,73 m2 sin otros signos de enfermedad renal.
ANSWER:D
Cual no es marcadores de daño renal:
A. Proteinuria elevada
B. Alteraciones en el sedimento urinario
C. Alteraciones estructurales histológicas
D. Proteinuria Disminuida
E. ANSWER:D
Entre los principales objetivos del tratamiento de la insuficiencia renal aguda se encuentran?
A. La optimización de la hemodinámica, La corrección de desequilibrios hidroelectrolíticos.
B. La optimización de la hemodinámica, la disminución de la tasa de filtración.
C. La corrección de desequilibrios hidroelectrolíticos, la disminución de la tasa de filtración.
Banco de preguntas grupo 11

D. Todas.
ANSWER A
En La Insuficiencia Renal Aguda causada por esclerodermia que medicamentos se debe suministrar.
A. Inmunodepresores
B. plasmaféresis
C. Inhibidores de la ACE
D. AINES
ANSWER C
Al hablar del aporte nutricional en la insuficiencia renal aguda para prevenir las complicaciones, nos
referimos a?
A. Restriccion de proteinas a menos de 0.8 g/kg de peso en pacientes no hipercatabolicos.
B. Hidratos de carbono mínimo de 100 g /dia
C. Calorias entre 20 a 30 kcal /kg de peso
D. Todas
ANSWER D
Fisiopatológicamente la causa prerrenal de la Insuficiencia renal aguda corresponde a una perfusión
sanguínea inadecuada?

• Verdadero

• Falso
ANSWER verdadero
Fisiopatológicamente la causa postrenal de la insuficiencia renal aguda corresponde a una alteración
en la integridad del parénquima renal
• Verdadero

• Falso
ANSWER falso
Indique la respuesta correcta, los siguientes son procesos que afectan directamente el parénquima
renal en la fisiopatología de la insuficiencia renal aguda por causa renal, excepto
A. Lesión aguda de tubulos renales
B. Enfermedades tubulointersticiales
C. Enfermedades de grandes vasos del riñon
D. Disminución del gasto cardiaco
ANSWER D
Banco de preguntas grupo 11

¿Cuál de los siguientes es una contraindicación para el trasplante renal?:


A) Síndrome de Alport
B) Amiloidosis
C) Cistinosis
D) Todas las anteriores
E) Ninguna de las anteriores
ANSWER E
¿Cuál de los siguientes hallazgos NO es indicación de diálisis en pacientes con insuficiencia renal
aguda?:
A) Sobrecarga de volumen.
B) Hipercalemia severa.
C) Acidosis metabólica.
D) Hipocalcemia severa.
E) Pericarditis urémica.
ANSWER D
En el estudio inicial de un paciente con insuficiencia renal ¿qué prueba complementaria resultaría más
útil para diferenciar si es aguda o crónica?:
A) Hemograma.
B) Electromiograma.
C) Radiología ósea.
D) Sedimiento.
E) Ecografía renal.
ANSWER E
¿De cuál de los siguientes cuadros es característica la aparición de cilindros hemáticos en el sedimento
urinario?:
A) Necrosis tubular aguda.
B) Lesión a cualquier nivel de las vías urinarias.
C) Daño glomerular severo.
D) Daño tubular.
E) Cualquier lesión de la nefrona.
ANSWER C
Banco de preguntas grupo 11

La tasa de filtración glomerular aumenta cuando:


A) Aumenta la resistencia en la arteriola aferente glomerular.
B) Disminuye la resistencia en la arteriola eferente glomerular.
C) Aumenta la actividad de los nervios simpáticos renales.
D) Se produce obstrucción de la vía urinaria.
E) Disminuye la concentración de las proteínas plasmáticas.
ANSWER E
¿Qué es litiasis renal?
A. Cálculos en el aparato urinario
B. Cálculos en el hígado
C. Lesión de la glándula suprarrenal
D. Ninguna de las anteriores
ANSWER: A
¿A qué edad en hombres es más frecuente la litiasis renal?
A. 20-30 años
B. 30-40 años
C. 40-60 años
D. Ninguna de las anteriores
ANSWER: C
¿Cuál es el cálculo más frecuente en pacientes diabéticos?
A. Ac. Úrico
B. Calcio
C. Magnesio
D. Ninguna de las anteriores
ANSWER: A
Complete: El dolor provocado por un cálculo renal se acompaña de .............................
A. Fiebre, disfagia
B. Nauseas, fiebre
C. Náuseas, vómitos y fiebre
D. Disfagia, vómitos y fiebre
ANSWER: C
Banco de preguntas grupo 11

Complete: El dolor que provoca un cálculo localizado en el uréter inferior se irradia a:


A. La región lumbar
B. Al labio mayor
C. Al testículo ipsilateral
D. B Y C son correctas
ANSWER: D
Complete: Los cálculos renales de ................ se da por una infección por baterías ureasas positivas
(Proteus mirabilis):
A. Ácido úrico
B. Estruvita
C. Calcio
D. Cistina
ANSWER: B
¿Qué cálculos renales se observan radiolúcidos en la radiografía abdominal?
A. Ácido úrico
B. Estruvita
C. Calcio
D. Cistina
ANSWER: A
Señale, entre las siguientes, la afirmación correcta respecto a la sensibilidad de la ecografía en el
diagnóstico de la litiasis renal:
A. Es de alrededor del 0.95 para los cálculos vesicales.
B. Para los cálculos ureterales es mayor cuando se localizan en la porción sacroilíaca del uréter.
C. No varía con el tamaño de los cálculos.
D. Varía según la composición de los cálculos.
ANSWER: A
¿cuál es el tratamiento de elección de la hipercalciuria idiopática?:
A. Furosemida.
B. Restricción de calcio en lo dieta.
C. Hidroclorotiazida.
D. Calcitonina.
ANSWER: C
Banco de preguntas grupo 11

¿Cuál de las siguientes circunstancias es una contraindicación absoluta para la litotricia extracorpórea
con ondas de choque de la litiasis renal?:
A. Embarazo.
B. Coagulopatía.
C. Marcapasos cardiaco.
D. Aneurisma aórtico abdominal.
ANSWER: A
1. Sospecharemos insuficiencia renal aguda de origen pre-renal en presencia de:

A. Hipotensión, elevación de las cifras de urea y creatinina sérica, osmolaridad en orina inferior
a 500 mOsm/kg y excreción fraccional de sodio superior a 1%.

B. Intolerancia digestiva, elevación de las cifras de urea y creatinina séricas, osmolaridad en


orina superior a 500 mOsm/kg y excreción fraccional de sodio inferior a 1%.

C. Administración intravenosa previa de contraste iodado, elevación de las cifras de urea y


creatinina séricas, osmolaridad en orina inferior a 500 mOsm/kg y excreción fraccional de sodio
superior a 1%.

D. Administración previa de un antibiótico aminoglicósido, elevación de las cifras de urea y


creatinina séricas e isostenuria.

2. Un paciente de 65 años acude al servicio de urgencias por un cuadro de hipotensión


ortostática, taquicardia y anuria de 24 horas de evolución. La exploración física y estudios
complementarios revelan una reducción de la presión venosa, disminución de la turgencia
cutánea, creatinina en plasma de 400 micromol/l (4,47 mg/dl) y sodio urinario inferior a 10
mmol/l. El paciente refiere además un cuadro de gastroenteritis de 5 días de evolución y bajo
aporte hídrico. Conteste la respuesta correcta:

A. El fracaso renal agudo puede desaparecer rápidamente tras restablecer la perfusión renal.

B. El tratamiento con antiinflamatorios no esteroides puede ser beneficioso para la resolución


del cuadro.

C. Es preciso realizar siempre una biopsia renal en estos casos para la obtención del
diagnóstico. D. Es necesario la monitorización hemodinámica invasiva en la UVI.

3. Hombre de 35 años. Antecedentes de epilepsia y adicción a drogas no parenterales. Ingresa


en Urgencias tras ser encontrado comatoso en la calle. Al ingreso estuporoso, sin localidad
neurológica. Exploración cardiovascular normal. Tensión arterial 135/78 mmHg. Dolor difuso a
la compresión en miembros superior e inferior derechos, con pantorrilla derecha caliente y
edematosa. Tras sondaje vesical se recuperan 200 ml de orina oscura. Analítica: hemoglobina
14,4 g/dl, 7.800 leucocitos/ mm3, glucemia 68 mg/dl, urea 114 mg/dl, creatinina 4,4 mg/dl,
úrico 9,9 mg/dl, calcio 7,0 mg/dl. Analítica de orina: densidad 1012, pH 5,5, proteinuria +,
sedimento normal, sodio urinario 64 mmol/1. Ecografia renal normal. ¿A cuál de las siguientes
pruebas o determinaciones analíticas le ve más utilidad inmediata para identificar la causa de
la insuficiencia renal de este paciente?

A. Niveles de anticomiciales en sangre.

B. Nivel de creatinkinasa (CPK) en sangre.

C. Patrones de citólisis y colostasis hepática.

D. Hemocultivos seriados y urocultivo.


4. Una mujer de 58 años acude para una visita de seguimiento por diabetes mellitus e
hipertensión. Se siente bien, pero afirma que ha dejado de tomar verapamilo por
estreñimiento. Presenta intolerancia a los IECA por tos. En la exploración, la presión arterial es
de 156/92 mmHg. En la analítica incluyen una creatinina de 1,6 mg/dl, excreción de proteínas
en orina de 24 horas de 1,5 g/día y un aclaramiento de la creatinina de 45 ml/min. Sobre esta
base, ¿cuál es el tratamiento más eficaz para enlentecer la progresión de la nefropatía
diabética tipo 2 de la paciente?

A. Inhibidor de la enzima convertidora de la angiotensina.

B. Bloqueador del receptor de la angiotensina.

C. Antagonista del calcio.

D. Alfabloqueante.

5. ¿Cuál de las siguientes glomerulopatías es la causa más frecuente de síndrome nefrótico en


el anciano?

A. Glomerulonefritis membranosa.

B. Glomeruloesclerosis segmentaria focal.

C. Nefropatía diabética.

D. Amiloidosis renal.

6. Con respecto al síndrome nefrótico idiopático, señale la respuesta correcta:

A. El tipo histológico más habitual es la hiperplasia mesangial difusa.

B. El síndrome nefrótico idiopático por lesiones mínimas (síndrome nefrótico de cambios


mínimos) es más frecuente en los niños mayores de 8 años.

C. La glomeruloesclerosis segmentaria y focal es la forma histológica que mejor responde al


tratamiento con corticoides.

D. La dislipemia y la hipercoagulabilidad sanguínea son manifestaciones clínicas habituales


en los pacientes con síndrome nefrótico.

7. La tasa de filtración glomerular aumenta cuando:

A. Aumenta la resistencia en la arteriola aferente glomerular.

B. Disminuye la resistencia en la arteriola eferente glomerular.

C. Aumenta la actividad de los nervios simpáticos renales.

D. Disminuye la concentración de las proteínas plasmáticas.


8. Excepto en los pacientes cuyo fracaso renal crónico sea debido a nefropatía diabética o a
nefropatía tubulointersticial, el patrón de gasometría arterial que usted esperaría encontrar en
un paciente con fracaso renal crónico sería:

A. pH 7,30, HCO3 18 mEq/l, Cl 116 mg/dl, porque es característica la acidosis metabólica con
anión gap normal.

B. pH 7,46, HCO3 18 mEq/l, Cl 116 mg/dl, porque es característica la alcalosis metabólica


hiperclorémica.

C. pH 7,456, HCO3 18 mEq/l, Cl 100 mg/dl, porque es característica la acidosis metabólica con
anión gap aumentado.

D. pH 7,46, HCO3 30 mEq/l, Cl 90 mg/dl, porque es característica la alcalosis metabólica con


anión gap normal.

9. La acidosis metabólica con anión gap (hiato amónico) aumentado puede ser producida por
todas las siguientes causas menos una. Señalela:

A. Cetoacidosis diabética.

B. Acidosis láctica.

C. Diarrea aguda.

D. Insuficiencia renal aguda.

10. ¿Cuál de las siguientes cifras de albúmina en orina se define como microalbuminuria?

A. Menos de 30 mg en 24 horas.

B. Menos de 300 mg en 24 horas.

C. Entre 30 y 300 mg/g de creatinina.

D. Entre 300 y 1000 mg/g de creatinina.

11. ¿Cuál de los siguientes es un mecanismo de progresión de la enfermedad renal crónica


independientemente de su etiología?

A. La hiperfiltración.

B. La malnutrición.

C. La hiperpotasemia.

D. La acidosis.
12. ¿Cuál de las siguientes situaciones clínicas conlleva un mayor riesgo de progresión de la
enfermedad renal crónica y requeriría un control más estricto por parte del nefrólogo?

A. Paciente diabético con un filtrado glomerular de 46 mL/min y un cociente


albumina/creatinina en orina de 25 mg/g.

B. Paciente diabético con filtrado glomerular de 89 mL/ min y cociente albumina/creatinina


en orina de 475 mg/g.

C. Paciente hipertenso con filtrado glomerular de 65 mL/ min y cociente albumina/creatinina


en orina de 150 mg/g.

D. Paciente hipertenso de 70 años con 1 quiste simple en cada riñón, filtrado glomerular de 35
mL/min y cociente albumina/creatinina en orina de 10 mg/g.

13. Mujer de 72 años con diabetes tipo 2 e insuficiencia renal crónica en estadio 5. Ante la
sospecha de un tromboembolismo pulmonar indique qué prueba diagnóstica estaría
contraindicada:

A. Ecocardiograma transtorácico.

B. Gammagrafía pulmonar.

C. AngioTC pulmonar.

D. Radiografía de tórax.

14. La insuficiencia renal crónica es una patología cada vez mas prevalente en nuestros
pacientes. En las sociedades industrializadas la causa más frecuente es:

1. Diabetes mellitus.

2. Hipertensión arterial.

3. Glomerulonefritis.

4. Riñón quístico.

5. Infecciones urinarias de repetición.

15. Paciente de 7 años de edad que consulta por aumento de tamaño de la bolsa escrotal
derecha, siendo derivado a la consulta de Cirugía para descartar hidrocele o hernia inguinal. Es
visto en la consulta de Cirugía tres semanas más tarde donde aprecian edema escrotal y de
pene, edema palpebral y edema de miembros inferiores siendo remitido a Urgencias. No
refiere oliguria. Exploración: FC: 90 lpm TA: 105/65 mmHg. Ta: 36,5 ºC. Sat O2: 98%. Buen
estado general, color pálido de piel. Edemas palpebrales bilaterales. Edema con fóvea hasta raíz
de miembros inferiores. Edema escrotal y peneano. Auscultación cardiopulmonar normal.
Abdomen distendido, no doloroso, con aparente ascitis sin visceromegalias, ni masas
anormales. ORL: normal. Antecedentes personales sin relevancia. Antecedentes familiares:
Padre con diabetes insulinodependiente. ¿Cuál sería su primera sospecha diagnóstica?

1. Insuficiencia cardiaca.

2. Síndrome nefrótico.

3. Glomerulonefritis proliferativa.

4. Insuficiencia hepática.

5. Insuficiencia renal.
LECCIÓN DE NEFRO

En cuál nefropatía es más frecuente la trombosis de la vena renal como complicación.

a. Rápidamente progresiva
b. Cambios mínimos
c. Membranosa
d. Membrano proliferativo

La presencia de que hallazgo determina un Síndrome nefrótico impuro.

a. Proteinuria
b. Aumento de los tiempos de coagulación
c. Hematuria
d. Trombosis de la arteria renal

¿Cuál de los siguientes hallazgos NO es frecuente en el síndrome nefrótico?.

a. Edema
b. Hipoproteinemia
c. Hipertensión
d. Hiperlipemia

Un paciente de 38 años acude a urgencias por dolor lumbar bilateral, para el cual ha estado
consumiendo ibuprofeno cada 8 horas las últimas 48 horas, sin clara mejoría de la sintomatología. En la
analítica de sangre realizada a su llegada a urgencias destaca eosinofília y creatinina de 4 mg/dl con urea
118 mg/dl. La diuresis se encuentra conservada y en orina presenta un Na 170 mmol/l y leucocituria.
¿Cuál cree que es la etiología más probable del cuadro que presenta el paciente?

a. Nefritis interstiticial aguda inmunoalérgica por AINE.


b. Fracaso renal agudo prerrenal por consumo de AINE.
c. Ateroembolia renal.
d. Uropatía obstructiva.
¿Cuál es el principal mecanismo que origina un Na en orina menor 20 mEq/L y una fracción de excreción
menor del 1% en la lesión renal aguda prerrenal?.

a. Aumento de la secreción de ADH


b. Aumento de la síntesis de angiotensina II
c. Aumento de la secreción de aldosterona
d. Mayor producción de péptidos natriuréticos

Varón de 75 años, con antecedentes de DM, HTA y cardiopatía isquémica revascularizada, con FEVI 30%.
Acude a Urgencias por aumento de disnea habitual en los últimos días, ortopnea de 3 almohadas y
disnea paroxísti-ca nocturna, acompañado de edemas progresivos en la última semana en miembros
inferiores, con fóvea, hasta rodilla. A la exploración física llaman la atención crepitantes bibasales. Hace
15 días tenía analítica en su médico de Atención Primaria sin alteraciones. Se realiza analítica de sangre,
que muestra: Hb 14 g/dl, leucocitos 5.000 (fórmula leucocitaria normal), Cr 4,2 mg/dl, urea 184 mg/dl,
Na 135 mEq/l, K 4,9 mEq/l. En analítica de orina llama la atención: Na 14 mEq/l, EFNa 0,7%, cilindros
hialinos. Sin hematuria ni proteinuria. Señale cuál de las siguientes es verdadera.

a. El tratamiento consiste en la administración de suero salino.


b. En este caso la presencia de cilindros hialinos orienta a un probable sín-drome nefrítico.
c. La presencia de sodio en orina tan elevado orienta a fracaso renal de tipo parenquimatos
d. Se trata de un fracaso renal agudo prerrenal en contexto de descompen-sación de insuficiencia
cardíaca congestiva.

¿Cuál de los siguientes no es hallazgo del síndrome nefrítico?.

a. Edema
b. Proteinuria mayor 3,5 g/día
c. Hematuria
d. Lesión renal aguda

¿Cuál es el principal mecanismo fisiopatológico relacionado con el edema en el síndrome nefrítico?.

a. Obstrucción linfática renal


b. Disminución de la presión oncotica vascular
c. Insuficiencia cardiaca congestiva por síndrome renocardiaco
d. Aumento de la presión hidrostática vascular
Cuál de los siguientes no es hallazgo del síndrome nefrítico?.

a. Edema
b. Proteinuria mayor 3,5 g/día
c. Hematuria
d. Lesión renal aguda

Varón de 31 años si antecedentes médicos que acude a Urgencias por oliguria. Presenta PA 160/100
mmHg y edemas en MMII. En la analítica de sangre se observa Cr 3 mg/dl y U 120 mg/dl. En la analítica
de orina existe hematuria de 50 hematíes/campo y proteinuria con índice prot/Cr de 1 g/g. ¿Qué
síndrome clínico presenta este paciente?.

a. Síndrome nefrítico.
b. Hematuria aislada.
c. Enfermedad renal crónica.
d. Síndrome nefrótico.

La presencia de que hallazgo determina un Síndrome nefrótico impuro.

a. Proteinuria
b. Deterioro del filtrado glomerular
c. Proteinuria por encima de 3,5 g/día
d. Deterioro del volumen circulante efectivo

La proteinuria en rango nefrótico corresponde a.

a. Hipoproteinemia
b. Proteinuria menor 3,5 g/día
c. Proteinuria igual 3,5 g/día
d. Proteinuria mayor 3,5 g/día

Cuál es el principal mecanismo fisiopatológico relacionado con el edema en el síndrome nefrótico.

a. Disminución de la presión oncotica vascular


b. Aumento de la presión hidrostática vascular
c. Obstrucción linfática renal
d. Insuficiencia cardiaca congestiva por síndrome renocardiaco
¿Cuál es el mayor factor que evalúa y controla la progresión de la enfermedad renal crónica?.

a. Hematuria
b. Proteinuria
c. Creatinina
d. Gammagrafía renal

Varón de 60 años que presenta síndrome nefrótico. ¿Cuál de los siguientes no haría sospechar nefropatía
diabética?.

a. Hipocomplementemia.
b. Hipoaldosteronismo hiporreninémico.
c. Insuficiencia renal.
d. Hematuria microscópica.

1. Sospecharemos insuficiencia renal aguda de origen pre-renal en presencia de:

A. Hipotensión, elevación de las cifras de urea y creatinina sérica, osmolaridad en orina inferior a
500 mOsm/kg y excreción fraccional de sodio superior a 1%.
B. Intolerancia digestiva, elevación de las cifras de urea y creatinina séricas, osmolaridad en orina
superior a 500 mOsm/kg y excreción fraccional de sodio inferior a 1%.
C. Administración intravenosa previa de contraste iodado, elevación de las cifras de urea y
creatinina séricas, osmolaridad en orina inferior a 500 mOsm/kg y excreción fraccional de sodio
superior a 1%.
D. Administración previa de un antibiótico aminoglicósido, elevación de las cifras de urea y
creatinina séricas e isostenuria.

2. Un paciente de 65 años acude al servicio de urgencias por un cuadro de hipotensión ortostática,


taquicardia y anuria de 24 horas de evolución. La exploración física y estudios complementarios revelan
una reducción de la presión venosa, disminución de la turgencia cutánea, creatinina en plasma de 400
micromol/l (4,47 mg/dl) y sodio urinario inferior a 10 mmol/l. El paciente refiere además un cuadro de
gastroenteritis de 5 días de evolución y bajo aporte hídrico. Conteste la respuesta correcta:

A. El fracaso renal agudo puede desaparecer rápidamente tras restablecer la perfusión


renal.
B. El tratamiento con antiinflamatorios no esteroides puede ser beneficioso para la
resolución del cuadro.
C. Es preciso realizar siempre una biopsia renal en estos casos para la obtención del
diagnóstico. D. Es necesario la monitorización hemodinámica invasiva en la UVI.

3. Hombre de 35 años. Antecedentes de epilepsia y adicción a drogas no parenterales. Ingresa en


Urgencias tras ser encontrado comatoso en la calle. Al ingreso estuporoso, sin localidad neurológica.
Exploración cardiovascular normal. Tensión arterial 135/78 mmHg. Dolor difuso a la compresión en
miembros superior e inferior derechos, con pantorrilla derecha caliente y edematosa. Tras sondaje
vesical se recuperan 200 ml de orina oscura. Analítica: hemoglobina 14,4 g/dl, 7.800 leucocitos/ mm3,
glucemia 68 mg/dl, urea 114 mg/dl, creatinina 4,4 mg/dl, úrico 9,9 mg/dl, calcio 7,0 mg/dl. Analítica de
orina: densidad 1012, pH 5,5, proteinuria +, sedimento normal, sodio urinario 64 mmol/1. Ecografia
renal normal. ¿A cuál de las siguientes pruebas o determinaciones analíticas le ve más utilidad inmediata
para identificar la causa de la insuficiencia renal de este paciente?

A. Niveles de anticomiciales en sangre.


B. Nivel de creatinkinasa (CPK) en sangre.
C. Patrones de citólisis y colostasis hepática.
D. Hemocultivos seriados y urocultivo.

4. Una mujer de 58 años acude para una visita de seguimiento por diabetes mellitus e hipertensión. Se
siente bien, pero afirma que ha dejado de tomar verapamilo por estreñimiento. Presenta intolerancia a
los IECA por tos. En la exploración, la presión arterial es de 156/92 mmHg. En la analítica incluyen una
creatinina de 1,6 mg/dl, excreción de proteínas en orina de 24 horas de 1,5 g/día y un aclaramiento de la
creatinina de 45 ml/min. Sobre esta base, ¿cuál es el tratamiento más eficaz para enlentecer la
progresión de la nefropatía diabética tipo 2 de la paciente?

A. Inhibidor de la enzima convertidora de la angiotensina.


B. Bloqueador del receptor de la angiotensina.
C. Antagonista del calcio.
D. Alfabloqueante.

5. ¿Cuál de las siguientes glomerulopatías es la causa más frecuente de síndrome nefrótico en el


anciano?

A. Glomerulonefritis membranosa.
B. Glomeruloesclerosis segmentaria focal.
C. Nefropatía diabética.
D. Amiloidosis renal.

6. Con respecto al síndrome nefrótico idiopático, señale la respuesta correcta:

A. El tipo histológico más habitual es la hiperplasia mesangial difusa.


B. El síndrome nefrótico idiopático por lesiones mínimas (síndrome nefrótico de cambios mínimos)
es más frecuente en los niños mayores de 8 años.
C. La glomeruloesclerosis segmentaria y focal es la forma histológica que mejor responde al
tratamiento con corticoides.
D. La dislipemia y la hipercoagulabilidad sanguínea son manifestaciones clínicas habituales en los
pacientes con síndrome nefrótico.

7. La tasa de filtración glomerular aumenta cuando:

A. Aumenta la resistencia en la arteriola aferente glomerular.


B. Disminuye la resistencia en la arteriola eferente glomerular.
C. Aumenta la actividad de los nervios simpáticos renales.
D. Disminuye la concentración de las proteínas plasmáticas.
8. Excepto en los pacientes cuyo fracaso renal crónico sea debido a nefropatía diabética o a nefropatía
tubulointersticial, el patrón de gasometría arterial que usted esperaría encontrar en un paciente con
fracaso renal crónico sería:

A. pH 7,30, HCO3 18 mEq/l, Cl 116 mg/dl, porque es característica la acidosis metabólica


con anión gap normal.
B. pH 7,46, HCO3 18 mEq/l, Cl 116 mg/dl, porque es característica la alcalosis metabólica
hiperclorémica.
C. pH 7,456, HCO3 18 mEq/l, Cl 100 mg/dl, porque es característica la acidosis metabólica
con anión gap aumentado.
D. pH 7,46, HCO3 30 mEq/l, Cl 90 mg/dl, porque es característica la alcalosis metabólica
con anión gap normal.

9. La acidosis metabólica con anión gap (hiato amónico) aumentado puede ser producida por todas las
siguientes causas menos una. Señalela:

A. Cetoacidosis diabética.
B. Acidosis láctica.
C. Diarrea aguda.
D. Insuficiencia renal aguda.

10. ¿Cuál de las siguientes cifras de albúmina en orina se define como microalbuminuria?

A. Menos de 30 mg en 24 horas.
B. Menos de 300 mg en 24 horas.
C. Entre 30 y 300 mg/g de creatinina.
D. Entre 300 y 1000 mg/g de creatinina.

11. ¿Cuál de los siguientes es un mecanismo de progresión de la enfermedad renal crónica


independientemente de su etiología?

A. La hiperfiltración.
B. La malnutrición.
C. La hiperpotasemia.
D. La acidosis.
12. ¿Cuál de las siguientes situaciones clínicas conlleva un mayor riesgo de progresión de la enfermedad
renal crónica y requeriría un control más estricto por parte del nefrólogo?

A. Paciente diabético con un filtrado glomerular de 46 mL/min y un cociente


albumina/creatinina en orina de 25 mg/g.
B. Paciente diabético con filtrado glomerular de 89 mL/ min y cociente
albumina/creatinina en orina de 475 mg/g.
C. Paciente hipertenso con filtrado glomerular de 65 mL/ min y cociente
albumina/creatinina en orina de 150 mg/g.
D. Paciente hipertenso de 70 años con 1 quiste simple en cada riñón, filtrado glomerular de
35 mL/min y cociente albumina/creatinina en orina de 10 mg/g.

13. Mujer de 72 años con diabetes tipo 2 e insuficiencia renal crónica en estadio 5. Ante la sospecha de
un tromboembolismo pulmonar indique qué prueba diagnóstica estaría contraindicada:

A. Ecocardiograma transtorácico.
B. Gammagrafía pulmonar.
C. AngioTC pulmonar.
D. Radiografía de tórax.

14. La insuficiencia renal crónica es una patología cada vez mas prevalente en nuestros pacientes. En las
sociedades industrializadas la causa más frecuente es:

1. Diabetes mellitus.
2. Hipertensión arterial.
3. Glomerulonefritis.
4. Riñón quístico.
5. Infecciones urinarias de repetición.
15. Paciente de 7 años de edad que consulta por aumento de tamaño de la bolsa escrotal derecha,
siendo derivado a la consulta de Cirugía para descartar hidrocele o hernia inguinal. Es visto en la consulta
de Cirugía tres semanas más tarde donde aprecian edema escrotal y de pene, edema palpebral y edema
de miembros inferiores siendo remitido a Urgencias. No refiere oliguria. Exploración: FC: 90 lpm TA:
105/65 mmHg. Ta: 36,5 ºC. Sat O2: 98%. Buen estado general, color pálido de piel. Edemas palpebrales
bilaterales. Edema con fóvea hasta raíz de miembros inferiores. Edema escrotal y peneano. Auscultación
cardiopulmonar normal. Abdomen distendido, no doloroso, con aparente ascitis sin visceromegalias, ni
masas anormales. ORL: normal. Antecedentes personales sin relevancia. Antecedentes familiares: Padre
con diabetes insulinodependiente. ¿Cuál sería su primera sospecha diagnóstica?

1. Insuficiencia cardiaca.
2. Síndrome nefrótico.
3. Glomerulonefritis proliferativa.
4. Insuficiencia hepática.
5. Insuficiencia renal.
1. ¿Cuál de los siguientes tipos de anemia se debe a una enzimopatia?
a. Esferocitosis hereditaria
b. Talasemia
c. Eliptocitosis congénita
d. Déficit de glucosa 6 fosfato deshidrogenasa
2. ¿Qué se foema cuando ocurre la precipitación de las moléculas de hemoglobina en el
eritrocito?
a. Cuerpos de Heinz
b. Cuerpos de Dohie
c. Cuerpos de Howell Jolly
d. Cuerpos de Auer
3. La mutacion del gen PK-LR. ¿En que enzimopatia se presenta?
a. Deficit de glucosa 6 fosfato deshidrogenasa
b. Déficit de pirimidina 5 nucleotidasa
c. Déficit de piruvatocinasa
d. A y B son correctas
4. ¿Cuándo encontramos un punteado basófilo grosero en los hematíes en un frotis de
sangre periférica, debemos sospechar en que tipo de anemia?
a. Anemia por hemoglobinopatía
b. Anemia por membranopatia
c. Anemia por déficit de pirimidina 5 nucleotidasa
d. Anemia falciforme
5. Elija la opción correcta: las causas de anemia por enfermedades crónicas son:
a. Inflamaciones crónicas, enfermedades autoinmunes, neoplasias
b. deficiencia de hierro
c. nutrición deficiente
d. todas
6. la anemia por enfermedades crónicas se presenta con:
a. Hiposideremia, ferritina alta
b. Hipersideremia
c. Ferritina baja
d. Hipersideremia y ferritina baja
7. Factores que intervienen en la anemia por enfermedades crónicas:
a. Eritrpoyesis deficiente
b. Activación de macrófagos y linfocitos T
c. Aumento de expresión de citosinas
d. Todas
8. ¿Por qué se produce alteración de absorción de hierro intestinal?
a. Disminuye la liberación de hierro por la hepcidina
b. Aumento de hierro plasmático
c. Bloqueo de hepcidina
d. Todas
9. ¿Qué tan frecuente es la anemia por enfermedades crónicas?
a. Es la mas rara
b. Segunda causa de anemia después de la ferropénica
c. Es la de mayor frecuencia
d. A y B son correctas
10. Los trastornos confenitos de las hemoglobinas pueden clasificarse en:
a. Talasemias y hemoglobinopatías estructurales
b. Falciformes y compuestas
c. Adquiridas y estructurales
d. Hemoglobinas alfa y beta
11. En que trastorno existe una disminución de la síntesis de las cadenas globina que
forman la hemoglobina:
a. Hemoglobinopatías adquiridas
b. Hemoglobinopatía S
c. En el rasgo drepanocitico
d. Talasemias
12. ¿Cómo surgen las hemoglobinopatías adquiridas?
a. Es una alteración de la hemoglobina que surge como consecuencia de otros
procesos como exposición a toxicos
b. Es una alteración de la hemoglobina que surge del adc
c. Es una alteración de la hemoglobina que surge de un desequilibrio de las
cadenas deficitarias
d. Ninguna
13. ¿En que trastorno congénito se produce la síntesis en cantidades normales de una
cadena de globina anormal?
a. Talasemias
b. Hemoglobinopatía adquirida
c. Hemoglobinopatía estructural
d. Hemoglobinopatía S
14. ¿En cual de las hemoglobinopatías se encuentran la hemoglobina E y la hemoglobina
de Lepore?
a. Hemoglobinopatía S
b. Hemoglobinopatía adquirida
c. En los hematíes
d. Hemoglobinopatía infecciosa
15. ¿Por qué se caracterizan las anemias megaloblasticas?
a. Síntesis defectuosa del ARN
b. Síntesis defectuosa de ARN y ADN
c. Síntesis defectuosa de ADN
d. Ninguna
16. Señale cual de estas opciones no es una función de la vitamina B12
a. Regulación del ADN
b. Regulación de la formación de eritrocitos
c. Controla nivel de homocisteina
d. Maduración de purinas y aminoácidos
17. ¿Cuál es el requerimiento diario de acido fólico?
a. 50 - 110 ug
b. 100 – 200 ug
c. 50 – 100 ug
d. 300 – 500 ug
18. ¿Cuál es el mecanismo habitual por el cual se da la absorción de cobalamina?
a. Mecanismo pasivo activo
b. Mecanismo activo
c. Mecanismo pasivo
d. Ninguna
19. Señale una causa de anemia aplásica adquiridas
a. Congénitas
b. Quimioterapias
c. Alcohol
d. Ninguna
20. ¿Cuál es el tiempo de presentación de la anemia aplásica?
a. Abrupto
b. Lento
c. Progresivo
d. Ninguno
21. Subraye una manifestación clínica de anemia aplásica
a. Fiebre
b. Leucocitosis
c. Hematomas
d. A y B son correctas
22. La anemia aplásica se diagnostica mediante muestra de sangre y análisis de medula
osea (Verdadera)
23. ¿Cuál es la clínica de la anemia de las enfermedades crónicas además de los síntomas
de la enfermedad base?
a. Síndrome constitucional, fiebre, síndrome anémico y mayor afectación
subjetiva del estado general de lo normal
b. Síndrome anémico, disnea, sin aumentar la afeccion del estado general del
paciente
c. Fiebre, dolor torácico, artralgias, hemoptisis, con mayor afeccion del estado
general del paciente
d. No presentan síntomas
24. En cuanto al diagnostico y desde el punto de vista morfológico como se presenta la
anemia de enfermedades crónicas
a. Anemia normocítica normocrómica cuando es moderada
b. Anemia microcitica hipocromica cuando es mas grave
c. Anemia macrocitica hipocromica cuando es moderada
d. A y B son correctas
25. En la anemia de las enfermedades crónicas. ¿Que revela el laboratorio?
a. Índice de reticulocitos bajo en relación con el grado de anemia, ferritina sérica
disminuido. Hierro sérico aumentado.
b. El indicie de reticulocitos bajo en relación con el grado de anemia, ferritina
sérica aumentada (mas de 100 ng/ml). Hierro sérico: bajo.
c. Índice de reticulocitos alto en relación con el grado de anemia, ferritina sérica
aumentada (mas de 50 ng/ml). Hierro sérico: aumentado.
d. Índice de reticulocitos normal. Hierro sérico normal.
26. Para el diagnostico diferencial de anemia por trastornos crónicos que nos indican
valores de la ferritina
a. Ferritina por encima de 100 ng/ml corresponde a AEC
b. Ferritina entre 30 y 100 ng/ml sugiere una anemia mixta (AEC con ferropenia
asociada.
c. El cociente RST/logaritmo de la ferritina con un índice inferior a 1 es indicativo
de AEC, y valores superiores a 2 son indicativos de ferropenia
d. Todas son correctas
27. Según el tratamiento señale la opción correcta
a. La transfusión de concentrado de hematíes no esta indicada, salvo en anemia
severa muy sintomática
b. El hierro esta contraindicado
c. El hise de rHuEPO se indica para evitar transfusiones y sobrecarga
correspondiente de hierro
d. Todas
28. Tipos de mecanismos por las que se produce la anemia hemolítica inducida por
fármacos
a. Mecanismo autoinmune, mecanismo hapteno y mecanismo neoantigeno
b. Mecanismo hapteno, mecanismo fagocitico
c. Mecanismo neoantigeno, mecanismo oxidativo
d. Todas
29. ¿En qué tipo de mecanismo el medicamento se fija a la membrana eritrocitaria y se
une al anticuerpo sin que este contacte directamente con ninguna estructura
eritrocitaria?
a. Mecanismo neoantigeno
b. Mecanismo hapteno
c. Mecanismo autoinmune
d. A y B son correctas
30. Las causas de anemias hemolíticas extracorpusculares no inmunes son el
hiperesplenismo, desordenes metabólicos y agentes infecciosos (verdadero)
31. Los siguientes hallazgos están presentes tanto en la deficiencia de acido fólico como en
la deficiencia de vitamina B12. Excepto uno:
a. Hematíes macrociticos
b. Neuropatía periférica
c. Maduración megaloblastica en la medula osea
d. Niveles elevados de lactato deshidrogenasa (LDH) y bilirrubina indirecta
e. Hipersegmentacion de neutrófilos
32. ¿Qué tipo de linfocitos son mas abundantes en sangre periférica?
a. Linfocitos B CD5+ (Bla)
b. Linfocitos T colaboradores CD4+
c. Linfocitos T citotóxicos CD8+
d. Linfocitos T gamma/delta
33. Un paciente de 29 años acude a su consulta con un diagnostico de aplasia medular
severa. ¿Cuál es el tratamiento de elección?
a. Transfusiones periódicas y antibióticos
b. Andrógenos y transfusiones de plaquetas
c. Trasplante alogénico de medula osea si hermano HLA idéntico
d. Trasplante autologo de medula osea para evitar rechazo
34. ¿Cuáles son los tipos principales de Talasemias?
a. Talasemia alfa
b. Talasemia beta
c. Talasemia hereditaria
d. A y B son correctas
35. ¿Cuáles de las siguientes afirmaciones es verdadera con respecto a las anemias
ferropénicas?
a. Es una anemia macrocitica
b. Las necesidades de hierro son especialmente intensas en el tercer trimestre
del embarazo
c. La causa mas frecuente de ferropenia en un individuo adulto es la perdida de
pequeñas cantidades de sangre de forma crónica por la orina
d. Todas son correctas
36. Una de las siguientes manifestaciones clinicas no corresponde a un cuadro de anemia
ferropénica:
a. Disfagia
b. Hipotensión
c. Cabello frágil
d. Glositis
37. ¿Cuál de las siguientes pruebas es útil en la diferenciación entre una anemia
ferropénica y una anemia inflamatoria?
a. Sideremia
b. Ferritina
c. Mielograma
d. Hemoglobina
38. ¿Cuál de los siguientes efectos secundarios no esta provocado por la ingesta de
preparados férricos?
a. Nauseas
b. Cefalea
c. Epigastralgias
d. Estreñimiento
39. Indique la causa mas frecuente de anemia ferropénica
a. Neoplasia del ciego
b. Hernia de hiato
c. Angiodisplasia intestinal
d. Hipermenorrea o metrorragia
40. La cantidad total de hierro del organismo es de unos:
a. 6 gramos
b. 2 gramos
c. 3,5 gramos
d. 8 gramos
41. El hierro procedente de la dieta se absorbe en:
a. Duodeno y primera procion del yeyuno
b. Unión del yeyuno e ileon
c. Íleon terminal
d. Colon sigmoideo
42. Causas de anemia ferropénica por aumento de las demandas de hierro:
a. Crecimiento rápido en la infancia o adolescencia y embarazo
b. Menstruación
c. Donación de sangre y tratamiento con eritropoyetina
d. Hemorragia aguda
43. Manifestación clínica de la anemia ferropénica, Cual no es:
a. Queilosis
b. Coiloniquia
c. Fatiga
d. Neuropatía periférica bilateral
44. El contenido de hierro alimentario se encuentra muy relacionado con el aporte
calórico total
a. Alrededor de 10 mg de hierro elemental/100 calorias
b. Alrededor de 6 mg de hierro elemental/1000 calorias
c. Alrededor de 1 mg de hierro elemental/10000 calorias
d. Alrededor de 25 mg de hierro elemental/1000 calorias
45. Señale la que NO corresponda con los hallazgos de labotatorio de una anemia
ferropénica
a. Disminución de ferritina
b. Reticulocitosis
c. Microcitosis
d. Aumento de transferrina
46. Señale lo incorrecto
a. El hierro serico representa las reservas de hierro en nuestro organismo
b. La ferritina se eleva en procesos inflamatorios
c. El índice de saturación de transferrina esta disminuido en anemia ferropénica
d. B y C son incorrectas
47. ¿Qué se encuentra en el frotis de sangre periférica en la anemia ferropénica?
a. Hipocromía
b. Microcitosis
c. Anisocitosis
d. Todas
48. ¿Cuánto hierro aporta una unidad de eritrocitos empaquetados?
a. 10 g
b. 200 g
c. 5 mg
d. 1 g
49. ¿Cuándo está indicado el hierro parenteral?
a. Falla de terapia oral
b. Intolerancia de hierro oral
c. Gastrectomía
d. Todas
50. ¿Cuál de las siguientes enzimopatias suele presentar crisis eritroblastopenicas
transitorias en la infancia?
a. Déficit de piruvatocinasa
b. Déficit de pirimidina 5 nucleotidasa
c. Anemia talasemia
d. Esferocitosis hereditaria
51. ¿Cuál de las siguientes drogas es útil en el síndrome mielodisplásico?
a. Azacitidina
b. Imatimib
c. Bortezomib
d. Rituximab
52. ¿Dónde se encuentra la alteración patogénica principal en los síndromes
mielodisplásicos?
a. En la célula germinal pluripotencial o célula de stem
b. En los mecanismos de depósito de hierro en la medula ósea
c. La alteración principal es en los receptores de los factores de crecimiento
celular
d. En el microambiente de la medula osea
53. En relación con los síndromes mielodisplásicos señale la afirmación correcta:
a. Son típicos de pacientes ancianos con predominio en varones
b. Suelen cursar con Pancitopenia y alteraciones morfológicas de sangre
periférica
c. La anemia es casi constante
d. La medula osea es normo o hipocelular
54. Respecto a la anemia refractaria. ¿Cuál de las siguientes respuestas es falsa?
a. Dispone de un tratamiento médico eficaz
b. El soporte transfusional con concentrados de hematíes es el tratamiento más
importante
c. La evolución a leucemia aguda es frecuente
d. Cuando las transfusiones son muy repetidas es aconsejable utilizar filtros
leucocitarios
55. ¿Cuál es el hallazgo característico en medula sea en la trombocitopenia refractaria?
a. Displasia eritroide aislada
b. Displasia granulocitica aislada
c. Displasia megacariocitica aislada
d. Displasia en una sola línea mieloide
56. ¿En cual de las siguientes circunstancias es excepcional la existencia de trombocitosis
reactiva?
a. Hemorragias
b. Neoplasias epiteliales
c. Anemia refractaria con exceso de blastos en transformación
d. Anemia ferropénica
57. ¿Cuál es el hallazgo en sangre de la citopenia refractaria con displasia unilineal?
a. Anemia
b. Neutropenia
c. Mono o Bicitopenia
d. Trombocitopenia
58. ¿Cuál es el hallazgo característico en medula osea en la anemia refractaria?
a. Displasia eritroide aislada
b. Displasia granulocitica aislada
c. Displasia megacariocitica aislada
d. Displasia en una sola línea mieloide
59. La policitemia vera es una patología hereditaria (verdadero)
60. ¿Cuál es el gen que determina el diagnostico definitivo de la policitemia vera?
a. JACK 2
b. Ptpn 35
c. Bcra 1
d. P53
61. La policitemia vera tiene posibilidad de transformarse en:
a. Trombocitopenia leve
b. Mielofibrosis primaria
c. Trombosis esencial
d. Anemia perniciosa
62. ¿Qué medicamentos se deben suspender en la dieta de pacientes con policitemia
vera?
a. Proteinas
b. Vitamina K
c. Granos
d. Ninguno
63. ¿Cuál de los siguientes grupos de síntomas tiene mayor porcentaje en la policitemia
vera?
a. Hemorragias
b. Manifestaciones neurológicas
c. Fenómenos tromboticos
d. Síntomas inespecíficos
64. Tratamiento en pacientes con policitemia vera sin riesgo a trombotico
a. Hidroxiurea
b. Fosforo radioactivo
c. Sangrías terapéuticas
d. Acido acetil salicílico
65. Tratamiento en pacientes con policitemia vera con riesgo trombotico que no sea de
edad avanzada
a. Hidroxiurea + AAS
b. AAS + fosforo radioactivo
c. Fosforo radioactivo + hidroxiurea
d. Sangría terapéutica
66. En la policitemia vera existe:
a. Disminución de globulos rojos
b. Aumento de plaquetas
c. Aumento de globulos rojos
d. Disminución de plaquetas
67. ¿A qué edad es más común el diagnostico de policitemia vera?
a. 20 – 30 añós
b. 10 – 18 años
c. En los primeros meses de vida
d. 60 años
68. ¿En qué fase de la policitemia vera es común encontrar esplenomegalia?
a. Fase Gastada
b. Fase inicial
c. Fase terminal
d. Fase de recidiva
69. ¿Qué valores encontramos elevados en la policitemia vera?
a. Conteo de globulos rojos + concentración de hemoglobina
b. Conteo de plaquetas
c. Leucocitos
d. Conteo de globulos rojos
70. ¿En que condiciones se puede administrar copidrogel en un paciente con policitemia
vera?
a. Riesgo de aplasia medular
b. Riesgo trombotico alto
c. Deficiencia de hierro
d. Riesgo de hemorragia digestiva
71. La policitemia vera esta relacionada con neoplasias (verdadero)
72. La trombocitosis esencial es un tipo de neoplasia (verdadero)
73. ¿Dónde se encuentra la mutacion celular de la trombosis esencial?
a. Bazo
b. Medula osea y sangre
c. Riñones
d. Pulmón
74. ¿Qué gen presenta mutaciones en la mayoría de pacientes con trombosis esencial?
a. JAK 2
b. Ptpn 35
c. Bcra 1
d. Bcra 2
75. ¿Cuál de los siguientes precursores celulares se encuentra afectado en la trombosis
esencial?
a. Megacarioblasto
b. Mieloblasto
c. Linfoblasto
d. Monoblasto
76. El gen Jak 2 se encuentra afectado en todos los pacientes con trombosis esencial
(Falso)
77. ¿Qué grupo de edad suele ser mas afectado por la trombosis esencial?
a. Niños
b. Jóvenes
c. Adultos jóvenes
d. Ancianos
78. La trombosis esencial disminuye la expectativa de vida: (Falso)
79. ¿Cuál de los siguientes no constituye un síntoma de trombosis esencial?
a. Ardor en los pies o manos
b. Sangrado normal
c. Hepatomegalia
d. Esplenomegalia
80. ¿Cuál de los siguientes no constituye una complicación de trombosis esencial?
a. Evento cerebrovascular
b. Infarto agudo de miocardio
c. Hemorragia digestiva alta
d. Retraso en el crecimiento fetal
81. ¿En que otra neoplasia mieloproliferativa puede evolucionar la trombosis esencial?
a. Policitemia vera
b. Melofibrosis primraria
c. Leucemia eosinofílica crónica
d. Todas
82. Los pacientes con trombosis esencial suelen ser asintomáticos (verdadero)
83. ¿a partir de que vslor se sospecha de una trombosis esencial?
a. 350 000 plaquetas/ul
b. 500 000 plaquetas/ul
c. 600 000 plaquetas/ul
d. 250 000 plaquetas/ul
84. ¿Cuál de los siguientes exámenes no es fundamental para diagnosticar la trombosis
esencial pero se puede hacer para confirmarlo?
a. Hemograma completo
b. Puncion esplénica
c. Exámenes de medula osea
d. Eritrosedimentacion
85. ¿Cuál de los siguientes no corresponde al tratamiento de trombosis esencial?
a. Hidroxiurea
b. Anagrelida
c. Interferón alfa
d. Citarabina
86. ¿Qué tipo de células se infiltran en la medula osea en la leucemia mielocitica agua?
a. Células adiposas
b. Células neoplásicas
c. Células epiteliales
d. Depósitos de colágeno
87. Etiología de leucemia mieloide aguda
a. Factores hereditarios
b. Radiación
c. Fármacos
d. Todas
88. Manifestaciones clinicas de LMA
a. Petequias y hemorragias
b. Visceromegalia e hipertrofia gingival
c. Tos, ictericia y fiebre
d. A y B son correctas
89. ¿A que tipo de personas afecta mas la anemia mieloide aguda?
a. Recién nacidos
b. Jóvenes
c. Adultos mayores varones
d. Niños 5 – 10 años
90. ¿Qué se encuentra en los exámenes de laboratorio?
a. Anemia, trombocitopenia, neutropenia, leucocitosis
b. Trombocitosis, leucopenia
c. Macrocitos, eosinofilopenia, linfopenia
d. Todas
91. Diagnóstico de leucemia mieloide aguda
a. Medulograma y biopsia de medula ósea
b. TAC, RM y biopsia
c. Biometría completa, ionograma
d. Hemocultivo, coagulograma y creatinina
92. ¿Cuál es el tratamiento de la leucemia mieloide aguda?
a. Quimioterapia
b. Trasplante de medula osea
c. Administrar fluidos y transfusiones
d. A y B son correctas
93. ¿Cuáles son las características de mielofibrosis primaria?
a. Varices esofágicas, disnea, diaforesis nocturna
b. Fibrosis de medula osea, hematopoyesis extramedular y esplenomegalia
c. Ascitis, dolores articulares, fiebre y leucemia aguda
d. Todas
94. ¿Cuáles son los trastornos no malignos que pueden producir mielofibrosis?
a. Infección por VIH, hiperparatiroidismo, LES
b. Linfoma, mieloma muliple, carcinoma metastasico
c. Policitemia vera
d. Enfermedad de Hodgkin
95. ¿Cuáles son los trastornos que pueden producir mielofibrosis?
a. Carcinoma metastasico, mieloma multiple, leucemia aguda
b. Deficiencia de vit D, tuberculosis
c. Infeccion por VIH, hiperparatiroidismo
d. Osteodistrofia renal
96. ¿Con que se vincula la fibrosis en la mielofibrosis?
a. Se vincula con la alteración de una chaperona
b. Se vincula con el deposito intraceluklar de lípido y obliteración del espacio
medular por remodelación osteoclastica
c. Se vincula con la sobreproducción del factor beta de crecimiento
transformador y de inhibidores histicos de las metaloproteinasas
d. A y B son correctas
97. ¿Con que se relaciona la osteoesclerosis en la mielofibrosis?
a. Se lo relaciona por un exceso de blastocitos por monosomia 7
b. Se le relaciona con la sobrepdocuccion de osteoprotegerina, un inhibidor de
los osteoclastos
c. Se lo relaciona con una deficiencia de granulos y segmentación en los
precursores granulociticos
d. Todas son correctas
98. ¿A que se debe la angiogénesis en la mielofibrosis?
a. Se debe al aumento de la producción del factor vascular de crecimiento
endotelial
b. Se debe a los rasgos dismorficos con una población de eritrocitos grandes
c. Se debe a que los neutrófilos tienen menos granulos de los normal
d. Todas son correctas
99. ¿En que características típicas de la hematopoyesis extramedular se encuentran en el
frotis sanguíneo del paciente con mielofibrosis?
a. Dacriocitos, eritrocitos nucleados, mielocitos
b. Esquistocito, queratocitos
c. Acantocitos, degamacitos
d. Eliptocitos, acantocitos
100. ¿Qué quevelan las radiografías en los casos de mielofibrosis?
a. Osteoesclerosis
b. Condroma periostico
c. Osteoma osteoide
d. B y C son correctas
101. ¿Qué puede desencadenar la hematopoyesis extramedular?
a. Hipogammaglobulinemia, exantema benigno, hemolisis
b. Aumento del volumen globular medio, disminución de reticulocitos
c. Inmunodepresión, hemorragia de tubo digestivo
d. Ascitis, hipertensión portal, pulmonar o intracraneal, obstrucción intestinal o
ureteral
102. ¿Cuáles son las características de la evolución natural de mielofibrosis?
a. Pancitopenia, infecciones recurrentes, inflamación articular
b. Insuficiencia progresiva de la medula osea, organomegalia creciente por
efecto de la hematopoyesis extramedular
c. Pancitopenia, purpuro trombocitopenica
d. Fiebre, linfadenopatia
103. ¿A que se debe la trombocitopenia en la mielofibrosis?
a. Anomalías intrínsecas del eritrocito y deficiencia de glucosa 6 fosfato
b. Por la presencia de anticuerpos contra el antígeno eritrocitico
c. A la deficiencia de la función de la medula osea, secuestro esplénico o
destrucción autoinmunitaria
d. Todas
104. ¿Cuáles son los factores pronósticos de mielofibrosis?
a. Alteraciones citogenéticas complejas
b. Trombocitopenia
c. Anemia dependiente de transfusiones
d. Todas
105. ¿Cuál es la razón por la que a menudo no se puede realizare una biopsia
medular en la mielofibrosis?
a. Por el incremento de la reticulina medular
b. Por el dolor que se produce en el paciente
c. Pro las ausencia de datos
d. Todas
106. ¿A que se debe la dificultad en el diagnostico de mielofibrosis?
a. Se debe a que todos los análisis de sangre tienen resultados alterados
b. Se debe a que la mielofibrosis y esplanomegalia también se presentan en
distintos trastornos malignos y benignos
c. Se debe a que existen multiples alteraciones genéticas
d. Todas
107. ¿Qué puede producir el crecimiento esplénico en la mielofibrosis?
a. Puede inducir el infarto del órgano acompañado de fiebre y dolor torácico y
pleurítico
b. Puede conducir a la producción de termoanticuerpos o crioanticuerpos
c. Puede conducir a una trombosis venosa, neutropenia grave y hemorragia
d. B y C son correctas
108. ¿Cuáles de los siguientes enunciados corresponden a Leucemia linfocítica
crónica?
a. Medula osea produce globulos blancos anormales
b. Resulta en la inhibición de la síntesis de ADN en la producción de globulos
rojos
c. Se inicia en las células mieloides primitivas como globulos rojos o
megacariocitos
d. Todas son correctas
109. ¿Cuál de los siguientes factores de riesgo NO corresponde a LLC?
a. Exposición a ciertos químicos
b. Fumar
c. Antecedentes familiares
d. Raza
110. La fisiopatología de la LLC es la celula madre linfoide se convierte en una celula
linfoblastica y luego en uno de los tres tipos siguientes de linfocitos (verdadero)
111. ¿Cuál de los siguientes enunciados corresponde a LLC?
a. Es frecuente en niños
b. Es mas común en mujeres
c. No influyen los factores hereditarios
d. Tipo mas común de leucemia en adultos
112. La hipogammaglobulinemia es un hallazgo frecuente en los enfermos con:
a. Linforma de Hodgkin
b. Leucemia linfática crónica
c. Linfoma de células grandes
d. Tricoleucemia
113. ¿Cuál de las siguientes complicaciones NO propia de la leucemia linfática
crónica?
a. Infecciones oportunistas por virus herpes
b. Anemia hemolítica autoinmune
c. Hipogammaglobulinemia
d. Infiltración del SNC
114. De los siguientes exámenes cual NO es relevante en el diagnostico de leucemia
linfocítica crónica
a. TAC
b. Biopsia de medula osea
c. Análisis de sangre
d. Inmunoelectroforesis en sangre
115. Los pacientes con leucemia linfocítica crónica por linfocitos B tienen un
aumento en la susceptibilidad a padecer infecciones ¿Cuál es la causa principal?
a. Anemia
b. Descenso de inmunoglobulinas
c. Aumento de linfocitos
d. Esplenomegalia
116. Paciente de 72 años con aceptable estado general, leucocitos 53 000/umol con
linfocitosis del 93% hemoglobina y plaquetas normales. El diagnostico mas probable
es:
a. Leucemia linfocítica crónica
b. Leucemia linfoide aguda
c. Leucemia mieloide aguda
d. Leucemia mieloide crónica
117. ¿Qué complicaciones presentan frecuentemente los pacientes con LLC?
a. Insuficiencia renal crónica
b. Cardiopatías
c. Fenómenos autoinmunes
d. Todos
118. ¿Cuántas etapas tiene el sistema RAI que clasifica la LLC?
a. Etapa Rai 0 – 3
b. Etapa Rai 0 – 4
c. Etapa Rai 1 – 3
d. Etapa Rai 1 – 4
119. La linfocitosis mas trombocitopenia, con o sin anemia, agrandamiento de los
ganglios linfáticos, bazo o higado, a que etapa de RAI pertenece:
a. Etapa 1
b. Etapa 2
c. Etapa 3
d. Etapa 4
120. La linfocitosis sin agrandamiento de los ganglios linfáticos, el bazo o el higado,
con recuentos de globulos rojos y plaquetas casi normales, que etapa de Rai
pertenece:
a. Etapa 0
b. Etapa 1
c. Etapa 4
d. Etapa 3
121. El sistema de clasificación de Binet se clasifica por:
a. El numero de grupos de tejido linfático afectado
b. Si el paciente tiene o no anemia
c. Si el paciente tiene o no trombocitopenia
d. Todas las anteriores
122. La clasificación de Binet tiene 3 etapas (A, B, C), que nos indica la etapa B:
a. Tres o mas áreas de tejido linfático están agrandadas, no se observa anemia
ni trombocitopenia
b. Menos de 3 areas de tejido linfático están agrandada, no se observa anemia ni
trombocitopenia
c. Se observa anemia, trombocitopenia o ambas
d. No se observa anemia
123. La leucemia mieloide crónica se caracteriza por:
a. Sobreproducción de células de la serie mieloide, lo que provoca una
esplenomegalia muy marcada y leucocitosis
b. Es consecuencia de una serie de acontecimientos malignos que afectan a un
precursor hematopoyético precoz
c. Estirpe de células que no se diferencian y siguen proliferando de forma
incontrolada
d. Leucemia de linfocitos B infrecuente e indolente
124. Cuando se aparecen los síntomas de LMC, se deben a:
a. Anemia y esplenomegalia
b. Falta de vitamina B12
c. Falta de vitamina B9
d. Falta de acido fólico
125. ¿Cuál es el signo físico mas frecuente en la LMC?
a. Fatiga
b. Esplenomegalia
c. Palidez
d. Leucocitosis
126. Alteración citogenética característica de LMC:
a. Leucocitosis
b. Presencia de cromosoma Philaderphia
c. Trisomía 21
d. Todas
127. La leucocitosis presente en LMC:
a. Se correlaciona con la carga tumoral , definida por el tamaño del bazo
b. Debida a infectiones
c. Por la anemia
d. Por la esplenomegalia
128. En la LMC, se le debe hacer diagnostico diferencial con:
a. Reacción mieloide
b. Metaplasia mieloide agnogenica
c. Policitemia vera
d. Todas
129. Las fases de la LMC son la fase acelerada y la fase blastica (Verdadero)
130. La fase blastica se diagnostica cuando se detectan un 30% o mas de células
blasticas en la medula osea y/o sangre periferica o cuando se identifican lesiones
blasticas extramedulares (Verdadero)
131. El tratamiento en la fase de aceleración se emplea el trasplante alogénico de
células madre + inhibidores de tirosina cinasa (verdadero)
132. La anemia megaloblasica no es una anemia macrocitica que resulta de la
inhibición de la síntesis de ADN en la producción de globulos rojos (falso)
133. De que vitaminas carece la anemia megaloblastica
a. Vitamina B12, Vitamina B9
b. Vitamina A, vitamina C
c. Vitamina B, vitamina B9
d. Vitamina B12, vitamina A
134. Las causas mas frecuentes de anemias megaloblasticas son
a. Mala absorción
b. Mal nutrición
c. Hiperconsumo
d. Todas
135. La etiología característica de la anemia megaloblastica:
a. Déficit de cobalamina
b. Deficiencia de folatos
c. Trastornos congénitos y adquiridos
d. Todos
136. ¿Cuáles son las causas de deficiencia de folatos?
a. Aporte insuficiente de carnes
b. Mala absorción gástrica
c. Aines
d. A y B correctas
137. Las manifestaciones de anemia megaloblastica son:
a. Alteración de la pigmentación cutante
b. Cefalea
c. Parestesis en extremidades inferiores
d. A y C son correctas
138. Para el diagnostico de la anemia se usa:
a. Hemograma
b. Examen de frotis sangrineo periférico
c. Índice de reticulocitos
d. Todos
139. El test de Schilling consiste en la administración oral de una pequeña dosis de
vitamina B12 marcada con un radioisótopo, luego de 1 h se inyecta en el musculo Vit
B12 no marcada y se recolecta la orina después de 24 h (verdero)
140. Causas de anemia aplásica:
a. Metrotexate, aines
b. Gasolina, insecticidas
c. VUH, VEB, CMV
d. Todos
141. ¿Cuáles son las manifestaciones clinicas de Anemia Aplasica?
a. Síntomas anémicos
b. Hemorragia, petequias, purpuras
c. Infecciones recurrentes
d. Todas son correctas
142. Para el diagnostico de anemia aplásica:
a. Análisis de sangre periférica
b. Biopsia de medula osea
c. Aspirado de medula osea
d. Todas
143. En la biopsua de medula osea de anemia aplásica se observa menos del 25% de
celularidad hematopoyética (verdadero)
144. Los criterios de anemia aplásica grave son:
a. Celularidad osea menos 25%
b. Recuento de neutrófilos menor a 1 500/ml
c. Plaquetas mayor de 50 000/ml
d. A y B son correctas
145. El tratamiento ideal para anemia aplásica es el trasplante de medula osea de
un pariente no HLA compatible (falso)
146. Las esferocitosis hereditaria se caracteriza por:
a. Deficiencia de proteínas de membrana de los hematíes
b. Anemia leve – grave, ictericia, litiasis biliar, esplenomegalia
c. Cefalea y tos
d. A y B correctas
147. La eliptocitosis hereditaria se caracteriza por:
a. Deficiencia de proteínas de la membrana de los hematies
b. Alteración en la permeabilidad ionica de la membrana
c. Estomatocitos
d. Acantocitos
148. La estomatocitosis hereditaria se caracteriza por alteraciones de la
permeabilidad ionica de la membrana, anemia moderada grave y la presencia de
estomatocitos (verdadero)
149. ¿Cuál de estos síntomas o signos NO pertenece a la hemoglobinopatía de alta
afinidad por el oxigeno?
a. Plétora
b. Hematocrito 60%
c. Cianosis
d. Mareo
150. En pacientes con hipoxia y aspecto cianótico, con PaCo2 suficientemente alta
como para que la hemoglobina este saturada por completo de O2. ¿Qué patología se
debe sospechar?
a. Metahemoglobinemia
b. Hemoglobinopatías con afinidad alterada por el oxigeno
c. Hemoglobinopatías con alteración de carga superficial
d. Hemoglobinopatías inestables
151. La talasemia es la elevación parcial o total de la síntesis de una de las cadenas
de hemoglobina (falso)
152. ¿Cuál de estas opciones no pertenece a las b-talasemias?
a. B-talasemia menor
b. B-talasemia intermedia
c. Delta-Beta talasemia
d. Alfa talasemia
153. ¿Cuál de estas talasemias NO necesita tratamiento?
a. Rasgo B-talasemico
b. B-talasemia mayor
c. B-talasemia intermedia
d. Todas
154. ¿Cuáles son las anemias hemolíticas mecánicas?
a. Hemoglobinuria de la marcha
b. Hemolisis por valvulopotia
c. Hemolisis microangiopatica
d. Todas
155. El síndrome de Zieve es la afectación metabolica aguda de presentacion por
abuso prolongado de alcohol, con anemia hemolítica, hiperlipoproteinemia, icteria y
dolor abdomial (Verdadero)
156. El síndrome mielodisplásico. ¿Cuál es la serie celular que se encuentra
afectada?
a. Serie eritrocitaria
b. Serie granulocitica
c. Serie plaqueraria
d. Todas
157. El síndrome mielodisplásico secundario, es originado por:
a. Quimioterapia
b. Lesiones citotóxicas
c. Activación y supresión de genes
d. Todos
158. En el síndrome mielodisplásico. ¿Cuál es las población con mayor
predisposición a desarrollarlo?
a. Niños
b. Mujeres en edad fértil
c. Adultos mayores
d. Adultos sin predisposición de sexo
159. ¿Cuál es la presentacion clínica mas frecuente de SMD?
a. Anemia
b. Dolor oseo
c. Febrícula
d. Perdida de peso de evolución rápida
160. En el caso de que exista la presencia de formas eritroblasticas en el análisis
sanguíneo. ¿Cuál es el porcentaje que indica SMD?
a. Mayor a 20%
b. Menor a 20%
c. Mayor a 40%
d. Menor a 40%
161. Cual es el método diagnostico de SMD?
a. Netamente clínico
b. De laboratorio
c. Reducción de la densidad osea por densitometría osea
d. Biopsia de medula osea
162. El motivo de sonsulta mas común en los pacientes con SMD es:
a. Hematomas y hemorragias
b. Infecciones recurrentes
c. Síndrome anémico: fatiga, disnea, taquicardia, cefalas
d. Todas las anteriores
163. Los siguientes antecedentes son de gran relevancia en el diagnostico de SMD:
a. Radioterapia y medicamentos
b. Medicamentos e infecciones previas
c. Radioterapia y antibioticoterapia con aminoglucosidos
d. Antecedentes de infecciones por VEB y VIH
164. La evolución natural del síndrome mielodisplásico es:
a. Leucemia linfoblastica aguda
b. Leucemia mieloide crónica
c. Leucemia mieloide aguda
d. Leucemia linfoblastica aguda
165. La OMS para la clasificación de los síndromes mielodisplásicos, toma en cuenta
los siguientes aspectos:
a. La hemoglobina y reticulocitos
b. Reticulocitos y Citometría ósea
c. Citometría de medula ósea y formas de glóbulos rojos en frotis sanguíneo
d. Ninguna
166. Las anomalías eritroides que se encuentran en el SMD son:
a. Sideroblastos en anillos, eritroblastos con mitocondrias cargadas de hierro y
granulos perinucleares en los aspirados o biopsias
b. Es patognomónico las anomalías de Pelger-Huet
c. Con mucha frecuencia se evidencia las anomalías de May-Hegglin
d. Las anomalías de Undritz es suficiente para hacer el diagnostico
167. ¿En que porcentaje se suele encontrar esplenomegalia en pacientes con
Trombocitopenia?
a. 15%
b. 90%
c. 10%
d. 50%
168. ¿Cuáles de las siguientes opciones NO pertenecen a regiones donde se da
Trombocitosis arterial?
a. Coronaria
b. Senos cerebrales
c. Cebrovascular
d. Vascular periférico
169. La Trombocitosis esencial se caracteriza por: EXCEPTO
a. Trombocitosis persistente
b. Riesgo aumentado de trombosis
c. Hemorragias continuas
d. Hiperplasia megacariocitica en medula osea
170. El 40-50% de pacientes se hallan asintomáticos en el momento del diagnostico
(V)
171. La trombosis venosa es mas frecuente que la arterial (F)
172. Las manifestaciones hemorrágicas son muy frecuentes y se relacionan con
antiagregantes (F)
173. Señale lo correcto: Trombocitosis esencial
a. Ferritina sérica alta
b. LDH disminuido
c. Medula osea con megacariocitos anormales grandes y nucleos
hiperlobulados
d. Plaquetas 450 x 108 – 1000 x 108
174. ¿A que se define mielofibrosis?
a. Fibrosis intensa de la medula osea y metaplasia linfoide de otros órganos
b. Fibrosis intensa de la medula espinal y metaplasia mieloide de otros órganos
c. Fibrosis intensa de la medula espinal y metaplasia linfoide de otros órganos
d. Fibrosis intensa de la medula osea y metaplasia mieloide de otros órganos
175. ¿Cuáles son las principales causas de mielofibrosis secundarias?
a. Tuberculosis, tumores solidos y neoplasias hematológicas
b. Aspergilosis, tumores metastasicos y neoplasias oseas
c. Tuberculosis, tumores solidos y neoplasias oseas
d. Aspergilosis, tumores metastasicos y neoplasias hematológicas
176. ¿Cuál es la causa de mielofibrosis?
a. Proliferación anómala de osteoclastos
b. Destrucción de células madres pluripotenciales
c. Proliferación clonal de una celula madre pluripotencial
d. Destrucción de la medula osea por un tumor
177. ¿Qué factor produce la angiogénesis en la mielofibrosis?
a. Factor de crecimiento derivado de las plaquetas
b. Factor de crecimiento del endotelio vascular
c. Factor de crecimiento transformante beta
d. Factor de crecimiento transformante alfa y gamma
178. ¿Qué produce la estimulación de los fibroblastos en la mielofibrosis?
a. Factor de crecimiento derivado de las plaquetas
b. Factor de crecimiento del endotelio vascular
c. Factor de crecimiento transformante beta
d. A y C son correctas
179. ¿Qué produce la estimulación de los osteoblastos en la mielofibrosis?
a. Factor de crecimiento derivado de las plaquetas
b. Factor de crecimiento del endotelio vascular
c. Factor de crecimiento transformante beta
d. A y C son correctas
180. ¿Cuáles son las manifestaciones principales de la mielofibrosis?
a. Anemia microcitica-microcromica, hipermetabolismo y metaplasia linfoide
b. Anemia normocítica-normocromica, hipermetabolismo y metaplasia
mieloide
c. Anemia macrocitica, hipometabolismo y metaplasia mieloide
d. Anemia macrocitica, hipermetabolismo y metaplasia mieloide
181. ¿Cuál es la causa mas común por la que llegan los pacientes con mielofibrosis a
la consulta y ayuda diagnostico?
a. Esplenomegalia con dolor en flanco izquierdo irradiado a la espalda
b. Hepatomegalia con dolor en flanco izquierdo irradiado a la pierna
c. Esplenomegalia con dolor en fosa iliaca derecha irradiada a la espalda
d. Hepatomegalia con dolor en flanco irradiado a la espalda
182. El gen afectado en la mielofibrosis en un 50% es el gen JAK2 (verdadero)
183. ¿Qué alteración cromosómica se puede encontrar en la mielofibrosis?
a. Delecion 13q14
b. Delecion17q20
c. Delecion 14q13
d. Delecion 17q14
184. ¿Cuál de las fases crónicas de LMC es la mas estable y benigna?
a. Fase crónica
b. Fase de resistencia
c. Fase blastica
d. Todas las anteriores
185. ¿Cuál de los siguientes hallazgos es característico de la leucemia mieloide
crónica?
a. LDH disminuida
b. Vitamina B12 disminuida
c. Acido urico disminuido
d. Fosfatasa alcalinca disminuida
186. Según la clasificación de la OMS de las neoplasias de origen linfoide. La
leucemia linfocítica crónica pertenece a:
a. Neoplasias maduras de origen T y NK
b. Neoplasias maduras de origen B
c. Neoplasias de precursores B o T
d. Ninguna de las anteriores
187. Con respecto al linaje de la enfermedad el 2% de los casos pertenece a:
a. Linfocitos T
b. Linfocitos NK
c. Linfocitos B
d. Ninguna de las anteriores
188. ¿Cuál de estas no es una causa de síndrome anémico del LLC?
a. Hiperesplenismo
b. Destrucción microangiohepatica de eritrocitos
c. Aplasia pura de células rojas
d. Anemia hemolítica autoinmune (Coombs+)
189. En el frotis de sangre periferica se encuentran manchas de Gumprecht en la
leucemia linfocítica crónica (V)
190. ¿En que pacientes hay abstención terapéutica y controles periódicos?
a. Pacientes sintomáticos con hepatomegalia severa
b. Pacientes asintomáticos
c. Solo pacientes que presentan adenopatías
d. Ninguna de las anteriores
191. El diagnostico de la anemia hemolítica se realiza gracias a cinco signos
biológicos características: elevación de los reticulocitos, positividad en test de coombs
directo, hiperbilirrubinemia no conjugada, incremento de lactatodeshidrogenasa
sérica (LDH) y descenso de haptoglobina. ¿Cuál de estos signos biológicos pueden
observarse también en las perdidas de sangre por hemorragias?
a. Descenso de la haptoglobina y positividad en test de coombs directo
b. Elevación de LDH y bilirrubina no conjugada
c. Descenso de haptoglobina y elevación de LDH
d. A y C son correctas
192. Indique lo correcto:
a. Las anemias asociadas a insuficiencia endocrina suelen ser microciticas
b. Las anemias inflamatorias o de trastorno crónico suelen ser normociticas
c. La uremia no suele ir acompañada de anemia
d. En el déficit de vitamina B12 la anemia siempre precede a la enfermedad
neurológica
193. Una mujer de 52 años presenta un síndrome anémico sin causa clínica
aparente. En la analítica se aprecia hemoglobina de 8.5 mg/dl, VCM 75 fl, HCM 25 pg.
¿Cuál de las siguientes determinaciones séricas le permitirá orientar con mas
seguridad el origen ferropenico de la anemia?
a. Sideremia
b. Saturación de tranferrina
c. Hemoglobina
d. Ferritina
194. En un paciente con anemia microcitica e hipocromica, con hierro serico y
ferritina bajos, aumento de la capacidad de fijación de hierro, nos encontramos ante
un cuadro de:
a. Anemia por deficiencia de hierro
b. Anemia de trastorno crónico
c. Anemia sideroblactica
d. Talasemia
195. Mujer de 32 años sometida a cirugía de valvula mitral hace 1 año, consulta por
presentar astenia progresiva desde hace 3 meses, que se ha acentuado en los últimos
días. Describe la aparición de hematomas espontaneas en extremidades inferiores, no
presenta epistaxis, gingivorragia ni hematuria. Al examen físico buen estado general,
intensa palidez, mucocutanea. Laboratorio HB 6.8 g/dl, HTo 20%, VCM 83.3 fl,
leucocitos 2.100/ul, neutrófilos 30%, linfocitos 59%, plaquetas 250 000/ul, LDH
normal, bilirrubina indirecta < 3, hierro serico 65pg/dl. ¿Cuál de los siguientes
diagnosticos explicaría el cuadro clínico?
a. Anemia de células falciformes
b. Anemia por enfermedad crónica
c. Anemia aplásica
d. Anemia perniciosa
196. La deficiencia enzimática mas común del eritrocitico que ocasiona anemia
hemolítica, es:
a. Decifiencia piruvatocinasa
b. Deficiencia de glucosa 6 fosfato deshidrogenasa
c. Deficiencia de fosfofructocinasa
d. Deficiencia de aldolasa
197. La principal causa de anemia aplásica es:
a. Inmunológica
b. Infecciosa
c. Idiopática
d. Radiación
Banco de preguntas grupo 11

PREGUNTAS

Entre las medidas no farmacológicas de nefro protección encontramos:

A. Dieta para el control metabólico y restricción de sal.

B. Control de exceso de peso.

C. Restricción de proteínas en caso de ERC Estadio II, III, IV.

D. Todas las anteriores

ANSWER: D

El tratamiento de mantenimiento con IECA Aumenta el aclaramiento de insulina y el FSR, a la vez que
disminuye la proteinuria:

A. Verdadero

B. Falso

C. Aumenta el aclaramiento de insulina y disminuye el FSR

D. Disminuye el FSR y la proteinuria.

ANSWER: A

En la necrosis tubular aguda por nefrotoxicidad se dice que es el efecto del fármaco sobre el:

A. Intersticio renal

B. Epitelio tubular renal

C. Todas las anteriores

D. Ninguna de las anteriores

ANSWER: B

En la nefritis túbulo-intersticial por nefrotoxicidad se dice que es el efecto del fármaco sobre el:

A. Intersticio renal

B. Epitelio tubular renal

C. Todas las anteriores

D. Ninguna de las anteriores

ANSWER: A

La nefritis túbulo-intersticial por nefrotoxicidad es causada por fármacos como Ciclosporina, Litio,
AINES:
Banco de preguntas grupo 11

A. Verdadero

B. Falso

C. Es producida solo por AINES.

D. Es producida solo por Litio y Ciclosporina

ANSWER: A

La obstrucción intratubular es causada por fármacos como: Metrotexato, Aciclovir, Sulfonamidas,


Indinavir:

A. Verdadero

B. Falso

C. Causada por Metrotexato y Aciclovir.

D. Causada solo por Sulfonamidas.

ANSWER: A

¿De los siguientes ítems, Cual No corresponde a los objetivos de la Nefrotección?

A. HbA1C ≤ 6,5%

B. PA < 130/80 mmHg

C. Proteinuria > 0,5gr/24h

D. Cese de hábito tabáquico.

ANSWER: C

¿Cuál de los siguientes medicamentos son considerados primera elección para la nefroprotección en
ERC?

A. Diuréticos y Antagonistas de calcio.

B. ARA II y Diureticos.

C. IECA y B bloqueantes.

D. IECA y ARA II

ANSEWR: D

¿Cuál de los siguientes NO corresponde a efectos renales de las IECA?

A. Bloqueo de efectos directos de la angiotensina II


Banco de preguntas grupo 11

B. Bloqueo del efecto vasoconstrictor de las catecolaminas.

C. Contribuye a mantener la TPG y la Volemia.

D. Disminuye la degradación de cininas.

ANSWER: C

¿Cuál de los siguientes NO corresponde a efectos renales de los ARA II?

A. Vasoconstricción de arterias preglomerulares.

B. Disminución del flujo medular.

C. Contracción de células mesangiales.

D. Actúa sobre la perfusión renal limitando la vasoconstricción y mejora la perfusión y aumenta la


formación de prostaglandinas.

ANSWER: D

El síndrome hepatorrenal (SHR) es una forma característica de insuficiencia renal que ocurre en los
pacientes con cirrosis hepàtica, de carácter funcional que aparece como consecuencia de una intensa:

A. vasoconstricción de la circulación renal secundaria a la disfunción circulatoria sistémica

B. vasodilatacionn de la circulación renal secundaria a la disfunción circulatoria sistémica

C. entidad que aparece en fases avanzadas de la cirrosis

D. alteración de la función renal

ANSWER :A

Cuál es el incremento de Creatinina sérica para que se manifieste como (SHR) tipo 1

A. >1.5 mg/dl

B. >1.5 mg /dl

C. >2.5mg/dl

D. >3mg /dl

ANSWER :C
Banco de preguntas grupo 11

Hay dos tipos de SHR: Como se manifiesta el SHR tipo 1 :

A. Inicio rápido, evolución progresiva y severa, con una supervivencia media de 15 días sin
tratamiento

B. Menos grave y progresivo, con una supervivencia media de 6 meses.

C. Evolución progresiva con una supervivencia media de 4 semanas

D. Evolución menos progresiva con una supervivencia media de 1 año

ANSWER :A

¿Como subdividió el club internacional de ascitis los criterios diagnósticos?

A. 2 tipos: de acuerdo a su velocidad y severidad de instauración

B. 3 tipos: de acuerdo a su velocidad, severidad de instauración y edad

C. Solo es de 1 tipo: de acuerdo a su velocidad

D. Solo es de 1 tipo: severidad de instauración

ANSWER: A

¿En qué año el Club Internacional de Ascitis estableció los criterios diagnósticos?

A.1994

B.1995

C.1996

D.1993

ANSWER: A

¿Cuál es el tratamiento farmacológico para el Hepatorrenal?

A. Vasoconstrictores y albumina

B. Trasplante hepático

C.Derivacion percutánea portosistémica intrahepática


Banco de preguntas grupo 11

D. DIALISIS

ANSWER: A

Señale. En el Síndrome Hepatorrenal. ¿Cuál es el primer mecanismo de resistencia vascular sobre la


presión arterial, a comienzos de la cirrosis en el paciente?

A. Disminución de la presión oncótica en el riñón

B. Aumento del gasto cardíaco

C. Aumento de endocannabinoides en el hígado

D. Disminución de la creatinina sérica

ANSWER: B

Señale. En el Síndrome Hepatorrenal. ¿Cuál es el primer mecanismo vasoconstrictor que se activa


después de fallar el aumento del gasto cardíaco en el paciente con cirrosis?

A. Hipersecreción de vasopresina

B. Aumento del gasto cardíaco

C. Sistema nervioso simpático

D. Sistema Renina – Angiotensina - Aldosterona

ANSWER: D

Señale. En el Síndrome Hepatorrenal. ¿Cuál es el objetivo a mantener en el paciente cirrótico, cuando


ya se han gastado todos sus mecanismos vasoconstrictores de compensación?

A. Mantener el volumen y la presión arterial constantes

B. Aumentar el gasto cardíaco

C. Disminuir la hipertensión portal

D. Evitar una complicación, como la peritonitis

ANSWER: A

Señale. En el Síndrome Hepatorrenal. ¿Cuál es la triada patológica que se ocasiona por retener sodio y
liberar agua a causa de la vasoconstricción renal?

A. Insuficiencia renal, diaforesis, alza térmica


Banco de preguntas grupo 11

B. Taquicardia, ascitis, fatiga

C. Ascitis, Edema, Hiponatremia dilucional

D. Edema, Shock séptico, Cirrosis

ANSWER: C

1. ¿Cuál es el agente etiológico más prevalente en un ITU de ámbito comunitario?

a) Proteus mirabilis

b) Escherichia coli

c) Proteus mirabilis

d) Klebsiella spp.

ANSWER: B

2. ¿Cuál es el agente etiológico más prevalente en un ITU en varones ancianos mayores?

a) Enteroccocus fecalis

b) Proteus mirabilis

c) Escherichia coli

d) Klebsiella spp.

ANSWER: A

3. Seleccione lo correcto. ¿Cuál es la vía MAS COMÚN de los gérmenes en una ITU?

a) Vía hematógena

b) Vía linfática

c) Vía ascendente

d) Vía oral

ANSWER: C

4. Seleccione la opción correcta. En la bacteriuria asintomática en mujeres gestantes ¿cuál es la


mejor opción terapéutica?

a) Al ser una colonización y no una infección, no requiere antibiótico terapéutico

b) No se debe tratar por que provocaría complicaciones fetales (RN de bajo peso y prematuridad)

c) Se recomienda Nitrofurantoína en el primer trimestre del embarazo


Banco de preguntas grupo 11

d) Se recomienda un tratamiento corto con uso seguros de betalactámicos según el germen


aislado y su antibiograma

ANSWER: D

5. Uno de los microorganismos que usualmente produce una pielonefritis por diseminación por
vía hematógena es:

a) S. aureus

b) K. pneumoniae

c) E. coli

d) P. mirabilis

ANSWER: A

6. Se define como infección recurrente del tracto urinario a la presencia de:

a) 2 o más ITU sintomáticas en 6 meses

b) 3 o más ITU sintomáticas en 12 meses

c) 2 ITU sintomáticas en un periodo de 12 meses

d) a y b son correctas

ANSWER: D

7. El microorganismo predominante que produce pielonefritis por vía ascendente es:

a) K. pneumoniae

b) P. mirabilis

c) E. coli

d) S. aureus

ANSWER: C

8. Seleccione los factores predisponentes de ITUs complicadas excepto

a) Diabetes

b) Obstrucción urinaria

c) Hepatitis

d) Reflujo vesicouteral

ANSWER: C
Banco de preguntas grupo 11

9. Como tratamiento empírico de ITUS complicadas se debe evitar:

a) Clavulánico

b) Amoxicilina

c) fluroquinaloas

d) todas las anteriores

ANSWER: D

10. Seleccione los enteropatógenos más frecuentes en ITUs

a) enterobacterias

b) E COLI

c) Proteus klebsiella

d) Todas las anteriores

ANSWER: D

En la ITU complicada se presentan los siguientes signos y síntomas, excepto

a) Disuria

b) Urgencia miccional

c) Polidipsia

d) Polipiuria

ANSWER: C

¿Cuál de los siguientes no corresponde a un cambio a nivel renal experimentado en el embarazo?

A. Aumento del tamaño del riñón

B. Hidronefrosis

C. Aumento del filtrado glomerular

D. Disminución de la actividad del SRAA.

ANSWER: D

Señale la incorrecta respecto a la hidronefrosis del embarazo:

A. Es más prominente del lado derecho

B. Se cree que es debido a la peristalsis disminuida por la progesterona


Banco de preguntas grupo 11

C. Se puede visualizar en el segundo trimestre del embarazo

D. Se resuelve en la primera semana después del parto.

ANSWER: D

¿Por qué se produce el aumento del gasto cardíaco durante la gestación? Señale la incorrecta:

A. Aumento de la masa eritrocitaria

B. Aumento del volumen sistólico

C. Reducción de la resistencia periférica

D. Aumento de la frecuencia cardiaca

ANSWER: A

Respecto a la anemia fisiológica del embarazo señale la incorrecta:

A. Se produce debido a un aumento del volumen plasmático sin un aumento de la masa


eritrocitaria

B. El aumento del volumen plasmático favorece las hemorragias postparto

C. El aumento del volumen plasmático provee un mayor flujo renal y placentario

D. Facilita la llegada de nutrientes al feto

ANSWER: B

Señale el enunciado incorrecto respecto a preeclampsia/eclampsia:

A. La más frecuente de las enfermedades hipertensivas del embarazo es la


preeclampsia/eclampsia.

B. La preeclampsia es la hipertensión que aparece después de las 20 semanas de gestación


acompañada de proteinuria significativa.

C. Se denomina eclampsia cuando la hipertensión se acompaña de convulsiones y/o coma.

D. Es una enfermedad exclusiva del embarazo humano, sin predisposición familiar.

ANSWER: D

De acuerdo con los signos y síntomas de preeclampsia/eclampsia, señale lo incorrecto:

A. La preeclampsia puede ser asintomática o causar edema o un aumento importante del peso.

B. El edema en los sitios no declive, como el rostro o las manos (la paciente no puede quitarse los
anillos de los dedos), es más específico que el edema en las regiones declive.
Banco de preguntas grupo 11

C. La actividad refleja puede estar aumentada, lo que indica irritabilidad neuromuscular, que
puede progresar en convulsiones (eclampsia).

D. No hay petequias y otros signos de coagulopatía.

ANSWER: D

En ausencia de proteinuria, la preeclampsia también se diagnostica si las mujeres embarazadas tienen


hipertensión de reciente comienzo junto con aparición de cualquiera de los siguientes, excepto:

A. Trombocitopenia (plaquetas < 100.000/microL)

B. Insuficiencia renal (creatinina sérica > 1,1 mg/dL o duplicación de la creatinina en suero en
mujeres sin enfermedad renal)

C. Deterioro de la función hepática (transaminasas > 2 veces el valor normal)

D. Sin síntomas cerebrales o visuales.

ANSWER: D

¿Cuál de estas no es una complicación del síndrome HELLP?

A. Isquemia hacia los órganos

B. Convulsiones.

C. Anemia

D. Trombosis venosa

ANSWER: D

Señale cuál de las siguientes manifestaciones no corresponde al síndrome de HELLP:

A. Hipertensión

B. Proteinuria

C. Elevación de enzimas hepáticas

D. Hiponatremia

ANSWER: D

En el 25% de los casos de síndrome de HELLP puede encontrarse:

A. Hiponatremia

B. Trastornos de visión y audición

C. Epigastralgia
Banco de preguntas grupo 11

D. Hipertensión

ANSWER: B

Defina. Qué es la poliquistosis renal

A: Es una enfermedad genética común que consiste en la aparición progresiva de lesiones quísticas en
los riñones

B: Los quistes remplazan el parénquima renal

C: Se produce una enfermedad renal crónica en estadio V

D: Todas son correctas

ANSWER: D

Cuáles son los patrones de herencia que se presenta en la poliquistosis renal

A: Autosómica Dominante (PKD1 y PKD2)

B: Autosómica recesiva (PKHD1).

D: Autosómica dominante y recesiva PKDY2 y PKHD3

E: Solo a y b son correctas

ANSWER: E

Cuáles son los cromosomas involucrados en la poliquistosis renal

A: el gen PKD1, localizado en el cromosoma 16

B: En el gen PKD2, localizado en el cromosoma 4

C: Tercer gen, PKD3, que aún no ha sido identificado

E: Todas son correctas

ANSWER: E

Prevalencia de la Poliquistosis Renal en el Mundo

A: 1 de cada 1000 personas

B: 1 de cada 500 personas

C: 10 de cada 1000 personas


Banco de preguntas grupo 11

D: 10 de cada 500 Personas

ANSWER: A

¿En la etiología de la Poliquistosis Renal, cual es el Gen que causa mayor severidad en la presentación
clínica?

A: PKD2

B: PKD1

C: PKD3

D: PKD4

ANSWER: B

¿En pacientes que tienen mutación del Gen PKD1 sus riñones se encuentran?

A: Aumentados de Tamaño

B: Disminuidos de Tamaño

C: Presenta una función renal disminuida

D: Presentan quistes

ANSWER: A

¿CUÁL es el gen y la proteína afectados en la PQRAR?

A: PKHD1 Fibrocistina

B: PKD 1/PKD 2 - Policistina 1/ Policistina 2

C: NPHP1 - Nefrocistina

D: HNF-1b - TCF2

E: No es una enfermedad genética

ANSWER: A
Banco de preguntas grupo 11

¿QUÉ debemos vigilar en la displasia multiquísica renal?

A: Tensión arterial

B: Malformaciones genitourinarias contralaterales

C: Crecimiento compensador del riñón contralateral

D: Todas las anteriores

ANSWER: D

¿Cuál es la EDAD más frecuente de presentación de la PQRAD?

A: 20-30 años, que son la 3ª -4ª década de la vida.

B: Intraútero.

C: Periodo de lactante (< 2 años).

D: Tercera edad

E: Infancia (2-12 años).

ANSWER: A

¿Cuál es la EDAD más frecuente de presentación de la PQRAD?

A: 20-30 años, que son la 3ª -4ª década de la vida.

B: Intraútero.

C: Periodo de lactante (< 2 años).

D: Tercera edad

E: Infancia (2-12 años).

ANSWER: A

1.) ¿El síndrome cardiorrenal tipo 1 a que hace referencia?

a.) Deterioro crónico de la función renal como consecuencia de un deterioro crónico de la


función cardiaca.

b.) Disfunción cardiaca aguda induce lesión o disfunción renal aguda.

c.) Enfermedad cardiovascular que acompaña a la ERC.

d.) Lesión concomitante renal y cardiaca aguda o crónica secundaria a una enfermedad
sistémica aguda o crónica.
Banco de preguntas grupo 11

2.) Señale el enunciado correcto.

a.) El deterioro agudo de la función cardiaca produce alteraciones hemodinámicas, entre las
que se incluyen reducción del volumen sistólico y del gasto cardiaco.

b.) El deterioro agudo de la función cardiaca produce alteraciones hemodinámicas, entre las
que se incluyen disminución de la presión en aurícula derecha y congestión venosa, que
conducen a un descenso del filtrado glomerular.

c.) El deterioro agudo de la función cardiaca produce alteraciones hemodinámicas, entre las
que se incluyen aumento del volumen sistólico y del gasto cardiaco.

d.) Ninguna.

3.) ¿Qué fármaco se utiliza como primera línea en el tratamiento del síndrome cardiorrenal tipo 1?

a.) IECAs.

b.) ARA II.

c.) Diuréticos.

d.) Beta-bloqueantes.

4.) De acuerdo a los tipos de Síndrome Cardiorenal, indique cuál pertenece al tipo 4:

a. Se caracteriza por afectación cardiovascular en pacientes con enfermedad renal crónica

b. Se caracteriza por la presencia simultánea de disfunción renal y cardiaca, asociada a


trastornos sistémicos agudos o crónicos

c. Deterioro crónico de la función renal como consecuencia de un deterioro crónico de la


función cardiaca

d. Disfunción cardiaca aguda induce lesión o disfunción renal aguda

5.) Señale lo correcto de acuerdo con el síndrome cardiorrenal secundario o Tipo 5, se caracteriza
por:

a. Choque séptico y el hemorrágico de origen traumático

b. Las quemaduras múltiples, el lupus eritematoso sistémico, la diabetes mellitus


Banco de preguntas grupo 11

c. activación del sistema nervioso simpático conlleva una estimulación del sistema renina-
angiotensina-aldosterona que contribuye a la liberación de angiotensina II

d. A y B son correctas

6.) De acuerdo con las Estrategias terapéuticas en el manejo del síndrome cardiorrenal, indique cual
es el blanco terapéutico en relación a la anemia y alteración del hierro

a. Hemodiálisis de alto flujo

b. Inhibidores de la enzima convertidora de angiotensina

c. Carnitina

d. Inhibidores de renina

7.) De acuerdo con los criterios de diagnósticos de la insuficiencia cardiaca y renal. Indique cuales se
presentan en la insuficiencia cardiaca crónica:

a. Vértigo, síncope, dolor torácico

b. Fiebre y malestar, deshidratación, edema

c. Hipertermia, hipotermia, fiebre y escalosfrios

d. Ortopnea, disnea de esfuerzo, ritmo de galope, hepatomegalia

8). EL SINDROME CARDIORRENAL TIPO 1 SE DA :

a) activación neural , alteraciones hemodinámicas y congestión venosa , reacción del eje hipotálamo -
hipofisis , inflamacion de señalización

b) activacion neumoral , alteraciones hemodinámicas y congestión venosa , reacción del eje hipotálamo
-hipófisis , inflamación de señalización

c) activación numeral , alteraciones dinámicas y congestion arteriosa , reacción del eje hipotálamo -
hipófisis , inflamación de señalización

d) ninguna de las anteriores

9). SINDROME CARDIORRENAL TIPO 2 EL TRATAMIENTO SE BASA

a) Se basa en una combinación de dieta, ejercicio y tratamiento de las enfermedades subyacentes, Los
betabloqueantes , Los IECAs
Banco de preguntas grupo 11

b)los vasodilatadores es disminuir las resistencias periféricas con el fin de reducir la regurgitación mitral,
aumentar el flujo sanguíneo y disminuir las presiones de llenado

c)diuréticos de asa y vasodilatadores, junto con el tratamiento de las arritmias y la suspensión de los
agentes nefrotóxicos.

D)Ninguno de los anteriores

10. EN LA FISIOPATOLOGÍA CARDIORRENAL TIPO 3 SE DA

a) El daño renal agudo se asocia a activación neuroendocrina y del sistema inmunitario. La activación
del sistema nervioso simpático es característica tanto en el FRA como en la insuficiencia cardiaca aguda

b) El deterioro agudo de la función cardiaca produce alteraciones hemodinámicas, incluyen


reducción del volumen sistólico y del gasto cardiaco, elevación de la presión en aurícula
derecha

c) Las anormalidades neurohumorales, con excesiva producción de mediadores


vasoconstrictores (epinefrina, angiotensina y endotelina) y alteración de la liberación de
vasodilatadores endógenos

d) Ninguna de los anteriores

La definición de síndrome nefrítico consiste en un cuadro caracterizado por las siguientes


características, excepto...

A. Hematuria

B. HTA

C. Polaquiuria

D. Edema

ANSWER; C

Todas son causas primarias de glomerulonefritis, excepto...

A. Glomerulonefritis postinfecciosa

B. Membrano proliferativa

C. Mesangial IgA

D. Goodpasture
Banco de preguntas grupo 11

ANSWER; D

En la endocarditis bacteriana aguda, están presente los siguientes depósitos inmunitarios, excepto…

A. IgA

B. IgM

C. Fracción C3 del complemento

D. IgG

ANSWER; A

Señale lo INCORRECTO, los criterios diagnósticos de sx nefrítico son:

A. Edema

B. Elevación creatinina y urea por días o meses

C. Hematuria menos de 5 hematíes por campo

D. Hipertensión

ANSWER; C

Las Glomerulonefritis asociadas a ANCA – C son:

A. Granulomatosis de Wegener

B. Granulomatosis de Wegener y Sx Goodpasture

C. Granulomatosis de Wegener y Poliangeitis microscópica

D. Granulomatosis de Wegener y Sx Churg Strauss

ANSWER; D

El tratamiento de elección para las sobrecargas leves o moderadas en el contexto de una


glomerulonefritis es:

A. Furosemida, 2-5 mg/kg/día por vía oral

B. Furosemida, 0,5-2 mg/kg/día por vía oral


Banco de preguntas grupo 11

C. Furosemida, 3 - 8 mg/kg/día por vía oral

D. Furosemida, 0,3 – 0,8 mg/kg/día por vía oral

ANSWER; B

En la glomerulonefritis, si a pesar de la restricción hidrosalina y el uso de diuréticos no se consigue


controlar la presión arterial, ¿qué fármacos se pueden asociar?

A. IECA

B. Hidralazina

C. Hidralazina o Nifedipino

D. IECA o Hidralazina

ANSWER; C

Porque en la glomerulonefritis ser produce reducción de los niveles séricos de complemento.

a) porque se depositan cristales

b) por aumento de los inmunocomplejos

c) deposito en la membrana basal de inmunocomplejos

d) ninguno es correcto

ANSWER: C

Cuando existe un aumento en el diámetro de los poros de la membrana basal en el síndrome nefrítico
que manifestaciones presenta en la orina.

a) hematuria- oliguria- hiperproteinuria

b) hematuria-proteinuria cilindros de GR

c) hematuria- anuria – proteinuria

d) ninguna es correcta

ANSWER: B

En el síndrome nefrítico cuando disminuye la superficie de filtración glomerular presenta

a) disminuye la tasa de filtración- oliguria- azoemia- edema

b) aumenta la tasa de filtración- anuria- proteinuria- edema


Banco de preguntas grupo 11

c) disminuye la tasa de filtración- poliuria- azoemia- sin edema

d) ninguna es correcta

ANSWER: A

1. ¿Cuál de las siguientes afirmaciones es correcta respecto a los criterios de diabetes?

a. Glucemia basal >= 126 mg/dl

b. Glucemia >= 100mg/dl dos horas tras la SOG con 75 g de glucosa

c. HbA1C >= 65%

d. Paciente sintomático con glucemia de 100 mg/dl

ANSWER: A

2. ¿En qué pacientes usted pensaría una diabetes de tipo MODY?

a. Adulto de 40 – 60 años, anticuerpos negativos

b. Adulto joven de 25 – 40 años, anticuerpos negativos

c. Joven de 18 – 25 años, anticuerpos negativos

d. Niños de 0 – 18 años, anticuerpos negativos

ANSWER: B

3. ¿Cuál es el diagnostico apropiado para nefropatía diabética?

a) Pedir microalbuminuria a los 5 años del diagnóstico de DM1

b) Pedir microalbuminuria inmediatamente después del diagnostico de DM2

c) Pedir microalbuminuria a los 5 años del diagnóstico de DM2

d) A Y B son correctas

ANSWER: D

4. ¿Cuál es el valor normal de la albuminuria?

a) > 30 mg/g

b) <30 mg/g

c) 60 – 100 mg/g

d) 90 mg/g

ANSWER: B
Banco de preguntas grupo 11

5. Valor normal de la tasa de filtrado glomerular (TFG)

a) Mayor o igual a 90 ml/min/1,73m2

b) 60 ml/min/1,73m2

c) 30 – 60 ml/min/1,73m2

d) 80 ml/min/1,73m2

ANSWER: A

6. El primer cambio funcional en la nefropatía diabética és:

a) Hiperfiltración

b) Albuminuria

c) Proteinuria

d) Disminución de la TFG

ANSWER: A

7. ¿Cuál de los siguientes no es un cambio estructural en una afección renal por diabetes?

a) Aumento del ovillo glomerular

b) Engrosamiento de la membrana basal glomerular

c) Glomeruloesclerosis

d) Aumento del tamaño tubular

ANSWER: D

8. ¿La fase de nefropatía incipiente se caracteriza por?

a. Hiperfiltración

b. Albuminuria

c. Proteinuria

d. Disminución de la TFG

ANSWER: B

9. ¿En el manejo del diabético tipo 2 con insuficiencia renal moderada (creatinina sérica > 2
mg/dl) cuál de las siguientes afirmaciones es falsa?

a) Es más importante el uso, en sí mismo, de un nefroprotector como IECAs que el control


estricto de la PA.
Banco de preguntas grupo 11

b) Es bastante frecuente que la causa de la insuficiencia renal no sea la nefropatía


diabética específica.

c) Habitualmente se requiere tratamiento farmacológico combinado para el control


estricto de la PA.

d) Las necesidades de insulina suelen disminuir porque, aunque se incrementa la


resistencia insulínica predomina la acumulación insulínica por el fallo renal.

ANSWER: A

10. ¿En el manejo del paciente diabético tipo 2 con nefropatía incipiente (microalbuminuria) cuál
de las siguientes afirmaciones es cierta?

a) Con el uso de IECAs a dosis bajas no es preciso hacer un control precoz de creatinina y
potasio séricos.

b) El abandono del tabaco probablemente influye favorablemente sobre la nefropatía del


diabético tipo 2.

c) En el diabético tipo 2 con microalbuminuria el objetivo de control glucémico debe ser


HbA1c <8,5%.

d) Hay que evitar el uso de tiazidas en el diabético tipo 2 con microalbuminuria.

ANSWER: C

Elija la correcta definición de diálisis peritoneal:

A. terapia de reemplazo renal basada en la infusión de una solución estéril en la cavidad peritoneal
a través de un catéter y permite la eliminación de solutos y agua utilizando la membrana
peritoneal como superficie de intercambio.

B. Convencional, de alta eficacia, de alto flujo, tradicional

C. Tratamiento inmunosupresor debe ser individualizado buscando la sinergia inmunosupresora y


el mejor perfil de seguridad, y debe adaptarse a las diferentes etapas.

D. Extracción de un riñón de un individuo previamente sano (cadáver o de vivo) y su colocación en


un enfermo con insuficiencia renal crónica.

ANSWER: A

Señale cuál de las siguientes opciones contiene contraindicaciones relativas de la diálisis peritoneal:

A. Limitación severa de capacidad respiratoria, imposibilidad de autodiálisis sin apoyo, enfermedad


inflamatoria intestinal activa
Banco de preguntas grupo 11

B. Negativa del paciente, múltiples adherencias quirúrgicas

C. Múltiples hernias/fugas de repetición

D. Enfermedad psiquiátrica grave

ANSWER: A

Señale cuál de las siguientes alternativas corresponde a las complicaciones de diálisis peritoneal:

A. Esclerosis peritoneal encapsulante, disfunción del catéter, edema y falla de ultrafiltración

B. Enfermedad psiquiátrica grave

C. Limitación severa de capacidad respiratoria, imposibilidad de autodiálisis sin apoyo, enfermedad


inflamatoria intestinal activa

D. Negativa del paciente, múltiples adherencias quirúrgicas

ANSWERS: A

¿Cuál de las siguientes es una indicación inmediata de hemodiálisis de urgencia en un paciente con
insuficiencia renal?

A. Sodio urinario menor a 120 mEq/L

B. Creatinina plasmática mayor a 6 mg/dl

C. Hiperkalemia grave que no responde a tratamiento médico

D. Oliguria menor a 500 ml/día

ANSWER: C

En cuanto a la hemodiálisis convencional nos referimos a:

A. dializadores de baja permeabilidad y baja superficie (1,2 – 1,6 m2) flujo sanguíneo de 200-300
ml/min, flujo de líquido de diálisis 500ml/min

B. mayor aclaramiento de solutos. Suoerficie de 1,8-2,2 m2, flujo sanguíneo de 300-400 ml/min,
flujo de líquido de diálisis 700-1000ml/min. Requiere control preciso de ultrafiltración

C. empleo de lializadones de alta permeabilidad (mayor aclaramiento). Requiere membranas


biocompatibles. Riesgo de bacteremia
Banco de preguntas grupo 11

D. dializadores de baja permeabilidad y baja superficie (2,5 – 5,6 m2) flujo sanguíneo de 100-200
ml/min, flujo de líquido de diálisis 800ml/min

ANSWER: A

Los tipos de hemodiálisis se clasifican en los siguientes excepto

A. convencional

B. de alta eficacia

C. de alto flujo

D. tradicional

ANSWER: D

¿Cuánto es el flujo sanguíneo en HD convencional?

A. 100-200ml/min

B. 200-300 ml/min

C. 300-400ml/min

D. 700-1000mi/min

ANSWER: B

¿De los criterios de exclusión de donantes vivos, cual no es considerada como contraindicación
absoluta?

A. Proteinuria > 300mg/24 h

B. FG anormal para la edad

C. Hematuria

D. Infección por VIH

ANSWER: D

De las siguientes son contraindicaciones relativas de criterios de donantes vivos, excepto:

A. Trastornos psiquiátricos

B. Infección activa crónica (Hepatitis B/C)


Banco de preguntas grupo 11

C. Obesidad

D. Tumor maligno creciente.

ANSWER: D

Los siguientes son criterios de AMSTERDAM de aceptación de donante vivo, excepto:

A. IMC < 35 kg/m2

B. Albuminuria > 300 mg/24h

C. Filtrado glomerular > 80ml/min/1.73m2

D. Ausencia de antecedente de DM

ANSWER: B

¿CUÁL DE LOS SIGUIENTES NO ES UN CRITERIO CLINICO Y BIOQUÍMINO DEL SIÍDROME NEFRÓTICO?

A. Anasarca

B. Hipoproteinemia

C. Anemia ferropénica

D. Cociente proteinuria/creatinuria

E. Hipercolesterolemia

ANSWER: C

¿CUÁL DE LOS SIGUIENTES SI ES UN METODO DE MEDICIÓN DE PROTEINURIA EN EL SÍNDROME


NEFRÓTICO?

A. Eco doppler

B. Colirométrico

C. Coagulación

D. Iones en biometría

E. Recuento leucocitario

ANSWER: B
Banco de preguntas grupo 11

¿CUÁL DE LOS SIGUIENTES NO ES UN CRITERIO PARA REALIZAR UNA BIOPSIA RENAL EN NIÑOS EN
CASO DE SOSPECHA DEL SÍNDROME NEFRÓTICO?

A. Insuficiencia renal

B. Nefritis familiar

C. Cortico resistencia

D. Macro/micro hematuria

E. Hemoglobina < 8

ANSWER: E

¿CUÁL DE LOS SIGUIENTES NO ES UN DIAGNOSTICO DIFERENCIAL DEL SÍNDROME NEFRÓTICO?

A. Enteropatía perdedora de proteínas

B. Insuficiencia hepática

C. Glomerulonefritis aguda o crónica

D. Malnutrición protéica

E. Escoliosis

ANSWER: E

SELECCIONE LA INCORRECTA: ¿EN CUÁL DE LOS SIGUIENTES CASOS SE DEBE PROCEDER A LA


HOSPITALIZACION EN CASO DE SOSPECHA DE SINDROME NEFRÓTICO?

A. Presencia de edema severo

B. Sobreinfección

C. Oliguria

D. Diabetes diagnosticada hace más de 5 años

E. Azoemia

ANSWER: D

Todo lo que sigue acerca del síndrome nefrótico en la infancia, es cierto EXCEPTO:

A. Colesterol sérico elevado.

B. El 85% experimenta cambios mínimos de la enfermedad.


Banco de preguntas grupo 11

C. Reabsorción reducida de sodio por el riñón.

D. Triglicéridos séricos elevados.

E. La hipoalbuminemia es la causa de la hipoproteinemia.

ANSWER:C

Señala cuál entre las siguientes nefropatías primarias NO se presenta clínicamente cómo síndrome
nefrótico más que de modo excepcional:

A. La glomerulopatía membranosa.

B. La glomeruloesclerosis focal y segmentaria.

C. La enfermedad de cambios mínimos.

D. La nefropatía por IgA (enfermedad de Berger).

E. La nefropatía asociada con el VIH, sin o con colapso glomerular.

ANSWER:D

En un paciente con síndrome nefrótico, las medidas generales no específicas, para corregir la proteinuria
incluye uno de los siguientes procedimientos:

A. Dieta hiperproteica.

B. Diuréticos y/o b-bloqueantes.

C. Inhibidores del enzima conversor de la Angiotensina (IECAs).

D. Calcioantagonistas no dihidropiridínicos.

E. Corticoides por vía sistémica.

ANSWER:C

Todas las nefropatías primitivas enumeradas a continuación se manifiestan típicamente como síndrome
nefrótico, EXCEPTO una. Señálela:

A. Glomerulonefritis de mínimos cambios.

B. Glomerulonefritis segmentaria y focal.

C. Glomerulonefritis proliferativa focal.


Banco de preguntas grupo 11

D. Glomerulonefritis membranosa.

E. Glomerulonefritis membranoproliferativa.

ANSWER: C

¿Cúal enunciado no entra en el criterio diagnóstico de nefroesclerosis hipertensiva?

A. Mayores de 55 años de edad

B. Hipertensión arterial de larga evolución

C. Insuficiencia renal de evolución rápida

D. Retinopatía, hipertrofia ventricular izquierda

ANSWER: C

2.-Señale el enunciado correcto respecto a nefroescleorosis benigna

A. Arteriola con deposito hialino que afecta a todo el perímetro del vaso

B. Necrosis fibrinoide de las arteriolas aferentes

C. El diagnóstico está íntimamente ligado a los resultados del examen del fondo de ojo

D. Se detectan hemorragias y exudados, se habla de HTA acelerada.

ANSWER : A

3.- Patología relacionada al endurecimiento renal y es el resultado final de la sustitución del


parénquima renal normal por un tejido más denso con abundante componente colágeno producto de
la HTA crónica

A. Litiasis renal

B. Nefropatía hipertensiva

C. Hidronefrosis

D. Hipertensión renovascular

ANSWER: B

4.- Cuando existe aumento pasajero de 20mmHg en la presión sistólica que sucede en el consultorio
médico. Se denomina como:

A. HTA RESISTENTE

B. HTA SISTOLICA ASILADA

C. HTA DE BATA BLANCA


Banco de preguntas grupo 11

D. HTA ENMASCARADA

ANSWER: C

5.- Cuando existe una presión arterial ˃ 140/90 a pesar del uso 3 de medicamentos antihipertensivos a
dosis máxima e incluido un diurético. Se denomina:

A. HTA RESISTENTE

B. HTA SISTOLICA ASILADA

C. HTA DE BATA BLANCA

D. HTA ENMASCARADA

ANSWER: A

6.- Si existe un aumento de la PA marcado que provoca daño a órganos diana que requieren tto
inmediato. Se trata de:

A. HTA RESISTENTE

B. HTA SISTOLICA ASILADA

C. EMERGENCIA HIPERTENSIVA

D. HTA ENMASCARADA

ANSWER: C

7.- Cuando existe una PA normal en el consultorio, pero por medición ambulatoria de la PA se
encuentra elevada en más de 20 mmHg.

A. HTA RESISTENTE

B. HTA SISTOLICA ASILADA

C. EMERGENCIA HIPERTENSIVA

D. HTA ENMASCARADA

ANSWER: D

8.- Cual de las siguientes no es una causa secundaria de hipertensión arterial

A. QUISTES RENALES

B. UROPATIA OBSTRUCTIVA

C. MICOSIS

D. ALDOSTERONISMO PRIMARIO
Banco de preguntas grupo 11

ANSWER: C

9.- Características de los pacientes que deben suscitar la sospecha de HTA secundaria son las
siguientes, excepto:

A. PACIENTES JÓVENES MENORES A 40 AÑOS, CON HTA DE GRADO 2

B. CARACTERÍSTICAS CLÍNICAS DE APNEA OBSTRUCTIVA DEL SUEÑO

C. HTA RESISTENTE

D. EDEMA DE MIEMBROS SUPERIORES

ANSWER: D

10.- Cual de los siguientes métodos de diagnóstico no se usa para la hipertensión renovascular

A. RAYOS X

B. GAMMAGRAFIA CON CAPTOPRIL

C. ECO-DOPPLER

D. RM ANGIOGRAFICA

ANSWER: A

Cuáles son las Indicaciones orientativas del empleo de bloqueantes del SRAA en renoproteccion de
ERC:

A. ERC proteinurica (> 30 mg/24 hs)

B. ERC no proteinúrica si HTA o diabetes mellitus

C. ERC proteinurica (> 30 mg/24 hs)

D. A Y B

ANSWER: D

Cardio protectores de primera linea:

A. β bloqueantes

B. α bloqueantes

C. Los inhibidores de la renina

D. Todos

ANSWER: A
Banco de preguntas grupo 11

En la ERCA deben emplearse preferentemente:

A. Diuréticos tiazídicos

B. Diuréticos de asa

C. Diuréticos antialdosterónicos

D. Manitol

ANSWER: B

La asociación de alteraciones metabólicas más característica que podemos encontrar en presencia de


ERC en estadios avanzados es:

A. Hiperpotasemia, hipofosfatemia, hipercalcemia, acidosis metabólica

B. Hipopotasemia, hipofosfatemia, hipocalcemia, acidosis metabólica

C. Hiperpotasemia, hiperfosfatemia, hipocalcemia, acidosis metabólica

D. Hiperpotasemia, hiperfosfatemia, hipocalcemia, alcalosis metabólica

ANSWER:C

Cual es la causa más frecuente de ERC.

A. Diabetes mellitus

B. Hipertensión arterial

C. Glomerulonefritis

D. Riñón quístico

E. Infecciones urinarias de repetición

ANSWER: A

¿EN EL ANCIANO LA ERC SE DIAGNOSTICA POR?

e) CIFRAS NORMALES DE UREA

f) CIFRAS BAJAS DE UREA

g) CIFRAS BAJAS DE CRATINICA

h) CIFRAS ELEVADAS DE UREA Y CRATININA

ANSWER: D

¿CUÁNDO LA TFG ES MENOR O IGUAL A 15ML/MIN/1.73M2?


Banco de preguntas grupo 11

A. SE PRESENTA PROTEINURIA

B. SE PRODUCE LA UREMIA

C. PRODUCE HIPERTENCION

D. PRODUCE ORINA DILUIDA

ANSWER: B

LA ANEMIA EN LA ERC ES DE TIPO

e) NORMOCRONICA Y NORMOCITICA

f) HIPERCROMICA

g) SOLO NORMOCRONICA

h) SOLO NORMOCITICA

ANSWER: A

Global Outcomes )

Fecha actualización: 13/06/2020 00:00:00

Información del artículoTablas Imágenes Texto completo Bibliografía Descargar PDF

La ERC en el adulto se define como

A. La presencia de una alteración estructural o funcional renal (sedimento, imagen, histología) que
persiste más de 6 semnas, con o sin deterioro de la función renal; o un filtrado glomerular (FG) < 60
ml/min/1,73 m2 sin otros signos de enfermedad renal.

B. La presencia de una alteración estructural o funcional renal (sedimento, imagen, histología) que
persiste más de 3 meses, con o sin deterioro de la función renal; o un filtrado glomerular (FG) < 90
ml/min/1,73 m2 sin otros signos de enfermedad renal.

C. La presencia de una alteración estructural sin deterioro de la función renal; o un filtrado glomerular
(FG)  60 ml/min/1,73 m2 conn otros signos de enfermedad renal.

D. La presencia de una alteración estructural o funcional renal (sedimento, imagen, histología) que
persiste más de 3 meses, con o sin deterioro de la función renal; o un filtrado glomerular (FG) < 60
ml/min/1,73 m2 sin otros signos de enfermedad renal.

ANSWER:D

Cual no es marcadores de daño renal:


Banco de preguntas grupo 11

A. Proteinuria elevada

B. Alteraciones en el sedimento urinario

C. Alteraciones estructurales histológicas

D. Proteinuria Disminuida

E. ANSWER:D

Entre los principales objetivos del tratamiento de la insuficiencia renal aguda se encuentran?

A. La optimización de la hemodinámica, La corrección de desequilibrios hidroelectrolíticos.

B. La optimización de la hemodinámica, la disminución de la tasa de filtración.

C. La corrección de desequilibrios hidroelectrolíticos, la disminución de la tasa de filtración.

D. Todas.

ANSWER A

En La Insuficiencia Renal Aguda causada por esclerodermia que medicamentos se debe suministrar.

A. Inmunodepresores

B. plasmaféresis

C. Inhibidores de la ACE

D. AINES

ANSWER C

Al hablar del aporte nutricional en la insuficiencia renal aguda para prevenir las complicaciones, nos
referimos a?

A. Restriccion de proteinas a menos de 0.8 g/kg de peso en pacientes no hipercatabolicos.

B. Hidratos de carbono mínimo de 100 g /dia

C. Calorias entre 20 a 30 kcal /kg de peso

D. Todas

ANSWER D

Fisiopatológicamente la causa prerrenal de la Insuficiencia renal aguda corresponde a una perfusión


sanguínea inadecuada?

• Verdadero

• Falso
Banco de preguntas grupo 11

ANSWER verdadero

Fisiopatológicamente la causa postrenal de la insuficiencia renal aguda corresponde a una alteración


en la integridad del parénquima renal

• Verdadero

• Falso

ANSWER falso

Indique la respuesta correcta, los siguientes son procesos que afectan directamente el parénquima
renal en la fisiopatología de la insuficiencia renal aguda por causa renal, excepto

A. Lesión aguda de tubulos renales

B. Enfermedades tubulointersticiales

C. Enfermedades de grandes vasos del riñon

D. Disminución del gasto cardiaco

ANSWER D

¿Cuál de los siguientes es una contraindicación para el trasplante renal?:

A) Síndrome de Alport

B) Amiloidosis

C) Cistinosis

D) Todas las anteriores

E) Ninguna de las anteriores

ANSWER E

¿Cuál de los siguientes hallazgos NO es indicación de diálisis en pacientes con insuficiencia renal
aguda?:

A) Sobrecarga de volumen.

B) Hipercalemia severa.

C) Acidosis metabólica.
Banco de preguntas grupo 11

D) Hipocalcemia severa.

E) Pericarditis urémica.

ANSWER D

En el estudio inicial de un paciente con insuficiencia renal ¿qué prueba complementaria resultaría más
útil para diferenciar si es aguda o crónica?:

A) Hemograma.

B) Electromiograma.

C) Radiología ósea.

D) Sedimiento.

E) Ecografía renal.

ANSWER E

¿De cuál de los siguientes cuadros es característica la aparición de cilindros hemáticos en el sedimento
urinario?:

A) Necrosis tubular aguda.

B) Lesión a cualquier nivel de las vías urinarias.

C) Daño glomerular severo.

D) Daño tubular.

E) Cualquier lesión de la nefrona.

ANSWER C

La tasa de filtración glomerular aumenta cuando:

A) Aumenta la resistencia en la arteriola aferente glomerular.

B) Disminuye la resistencia en la arteriola eferente glomerular.

C) Aumenta la actividad de los nervios simpáticos renales.

D) Se produce obstrucción de la vía urinaria.

E) Disminuye la concentración de las proteínas plasmáticas.

ANSWER E
Banco de preguntas grupo 11

¿Qué es litiasis renal?

A. Cálculos en el aparato urinario

B. Cálculos en el hígado

C. Lesión de la glándula suprarrenal

D. Ninguna de las anteriores

ANSWER: A

¿A qué edad en hombres es más frecuente la litiasis renal?

A. 20-30 años

B. 30-40 años

C. 40-60 años

D. Ninguna de las anteriores

ANSWER: C

¿Cuál es el cálculo más frecuente en pacientes diabéticos?

A. Ac. Úrico

B. Calcio

C. Magnesio

D. Ninguna de las anteriores

ANSWER: A

Complete: El dolor provocado por un cálculo renal se acompaña de .............................

A. Fiebre, disfagia

B. Nauseas, fiebre

C. Náuseas, vómitos y fiebre

D. Disfagia, vómitos y fiebre

ANSWER: C

Complete: El dolor que provoca un cálculo localizado en el uréter inferior se irradia a:

A. La región lumbar

B. Al labio mayor
Banco de preguntas grupo 11

C. Al testículo ipsilateral

D. B Y C son correctas

ANSWER: D

Complete: Los cálculos renales de ................ se da por una infección por baterías ureasas positivas
(Proteus mirabilis):

A. Ácido úrico

B. Estruvita

C. Calcio

D. Cistina

ANSWER: B

¿Qué cálculos renales se observan radiolúcidos en la radiografía abdominal?

A. Ácido úrico

B. Estruvita

C. Calcio

D. Cistina

ANSWER: A

Señale, entre las siguientes, la afirmación correcta respecto a la sensibilidad de la ecografía en el


diagnóstico de la litiasis renal:

A. Es de alrededor del 0.95 para los cálculos vesicales.

B. Para los cálculos ureterales es mayor cuando se localizan en la porción sacroilíaca del uréter.

C. No varía con el tamaño de los cálculos.

D. Varía según la composición de los cálculos.

ANSWER: A

¿cuál es el tratamiento de elección de la hipercalciuria idiopática?:

A. Furosemida.

B. Restricción de calcio en lo dieta.

C. Hidroclorotiazida.

D. Calcitonina.
Banco de preguntas grupo 11

ANSWER: C

¿Cuál de las siguientes circunstancias es una contraindicación absoluta para la litotricia extracorpórea
con ondas de choque de la litiasis renal?:

A. Embarazo.

B. Coagulopatía.

C. Marcapasos cardiaco.

D. Aneurisma aórtico abdominal.

ANSWER: A
TEMA: SINDROME NEFRITICO

¿Señale lo correcto con respecto a las glomerulonefritis paucinmune tienen las


siguientes características?

A. C3 normal o elevado
B. C3 y C4 disminuido
C. .ANCA P y ANCA C negativos
D. C3 es normal o elevado y ANCA p y ANCA c positivos

ANSWER: D

¿Señale lo correcto con respecto a la enfermedad de Goodpasture?

A. ​glomerulonefritis, con o sin hemorragia pulmonar


B. anticuerpos circulantes ASTO
C. Factor reumatoide positivo
D. ANA positivo

ANSWER: A

¿Señale lo correcto con respecto al síndrome de Churg Strauss?

A. Vasculitis necrotizante sistémica


B. Afecta vasos sanguíneos de pequeño y mediano calibre
C. Usualmente se presenta entre los 14 y 75 años
D. Todas las anteriores son correctas

ANSWER: D

El síndrome nefrítico se caracteriza por las siguientes características clínicas,


EXCEPTO:

A. Cilindros hemáticos
B. Anuria
C. Proteinuria < 3.5 g/día
D. Aumento de urea y creatinina

ANSWER: B

¿Cuándo el C3 vuelve a sus valores normales en la glomerulonefritis


postestreptococica?

A. Al momento de la resolución del cuadro clínico


B. Después de una semana de la resolución del cuadro clínico
C. Hasta 6 meses después de la resolución del cuadro clínico
D. Hasta 1 año después de la resolución del cuadro clínico

ANSWER: C.

¿Cuál es la característica del edema en paciente con glomerulonefritis


postestrptococica?

A. Edema pretibial matutino


B. Edema periférico y periorbital
C. Edema circunscrito al área afectada
D. No hay edema en esta patología

ANSWER: B.

En qué estadío de la nefritis lúpica encontramos una proliferación endo o


extracapilar, menor al 50% de los glomérulos

A. Minina mesangial (I)


B. Proliferativa mesangial
C. Proliferativa focal
D. Proliferativa difusa

ANSWER: C.

A qué virus está asociada la crioglobulinemia

A. Adenovirus
B. Hepatitis A
C. Rinovirus
D. Hepatitis C

ANSWER: D.

El síndrome de Alport está relacionado con la siguiente clínica, EXCEPTO:

A. Esplenomegalia
B. Hematuria
C. Proteinuria
D. Sordera neurosensorial

ANSWER: A.

Qué enfermedad baja el C4 del complemento, pero mantiene normal el C3

A. Nefritis lúpica
B. Enfermedad de Berger
C. Crioglobulinemia
D. Todas

ANSWER: B.

TEMA: HTA primaria y secundaria, nefropatía hipertensiva

Según la clasificación de la JNC, a que estadio corresponde los siguientes parámetros de


presión arterial de 140-159 mmhg sistólica y 90-99 mmhg diastólica

A. ​Normal

B. ​Prehipertensión

C. Estadio 1
D. Estadio 2

ANSWER: C

A qué factores inductores de la hipertensión arterial corresponde los factores propios de la


madre, los relacionados con el individuo y los dependientes del medio

A. Factores genéticos
B. Factores ambientales
C. ​Factores inmunológicos

D. ​Ninguna

ANSWER: B

¿Cuánto tiempo se debe esperar para tomar la presión arterial en pacientes que acaban de
realizar ejercicio físico o han tomado café?

A. 30 minutos
B. ​20 minutos

C. 10 minutos
D. ​Inmediatamente

ANSWER: A

Según la American Heart Association ¿cuántas veces se debe tomar la presión en la


primera consulta?

A. Solo 1 vez y se diagnostica


B. Se deben tomar 5 veces en ambos brazos
C. Se deben hacer 2 mediciones que deben realizarse a intervalos de al menos 1
minuto
D. Ninguna

ANSWER: C

¿Cuándo se considera una emergencia hipertensiva?

A. PA mayor o igual a:180/120 mmHg + lesión aguda de órgano diana


B. PA mayor o igual a:180/120 mmHg sin lesión aguda de órgano diana
C. PA mayor o igual a:140/90 mmHg + lesión aguda de órgano diana
D. PA mayor o igual a:140/90 mmHg sin lesión aguda de órgano diana

ANSWER: A

Seleccione la opción INCORRECTA sobre coartación de la aorta.

A. HTA en miembros superiores


B. pulsos femorales disminuidos o retrasados
C. causa de HTA secundaria en adultos
D. causa de HTA secundaria en niños

ANSWER: C

¿Qué puede presentar un paciente con SAHS?

A. depresión y arritmias cardiacas.


B. cefalea, somnolencia diurna
C. fatiga, confusión matutina
D. todas son correctas

ANSWER: D

Seleccione la opciòn incorrecta de acuerdo a los criterios diagnòsticos de nefroesclerosis


hipertensiva:

A. sexo femenino
B. mayores de 55 años
C. hipertensiòn arterial de larga evoluciòn
D. riñones simètricos,normales o pequeños

ANSWER: A

La meta de cifras de presión arterial en el tratamiento de pacientes con nefroesclerosis


hipertensiva es:

A. 120/80
B. 90/60
C. menor a 140/90
D. mayor a 140/90

ANSWER: C

Cuàl NO es causa de hipertensiòn secundaria:

A. embarazo
B. enfermedades tiroideas
C. estenosis de la arteria renal
D. coartaciòn de la aorta
ANSWER: A

TEMA: NEFROPATÍA DIABÉTICA


En un pcte con una nefropatía avanzada cuál lesión es más frecuente y está en forma
nodular

A. Glomeruloesclerosis nodular
B. Glomeruloesclerosis difusa
C. Gota capsular
D. Depósitos extracapilares de fibrina

ANSWER: B

En qué tipo y estadio de DM se da incremento del volumen mesangial, con depósito de


proteínas y albúmina

A. Dm2 estadio 1 Hipertrofia renal


B. Dm1 estadio 3 Nefropatía incipiente
C. Dm2 estadio 3 Nefropatía incipiente
D. Dm1 estadio 2 Lesión renal sin signos clínicos

ANSWER: D

El estadio 5 de la DM2 que presenta

A. Cambios hemodinámicos de vasodilatación e hiperfiltración glomerular no siempre están


presentes
B. Presencia de proteinuria, detectable por métodos clínicos habituales -albuminuria superior a
200 µg/min o 300 mg/24 horas
C. La esclerosis glomerular, la fibrosis intersticial y la atrofia tubular se acompañan de un
descenso considerable del filtrado glomerular
D. Disminución de la fracción de filtración e histológicamente coincide con la esclerosis nodular
o difusa.

ANSWER: C

¿A partir de qué estadio de la nefropatía diabética existe macroalbuminuria?


A. Estadio 2
B. Estadio 3
C. Estadio 4
D. Estadio 5
ANSWER: C
¿En qué estadio de la nefropatía diabética existe microalbuminuria transitoria?
A. Estadio 2
B. Estadio 3
C. Estadio 4
D. Estadio 5
ANSWER: A
Escoja el literal incorrecto sobre los trastornos por Neuropatía diabética
A. Frialdad, livideces.
B. ​Pérdida de sensibilidad al dolor
C. ​Calor, enrojecimiento.
D. Atrofia y paresia muscular de las extremidades inferiores
ANSWER: A

De acuerdo al cociente de excreción Albúmina/Creatinina, cuando se sitúa en una


nefropatía incipiente?

A. Un cociente de excreción < 30 mg/g


B. Un cociente de excreción con dos pruebas positivas de > 300 mg/g
C. Un cociente de excreción > 30 mg/g < 300 mg/g
D. Un cociente de excreción con dos pruebas positivas de > 30 mg/g - < 300 mg/g
ANSWER: D

Respecto a la prevalencia de microalbuminuria y macroalbuminuria en nuestros pacientes


diabéticos. ¿Cuál de estas afirmaciones es cierta?

A. La detección precoz es poco importante ya que la capacidad de intervención es


limitada.
B. La prevalencia de microalbuminuria en la población general es menor del 3%
C. Se puede hallar microalbuminuria en el paciente diabético tipo 2 desde el momento
del diagnóstico de diabetes
D. En el paciente diabético tipo 1 no se presenta microalbuminuria
ANSWER: C

De los siguientes problemas. ¿Cual no es considerado como factor de riesgo para el inicio
y/o progresión de la nefropatía diabética en el diabético tipo 2?

A. Hipertensión arterial.
B. Sexo femenino.
C. Mal control glucémico.
D. Historia familiar de nefropatía diabética.

ANSWER: B

Qué cociente de Albúmina/Creatinina indica nefropatía establecida

A. Mayor a 30 mg/g y menor a 300 mg/g


B. Menor a 30 mg/g
C. Mayor a 300 mg/g
D. Menor a 3000 mg/g

ANSWER C

TEMA: SÍNDROME NEFRÓTICO


Se considera proteinuria en intervalo nefrótico a aquella superior a:
A. 4.5 g/24horas/1.73m
B. 2.5 g/24horas/1.73m
C. 3.5 g/24horas/1.73m
D. 4.0 g/24horas/1.73m

ANSWER: C

¿Cuál es la principal etiología del síndrome nefrótico biopsiado en adultos?

A. Nefropatía diabética
B. Nefropatía membranosa
C. Nefropatía de cambios mínimos
D. Glomeruloesclerosis focal

ANSWER: B

¿Cuál de los siguientes enunciados corresponde a un mecanismo por el que se produce la


disfunción podocitaria?

A. ​Daño directo podocitario por toxinas sistémicas o generadas localmente,


infecciones, anticuerpos, fármacos
B. ​Trastornos congénitos o adquiridos que constituyen la membrana gruesa
C. ​Alteraciones de las proteínas estructurales del endotelio fenestrado, como de
la α-actina 2
D. Alteraciones de las cargas positivas de los glucosaminoglucanos

ANSWER: A

¿Cuándo aparece la hipoalbuminemia en el síndrome nefrótico?

A. ​Cuando el hígado disminuye la síntesis de albúmina hasta en un 300% por


mecanismos transcripcionales
B. ​Por el aumento en la presión oncótica del plasma y concentración sérica de
albúmina
C. Cuando la proteinuria y el catabolismo renal de la albúmina filtrada superan
la capacidad de síntesis hepática
D. Cuando el catabolismo de la albúmina está aumentado y por el contrario la
síntesis de albúmina disminuida

ANSWER: C

¿Cuál no es una característica del edema en síndrome nefrótico?

A. ​Con fóvea
B. ​Esblando
C. No deja fóvea
D. Se acumula en zonas declive y en regiones con presión tisular pequeña
ANSWER: C

Cuando la trombosis de la vena renal es crónica asintomática observamos:

A. Embolia
B. Curso insidioso
C. Edema en miembros inferiores
D. Todas son correctas

ANSWER: D

¿Cuál de los siguientes factores no ocasiona insuficiencia renal aguda en pacientes con
síndrome nefrótico?

A. Hipovolemia
B. ​Edema intrarrenal

C. Administración de IECAS
D. ​Ninguna es correcta

ANSWER: D

Una Complicación característica del síndrome nefrótico es:

A. Pericarditis urémica.
B. Hipercoagulabilidad sanguínea con elevada incidencia de trombosis.
C. Hipotiroidismo por disminución de la globulina de tiroxina.
D. Edema pulmonar

ANSWER: C

¿Cuál de los siguientes fármacos produce relativa frecuencia de necrosis tubular aguda?

A. Diuréticos
B. Antibióticos aminoglucosidos
C. Antitiroideos
D. Antidiabeticos orales

ANSWER: B

Señale lo correcto con respecto a las alteraciones endocrinas en el síndrome nefrótico:

A. hipotransferrinemia
B. aumento de 17-hidroxicorticosteroides
C. Eritropoyetina aumentada hasta un 5%.
D. hipertiroidimos clínico

ANSWER: A

TEMA: ITU
De los siguientes enunciados mencione cual no es compatible con los exámenes
complementarios utilizados en ITU:
A. Sedimento Urinario
B. Esterasa leucocitaria
C. Nitritos
D. Todas son correctas
ANSWER: D

Cual es el microorganismo que se aísla con más frecuencia en un urocultivo por ITU:
A. Eschericha Coli
B. Proteus
C. Acinetobacter
D. Ninguna de las anteriores
ANSWER: A

En el varón, el diagnóstico diferencial de ITU debe incluir


A. Uretritis
B. Prostatitis
C. Cistitis
D. Todas las anteriores
ANSWER: D

En la cistitis y pielonefritis enfisematosa el diagnóstico se lo realiza por:


A. EGO
B. Biometría hemática
C. TC abdominal.
D. Ninguna de las anteriores
ANSWER: C

¿Cuál es la clínica que se presenta en la cistitis?


A. Disuria, polaquiuria y micción urgente
B. Polaquiuria, caídas al suelo y cefalea
C. Fiebre elevada, escalofríos y afectación del estado general
D. Dolor abdominal, confusión y disminución de la movilidad
ANSWER: A

¿Cuál es el agente infeccioso que provoca nefrolitiasis?


A. Echerichia coli
B. Klebsiella pneumoniae
C. Proteus mirabilias
D. Corynebacterium uralyticum
ANSWER: D

¿Cuál es el tratamiento de la pielonefritis no complicada?


A. B-lactámicos o fosfomicina por 7 días
B. Amoxicilina-Clavulánico 500 mg/12h
C. Cefixima o ceftidoreno 200-400 mg/12h
D. Nitrofurantoína 100mg/12h
ANSWER: C
La mayoría de los episodios de ITU no complicada está producida por

A. Klebsiella pneumoniae
B. Staphylococcus saprophyticus
C. Proteus mirabilis
D. Escherichia coli
ANSWER: D

Describa situaciones que sugieren una posible ITU complicada

A. Modificaciones de las vías urinarias, como un asa ileal o un reservorio


B. Presencia de una sonda uretral permanente, una endoprótesis o una sonda de
nefrostomía
C. Sondaje vesical intermitente
D. Todas las anteriores

Answer: D

¿Cuáles son los factores de riesgo de la infección del tracto urinario no complicada?

A. Antecedentes de ITU, relaciones sexuales y usos de espermicidas


B. Antecedente de IAU en un familiar en primer grado, mujeres postmenopaúsicas y
obstrucción del tracto urinario
C. Mujeres con anomalías subyacentes del tracto urinario, cateterismo urinario
D. Diabetes mellitus, inmunodeficiencia

ANSWER: A

TEMA: NEFROPROTECCIÓN

En cuál de los siguientes casos no se precisa de un ajuste en la dosificación de la insulina.

A. Si la filtración glomerular es mayor a 40 ml/min


B. Si la filtración glomerular es mayor a 60 ml/min
C. Si la filtración glomerular es menor a 60 ml/min
D. Si la filtración glomerular es menor a 20 ml/min

ANSWER: B

En cuál de las siguientes situaciones los analgésicos NO llegan a causar daño renal.

A. Hipovolemia
B. Deshidratación
C. Insuficiencia cardíaca
D. Hipertensión arterial
ANSWER: D

¿Cuál de los siguientes analgésicos opioides es más seguro y no requiere ajuste de sus
dosis?

A. Fentanilo
B. Codeína
C. Tramadol
D. Morfina

ANSWER: A

¿Cuál de los siguientes medios de contraste entra en el grupo de los no iónicos o de baja
osmolaridad?

A. Iotalamato sódico
B. Meglumine
C. Diatrizoato
D. Gadoteridol

ANSWER: D

Cual es incorrecta respecto a recomendaciones para prescripción de medicamentos en


ERC

A. Jóvenes
B. Diabetes
C. Insuficiencia cardiaca
D. Estenosis de arteria renal

ANSWER: A

Se hace ajuste de dosis en ERC cuando:

A. FG debajo a 70 ml/min/1,73 m2
B. FG mayor a 60 ml/min/1,73 m2
C. FG debajo de 60 ml/min/1,73 m2
D. FG 65 a 70 ml/min/1,73 m2

ANSWER: C

La metformina a que porcentaje se reduce cuando el FG es 30 – 45 ml/min/1,73 m2:

A. No se utiliza
B. Reducir 50%
C. Es seguro usar dosis completa
D. Suspender administración

ANSWER: B

Cuando se limita el empleo de sulfonilureas

A. FG < 45 ml/min/1,73 m2
B. FG < 60 ml/min/1,73 m2
C. FG < 50 ml/min/1,73 m2
D. FG < 15 ml/min/1,73 m2

ANSWER: A

Cuál de las Gliptinas no requiere ajuste de dosis cuando el FG es < 50 ml/min/1,73 m2

A. Linagliptina
B. Sitagliptina
C. Vildagliptina
D. Saxagliptina

ANSWER: A

Cuando debe interrumpirse el tratamiento con dapagliflozina

A. si la FR cae por debajo de un FG de 70 ml/min/1,73 m2


B. si la FR cae por debajo de un FG de 15 ml/min/1,73 m2
C. si la FR cae por debajo de un FG de 40 ml/min/1,73 m2
D. si la FR cae por debajo de un FG de 60 ml/min/1,73 m2

ANSWER: D

TEMA: SÍNDROME CARDIORRENAL

Escoja el enunciado correcto sobre el síndrome cardiorrenal

A. Síndrome cardiorrenal es solo cuando los trastornos cardíacos crónicos


descompensados causan daño renal agudo.
B. La coexistencia de enfermedad cardiaca y renal se ha relacionado con mejor
pronóstico a medio y largo plazo ya que inducen compensación significativa.
C. Serie de “desórdenes del corazón y riñón en los que la disfunción aguda o
crónica en un órgano induce la disfunción aguda o crónica del otro”
D. Solo cuando un infarto de miocardio puede causar daño renal agudo

ANSWER: C

Cuantos tipos de síndromes cardiorrenal existen:

A. 2, agudo y crónico
B. 5 tipos
C. 3 agudo, subagudo y crónico
D. 4 tipos.

ANSWER: B

Los eventos cardíacos agudos que se han relacionado con el desarrollo de lesión
renal aguda para que se dé un Síndrome cardiorrenal tipo 1, se incluye excepto:

A. La insuficiencia cardiaca aguda


B. El síndrome coronario agudo
C. El shock cardiogénico.
D. Sepsis
E. Alteraciones asociadas a la cirugía cardiaca.

ANSWER: D

El síndrome cardiorrenal tipo 2 se caracteriza por un deterioro crónico de la función


renal como consecuencia de un deterioro crónico de la función cardiaca, esto se
evidencia en:

A. Insuficiencia cardiaca crónica


B. Diabetes
C. Amiloidosis
D. Uso de medios de contrastes
E. Infarto de miocardio

ANSWER: A

¿Cuáles son las características clínicas de un síndrome cardiorrenal tipo I?

A. Lesión renal aguda que provoca insuficiencia cardiaca aguda


B. Insuficiencia cardiaca crónica que lleva a insuficiencia renal crónica
C. Insuficiencia cardiaca aguda que causa lesión renal aguda.
D. Lesión renal y cardiaca aguda secundaria a enfermedad sistémica

ANSWER: C

En el síndrome cardiorrenal tipo II el empeoramiento de la función renal en el


contexto de una insuficiencia cardiaca crónica se ha asociado con:

A. Un pronóstico favorable y una hospitalización prolongada


B. Un pronóstico favorable y una hospitalización corta
C. Un pronóstico adverso y una hospitalización corta
D. Un pronóstico adverso y una hospitalización prolongada

ANSWER: D

¿Qué fármacos se deben evitar en paciente con síndrome cardiorrenal tipo I por
causar fallo renal agudo?

A. IECA
B. Diuréticos tiazídicos
C. Diuréticos de Asa
D. Betabloqueantes

ANSWER: A

TEMA: SÍNDROME HEPATORENAL

El SHR tipo 1 tiene un tiempo de evolución de:

A. Menor a 2 semanas
B. 4 semanas
C. 6 meses
D. 1 año

ANSWER​: A

El SHR tipo 2 tiene un tiempo de evolución de:

A. Menor a dos semanas


B. 3 meses
C. 4 a 6 meses
D. 6 a 12 meses

ANSWER​: C

El evento inicial de la patogenia del SHR es:

A. Vasoconstricción Renal
B. Vasodilatación Renal
C. Vasodilatación Esplácnica
D. Hipertensión Portal

ANSWER​: D

Señale lo correcto con respecto al SHR tipo I

A. Creatinemia alcanza valores superiores a 2,5 mg/dl


B. Oliguria progresiva
C. Hiponatremia dilucional
D. Todas son correctas

ANSWER: D

A menudo el SHR tipo I puede estar precedido por factores precipitantes ¿Cuáles
son?

A. PBE
B. Hemorragias Digestivas
C. Paracentesis
D. Todas las anteriores

ANSWER: D

¿Cuál es el cuadro clínico predominante del SHR tipo II?

A. Moderado aumento de nitrógeno ureico en sangre


B. Función hepática relativamente preservada
C. Ascitis refractaria al tratamiento diurético
D. Creatinina plasmática > 2,5 mg/dl

ANSWER: C

Son criterios de diagnóstico adicionales del SHR excepto:

A. Ausencia de Shock
B. Volumen urinario menos de 500 ml/día
C. Sodio urinario menos de 10 mEq/l
D. Sodio sérico menos de 130 mEq/l

ANSWER: A

El tratamiento definitivo para el SHR es:

A. Vasoconstrictores y albúmina.
B. Hemodiálisis.
C. Trasplante hepático
D. Ninguno de los anteriores.

ANSWER: C

Las indicaciones para hemodiálisis son:

A. Sobrecarga de volumen.
B. Acidosis metabólica.
C. Hipercalemia
D. Todas las anteriores.

ANSWER: D

En pacientes con hepatitis alcohólica grave el fármaco para prevenir SHR es:

A. Pentoxifilina
B. Norfloxacino.
C. Albúmina
D. Terlipresina.

ANSWER: A

TEMA: IRA

Según el tiempo de evolución, a que se define como injuria renal aguda

A. Daño renal mayor a 90 días


B. Daño renal menor a 7 días
C. Daño renal menor a 24h
D. Daño renal > 7 y < 90 días

ANSWER: B

Los túbulos renales más afectados en la NTA son:

A. túbulo colector
B. asa de henle y segmento s3 del T.proximal
C. túbulo distal
D. macula densa

ANSWER: B

¿En qué fase de la NTA hay regeneración gradual del epitelio tubular?

A. Fase de recuperación
B. Fase de mantenimiento
C. Fase de inicio
D. Fase de extensión

ANSWER: A

La Hipertrofia prostática benigna es una causa común de:

A. IRA renal
B. IRA parenquimatosa
C. IRA prerrenal
D. IRA posrenal

ANSWER: D

Todos son biomarcadores de IRA, excepto:


A. KIM1
B. N-GAL
C. CISTATINA C
D. IL-6

ANSWER: D

¿Cuál de los siguientes estudios imagenológicos es de preferencia para el estudio


de la oclusión de la arteria renal?

A. Tomografía computarizada
B. Radiografía
C. Eco-Doppler
D. Urografía intravenosa

ANSWER: C

Un EFNa mayor a 2, un IFR mayor a 2,5, un osmolaridad menor a 350 y un sodio


urinario mayor a 40, me sugiere el fracaso renal es por causa de:

A. NTA
B. FRA obstructiva
C. FRA prerrenal
D. NTIA

ANSWER: A

Que función tiene el manitol y la furosemida

A. Aumenta el flujo intratubular, minimizando la obstrucción de los túbulos.


B. Disminuye el flujo intratubular, minimizando la obstrucción de los túbulos.
C. Aumenta el flujo intratubular, aumentando la obstrucción de los túbulos.
D. Ninguna de las anteriores
ANSWER: A

Qué efecto podría ejercer el manitol

A. Disminuyendo el flujo intratubular


B. Un cierto efecto antioxidante, disminuyendo el edema celular que precede a la
necrosis
C. Aumenta la obstrucción tubular y efecto antioxidante
D. Efecto antioxidante, aumentando el edema celular y hay necrosis celular
ANSWER: B

Que efecto tiene la furosemida


A. tiene efecto vasodilatador, y disminuye los requerimientos energéticos de células
tubulares
B. Tiene un efecto vasoconstrictor y aumenta los requerimientos de las células
tubulares.
C. Efecto antioxidante y aumenta la obstrucción tubular aumentando el edema celular
D. Ninguna de las anteriores
ANSWER: A

TEMA LITIASIS Y UROPATÍA OBSTRUCTIVA

Son sustancias inhibidoras de la formación de litos renales, excepto

A. Citrato
B. Nefrocalcina
C. Uropontina
D. Sustancia A

ANSWER: D

El lito puede pasar a vejiga cuando su tamaño es:

A. <5 mm
B. <5cm
C. <7mm
D. <7cm

ANSWER: A

Signos radiográficos de la ureterohidronefrosis crónica:

A. Visualización tardía con un pielograma muy contrastado por la sumación del


contraste.
B. Renomegalia con reducción del grosor del parénquima renal.
C. Cálices en masa con dilatación simultánea de los infundíbulos del área piélica.
D. Todas las anteriores.

ANSWER: D

Señale el enunciado correcto.

La hiperuricosuria se caracteriza por:


A. Excreción inferior a 800 mg/día en el hombre y 700 mg/día en mujeres.
B. Excreción de 400 mg/día en ambos sexos.
C. Excreción superior de 800 mg/día en el hombre y 750 mg/día en mujeres.
D. ​Ninguna de las anteriores.

ANSWER: C

¿Cual de las siguientes no es causa de hipocitraturia?

A. Acidosis tubular renal


B. Ingesta elevada de proteínas
C. ​Hipermagnesemia

D. ​Inhibidores de la anhidrasa carbónica

ANSWER: C

¿Que fármacos pueden desencadenar una urolitiasis medicamentosa?

A. Triamtereno
B. Acetazolamida
C. Indinavir
D. Todos

ANSWER: D

¿En qué situaciones es conveniente hacer uso de la Litotricia por onda de choque?

A. Cálculos ureterales impactados


B. Cálculos de cistina
C. Cálculos Renales y Ureterales no impactados
D. Pacientes obesos

ANSWER: C

¿En qué situaciones es conveniente hacer uso de la Litotricia por ultrasonido?

A. Cálculos de cistina
B. Pacientes obesos
C. Cálculos Renales y Ureterales no impactados
D. Cálculos ureterales impactados

ANSWER​:​ B

Tratamiento para litiasis acompañada de hiperoxaluria

A. Dieta basada en oxalato


B. Administración de vitamina C
C. Dieta reducida en calcio
D. En pacientes requeridos, administración suplementaria de calcio 1g/día

ANSWER: D

Para un diagnóstico correcto de una U.O en TUI (Tracto urinario inferior) debemos tomar en
cuenta:

A. Características de glándula prostática


B. Globo vesical (Palpación y percusión)
C. Fimosis
D. Todas son correctas

ANSWER: D

PREGUNTAS DE RIÑON Y EMBARAZO

Definición de Hipertensión Arterial Crónica. Señale la correcta.

A. HTA previa al embarazo o que se descubre antes de la semana 20 y persiste


12 semanas después del parto.
B. ​HTA que aparece después de la semana 20 no se acompaña de proteinuria.
C. HTA previa al embarazo o que se descubre antes de la semana 20 con
aparición de proteinuria.
D. HTA que aparece después de la semana 20 con aparición de proteinuria.

ANSWER: A

Definición de Hipertensión Gestacional. Señale la correcta.

A. H
​ TA previa al embarazo o que se descubre antes de la semana 20 y persiste

12 semanas después del parto.


B. HTA que aparece después de la semana 20 no se acompaña de proteinuria.
C. HTA previa al embarazo o que se descubre antes de la semana 20 con
aparición de proteinuria.
D. ​HTA que aparece después de la semana 20 con aparición de proteinuria.

ANSWER: B

Definición de Hipertensión Crónica complicada con preeclampsia. Señale la


correcta.

A. HTA previa al embarazo o que se descubre antes de la semana 20 y persiste


12 semanas después del parto.
B. ​HTA que aparece después de la semana 20 no se acompaña de proteinuria.
C. HTA previa al embarazo o que se descubre antes de la semana 20 con
aparición de proteinuria.
D. HTA que aparece después de la semana 20 con aparición de proteinuria.

ANSWER: C

Cuál de los siguientes fármacos antihipertensivos son compatibles en la lactancia.


Señale la correcta.

A. Atenolol
B. Metildopa
C. Nadolol
D. ​Diliazem.

ANSWER: B

¿Cuántos centímetros aumenta el riñón durante el embarazo?

A. ​2 cm
B. ​1 cm
C. 5 cm
D. 4 cm

ANSWER: B

¿Cual de proceso hipertensivo NO se presenta en el embarazo?

A. Preeclampsia – Eclampsia
B. HTA gestacional
C. HTA Crónica
D. HTA secundaria

ANSWER: D

Cuál enunciado corresponde a Preeclampsia

A. HTA que se presenta después de la semana 20 de gestación y presenta


proteinuria + 0.3g en 24 horas
B. HTA previa al embarazo o que se descubre antes de la semana 20 y dura
hasta 12 semanas después del parto
C. HTA que aparece después de la semana 20 y que no presenta proteinuria ni
otras manifestaciones orgánicas
D. HTA que se presenta después de la semana 20 de gestación, presenta
proteinuria + 0.3g en 24 horas y convulsiones

ANSWER: A
Durante el embarazo qué medicamentos NO están contraindicados

A. Ciclofosfamida
B. ​Metotrexato
C. Azatioprina
D. ​Mofetilmicofenolato

ANSWER: C

Señale lo INCORRECTO en el Embarazo tras trasplante renal

A. Mejora la función reproductora


B. ​La fertilidad se recobra a partir de los 2 años
C. ​La concepción tiene mayor probabilidad de éxito
D. La HTA favorece los abortos espontáneos

ANSWER: B

Señale cuál de las siguientes opciones NO es una condición ideal de embarazo en


mujeres con trasplante renal

A. Tener una creatinina estable menor a 2 mg sobre dl


B. Proteinuria mínima menor a 0.5g en 24H
C. ​Presión arterial normal o controlada
D. ​Esperar al menos una año del trasplante para poder embarazarse

ANSWER: A

TEMA: ENFERMEDAD RENAL CRÓNICA.


Cuanto es el porcentaje de filtrado glomerular en el estadio 4 según la clasificación
de los estadios de ERC.

A. De 15 a 29
B. De 45 a 59
C. Menos de 15
D. Mayor a 90

ANSWER: A

Todos son factores predisponentes para ERC excepto:

A. Edad > 60 años


B. Diabetes
C. Raza Blanca
D. Bajo peso al nacer

ANSWER: C

Cuáles son los primeros síntomas que aparecen en los estadios iniciales de la ERC.

A. Polidipsia y polifagia
B. Poliuria y nicturia
C. HTA y miocardiopatías
D. Proteinuria y Cefalea

ANSWER: B

Todos son factores de progresión de ERC excepto:

A. HTA y diabetes mal controlada


B. Dislipidemia
C. Obesidad
D. Consumo de drogas

ANSWER: D

Para saber si se está frente a un cuadro de insuficiencia renal aguda es necesario


que haya:

A. Proteinuria de inicio súbito


B. Disminución de la función renal con filtrado glomerular menor a 60mL/min/
1.73 m2 durante al menos 3 meses, con daño renal o sin él.
C. Disminución del filtrado glomerular menos de 60 mL/min/1,73 m2 al menos 45
días
D. Todas las anteriores

ANSWER: B

¿Cuánto es el porcentaje de filtrado glomerular en el estadio 3ª según la


clasificación de KDIGO en la ERC?

A. Mayor a 90
B. 30 a 44
C. 45 a 59
D. 60 a 89

ANSWER: C

¿Cuál es la vía que causa daño renal en lípidos?

A. la formación de lipoproteínas oxidadas


B. la formación de óxido nitroso
C. la formación de glucosa
D. Todas las anteriores

ANSWER: A

Según la clasificación KDOQI la ERC en función del filtrado glomerular; el estadio 5


está definido por:

A. Un filtrado glomerular < 15 mL/min/1.73m2


B. Un filtrado glomerular < 29 – 15 mL/min/1.73m2
C. Un filtrado glomerular < 90 mL/min/1.73m2
D. Un filtrado glomerular <5 mL/min/1.73m2

ANSWER: A

Se define a la ERC como el descenso del filtrado glomerular por debajo de

A. 45 mL/min/1.73m2
B. 50 mL/min/1.73m2
C. 60 mL/min/1.73m2
D. 75 mL/min/1.73m2

ANSWER: C

Señale lo incorrecto: Sobre los factores de riesgo de ERC

A. Diabetes mellitus
B. Glomerulonefritis
C. Raza negra
D. Sexo Femenino

ANSWER: D

TEMA: POLIQUISTOSIS RENAL

Los procesos fundamentales para el desarrollo y el crecimiento de los quistes incluyen:

A. pérdida de la polaridad celular plana

B. aumento en las tasas de proliferación y de apoptosis celular

C. secreción de líquido en el lumen

D. todas las anteriores

ANSWER: D

¿Cuál es la etiología de esta enfermedad?


A. Enfermedad genéticamente heterogénea

B. Enfermedad autosómica

C. Enfermedad de origen idiopático

D. Origen Vírica

ANSWER: A

¿Cómo se originan los quistes?

A. Como dilataciones focales de las asas renales

B. Como consecuencia enfermedades hormonales

C. Como dilataciones focales de los túbulos renales

D. Ninguna

ANSWER: C

¿Que desempeña un papel central en la cistogénesis?

A. Mecanosensación y cilio primario.

B. Cilio primario

C. Vías Wnt y PCP

D. Mecanosensación.

ANSWER: A

Alrededor de qué porcentaje de pacientes con poliquistosis renal autosómica dominante


desarrolla cálculos renales

A. 20

B. 30

C. 40

D. 50

ANSWER: A

¿Cuál es la manifestación extrarrenal más frecuente?


A. Poliquistosis renal autosómica dominante

B. Poliquistosis renal autosómica recesiva

C. Poliquistosis hepática

D. Poliquistosis renal congénita.

ANSWER: C

Cual no es un gen identificado que afecta en la poliquistosis hepática autosómica dominante


(PQHAD)

A. PRKCHS

B. SEC63

C. PKD1

D. PKD63

ANSWER: C

En el caso de riñones poliquísticos en estadio terminal asociado a fiebre persistente después


de dos semanas de terapia antimicrobiana adecuada ¿qué conducta sería la correcta a seguir?

A. Drenaje percutáneo.

B. Drenaje quirúrgico.

C. Nefrectomía.

D. Embolización.

ANSWER: C

¿Cuál es la característica ecográfica principal de la poliquistosis renal autosómica recesiva?

A.​ R
​ iñones grandes con disminución de la ecogenicidad en la corteza y médula.

B.​ R
​ iñones grandes con aumento de la ecogenicidad en la corteza y médula.

C.​ R
​ iñones grandes con aumento de la ecogenicidad en la médula.

D.​ R
​ iñones grandes con disminución de la ecogenicidad en la médula.

ANSWER: B
¿Cuáles son las manifestaciones más graves dentro de la poliquistosis renal autosómica
recesiva?

A. Oligohidramnios y Síndrome de Potter.

B. Polihidramnios e hipoplasia pulmonar.

C. Oligohidramnios.

D. Síndrome de Potter.

ANSWER: A

TEMA: TERAPIA RENAL SUSTITUTIVA

La terapia de sustitución renal es el reemplazo de la función del riñón en pacientes


con:
A. Insuficiencia renal
B. Hipertensión arterial
C. Tumor de Wilms
D. Edema en esclavina
E. Ninguna de las anteriores

RESPUESTA: A

¿Cuáles son las técnicas utilizadas en la terapia renal sustitutiva?


A. Hemofiltración y hemodiálisis continuas
B. Hemodiálisis intermitente
C. Diálisis peritoneal
D. Ninguna de las anteriores
E. Todas las anteriores

RESPUESTA: E

¿Qué anomalía endocrina NO se corrige al realizar la terapia de sustitución renal?


A. Insuficiencia Renal
B. Desintoxicación
C. Disminución en la producción de eritropoyetina
D. Tumor de Wilms
E. Ninguna de las anteriores

RESPUESTA: C

¿Qué sucede durante la filtración?


A. Los solutos del suero, difunden en forma pasiva entre los compartimentos líquidos,
siguiendo un gradiente de concentración
B. El agua del suero pasa de un compartimento a otro siguiendo un gradiente de
presión hidrostática, arrastrando solutos con ella
C. Los solutos del suero pasa de un compartimento a otro siguiendo un gradiente de
presión hidrostática
D. Todas son correctas

RESPUESTA: B

¿Qué es la hemoperfusión?
A. Técnica poco usada que remueve las toxinas mediante el pasaje de la sangre sobre
un lecho de material adsorbente
B. Técnica de elección para pacientes que requieren Terapia renal sustitutiva
C. Técnica que necesitan un circuito directo arteriovenoso o venovenoso
D. Ninguna de las anteriores
E. Todas son correctas

RESPUESTA: A

¿El término terapia de reemplazo renal continua se refiere a?


A. Las terapias que purifican la sangre en forma extracorpórea, sustituyendo la función
renal en forma continua durante las 24 horas del día.
B. Las terapias que purifican la sangre en forma intracorpórea, sustituyendo la función
renal en forma continua durante las 24 horas del día.
C. Las terapias que purifican la sangre en forma intracorpórea, sustituyendo la función
renal en forma continua durante las 18 horas del día.
D. Las terapias que purifican la sangre en forma extracorpórea, sustituyendo la función
renal en forma continua durante las 18 horas del día.
E. Las terapias que purifican la sangre en forma intracorpórea, sustituyendo la función
renal en forma continua durante las 12 horas del día.

Answer: A

La terapia de reemplazo renal continua tiene algunas modalidades que son, excepto:
A. Ultrafiltración lenta continua
B. Hemofiltración venovenosa continua
C. Hemodiálisis venovenosa continua
D. Hemodiafiltración venovenosa continua
E. Ultrafiltración rapida continua

Answer: E

Las Indicaciones de terapia de reemplazo renal continua son:


A. Acidosis grave
B. Uremia severa
C. Hipervolemia
D. Hipercalcemia y otros electrolitos
E. Todas.
Answer: E

Cual de las siguientes opciones es incorrecta con respecto a las alternativas de


anticoagulantes en la terapia renal sustitutivas?
A. Administrar heparina no fraccionada a dosis de 40 UI/kg en bolo seguida
B. Administrar enoxaparina a dosis de carga de 0.15 mg/kg e infusión de 0.05 mg/kg/h
C. Administrar citrato y protaglandinas.
D. Administrar Daltaperina

Answer: D

Cuales son los criterios para los pacientes que no requieren terapia de
anticoagulación?
A. Plaquetas < 70,000
B. TPA > 65 seg
C. Coagulación intravascular diseminada
D. Hemorragia espontánea mayor
E. Toda las anteriores

Answer: E
Las alteraciones cardiovasculares son la causa más frecuente de morbimortalidad en
pacientes con ERC, e incluyen. Señale la incorrecta.
Hipertensión arterial se debe fundamentalmente a la sobrecarga de volumen secundaria a la
retención de sodio y agua.
La hipertrofia ventricular izquierda se encuentra en el 65%-75% de los pacientes con ERC
avanzada.
Insuficiencia cardíaca congestiva suele deberse a sobrecarga de volumen y HTA.
Pericarditis, suele correlacionarse con valores bajos de BUN generalmente 120mg/dl​.
¿Qué medicamento antidiabético está contraindicada en pacientes con estadio 4 de ERC
por el riesgo de inducir acidosis metabólica?
Glibenclamida
Gliquidona
Metformina
Glipizida
En la clasificación de la enfermedad renal crónica, a partir de cual estadio se diagnostica al
paciente como enfermo renal crónico, sin necesidad de la presencia de otros signos de
daño renal.
G1 en adelante
G2 en adelante
G3a en adelante
G3b en adelante
Cuál de los siguientes es considerado dentro de los criterios clínicos y analíticos para el
diagnóstico de Hipertensión renovascular
Hiperpotasemia
Proteinuria nefrótica
Buena respuesta al tratamiento antihipertensivo habitual
Edad inferior a 30 años en la mujer o mayor de 60 en el varón
Acidosis hipercaliémica
Mencione el factor fisiológico más común en la hipertensión arterial
Factor genético
Disfunción endotelial
Alteración de la matriz extracelular
Remodelado vascular alterado
Tono vascular normal
Dentro de los objetivos del manejo de la hipertensión arterial seleccione la correcta:
Mantener PA objetiva en 1 mes, si no se logra
Aumentar la dosis de primer fármaco u otro antihipertensivo, si no se logra
Aumentar la dosis del segundo fármaco más otro antihipertensivo
No usar IECA y ARA II combinados
Todas las anteriores
En la IRA de acuerdo a su clasificación funcional en el sistema rifle que determina la letra R
Incremento de crs X1,5 su valor basal descenso del filtrado glomerular > 25% y
diuresis < a 0.5 ml/kg/hora en 6 horas
Incremento de crs X 3 o > a 4 mg/dl descenso del filtrado glomerular > 75% y diuresis < a
0.5 ml/kg/hora x 24 horas o anuria de 12 horas
IRA persistente = pérdida funcional renal > 4 semanas
Enfermedad renal terminal = Diálisis > 3 meses
Lesión ureteral: Señale la INCORRECTA
Bolas Fúngicas
Tumores próximos
Carcinoma vesical
Necrosis papilar
Según la etiología de la IRA cual es la que se debe a trastornos que condicionan una
obstrucción al paso de la orina en cualquier zona del tracto urinario
IRA prerrenal
IRA intrínseca
IRA posrenal
Ninguna de las anteriores
Seleccione la opción falsa según la regla de las mitades en la hipertensión:
Por cada 800 adultos mayores 400 eran hipertensos.
De 400 hipertensos sólo 200 eran diagnosticados con hipertensión arterial.
De 400 hipertensos 200 tenían antecedentes familiares de hipertensión.
De 200 hipertensos sólo 100 han comenzado un tratamiento.
Seleccione la opción falsa según la regla de las mitades en la hipertensión:
Por cada 800 adultos mayores 400 eran hipertensos.
De 400 hipertensos sólo 200 eran diagnosticados con hipertensión arterial.
De 200 hipertensos sólo 100 han comenzado un tratamiento.
De 100 hipertensos sólo 50 alcanzan la meta de la presión sanguínea.
Seleccione la opción falsa según la regla de las mitades en la hipertensión:
Por cada 800 adultos mayores 400 eran hipertensos.
De 400 hipertensos sólo 200 eran diagnosticados con hipertensión arterial.
De 200 hipertensos sólo 100 han comenzado un tratamiento.
De 100 hipertensos sólo 50 alcanzan la meta de la presión sanguínea.
Seleccione la opción falsa según la regla de las mitades en la hipertensión:
De 400 hipertensos sólo 200 eran diagnosticados con hipertensión arterial.
De 200 hipertensos sólo 100 han comenzado un tratamiento.
De 50 hipertensos sólo 25 alcanzan la meta de la presión sanguínea.
De 50 hipertensos sólo 25 están usando el fármaco correcto.
La regla de las mitades en la hipertensión nos indica que, por cada 800 adultos en la
comunidad, 400 eran hipertensos, de ellos sólo 200 eran diagnosticados con hipertensión
arterial, de ellos solo 100 han comenzado un tratamiento, de ellos sólo 50 están usando el
fármaco correcto y de ellos sólo 25 alcanzan la meta de la presión sanguínea ¿Es
Verdadero o falsa esta proposición?
Verdadero.
Falso.
La regla de las mitades en la hipertensión nos indica que, por cada 800 adultos en la
comunidad, 400 eran hipertensos, de ellos sólo 200 eran diagnosticados con hipertensión
arterial, de ellos solo 100 han comenzado un tratamiento, de ellos sólo 50 alcanzan la meta
de la presión sanguínea ¿Es Verdadero o falsa esta proposición?
Verdadero.
¿Cuáles son los factores de riesgo que predisponen a la preeclampsia – eclampsia?
Trastorno hipertensivo en embarazo anterior (incluyendo preeclampsia)
Enfermedad renal crónica
Diabetes mellitus tipo 1 y 2
Todas las anteriores
¿Cuáles son las medidas no farmacológicas para el tratamiento de los trastornos
hipertensivos del embarazo?
No se recomienda la restricción de sodio por debajo de la ingesta diaria.
No se recomienda la reducción de peso durante el embarazo para prevenir la preeclampsia.
Se recomienda controlar el peso en cada control prenatal o control por algún trastorno
hipertensivo del embarazo, para determinar cambios significativos que alteren el manejo de
la paciente.
Todas las anteriores
¿Cuáles son las medidas farmacológicas para el tratamiento de los trastornos hipertensivos
del embarazo?
El uso de diuréticos y el de expansores del volumen plasmático no han demostrado mejoras
clínicas ni en la madre ni en el feto.
La evidencia más actual indica que la nifedipina oral y el labetalol intravenoso son igual de
efectivos y seguros en el manejo emergente de la hipertensión severa en el embarazo.
El riesgo de hipertensión grave se reduce a la mitad con el uso de antihipertensivos. Sin
embargo, los antihipertensivos no se asocian con cambios en el riesgo de preeclampsia,
muerte fetal-neonatal, parto prematuro o neonato pequeño para la edad gestacional.
Todas las anteriores
¿En el síndrome hepatorenal es frecuente el desarrollo de cirrosis con ascitis?
Verdadero
Falso
Las manifestaciones clínicas del síndrome hepatorrenal tipo 1 son, EXCEPTO:
Deterioro rápido y progresivo de la función renal.
Disfunción multiorgánica (insuficiencia: hepática grave, cerebral y suprarrenal relativa).
Casi siempre presenta hiponatremia.
La presión arterial suele ser baja.
Hipoalbuminemia e hiponatremia moderadas o marcadas.
¿Cuáles son algunas de las complicaciones relacionadas a la derivación portosistémica
intrahepática transyugular?
Hemorragias intraabdominales, arritmias cardíacas, migración y trombosis de derivación.
Anemia hemolítica, fiebre, infecciones.
Reacciones a los medios de radiocontraste (incluida la nefrotoxicidad).
Todas las anteriores.
Para establecer la sospecha ante un diagnóstico de síndrome nefrótico ¿en que valores se
debe encontrar la proteinuria del paciente?
Adultos: >2.5g/24h/1.73m2 Niños: 40g/24h/1.73m2
Adultos: >3.5g/24h/1.73m2 Niños: 40mg/24h/1.73m2
Adultos: >1.5g/24h/1.73m2 Niños: 40mg/24h/1.73m2
Adultos: >3.5g/24h/1.73m2 Niños: 20mg/24h/1.73m2
Ninguna de las anteriores
Característicamente el edema del síndrome nefrótico es:
Edema no intersticial con fóvea
Edema de carácter postural
Ninguna de las anteriores
Como examen complementario Gold estándar en el síndrome nefrótico es la recolección de
orina en 24 horas?
Falso
Verdadero
De acuerdo, a la American Diabetes Asocciation. ¿Cuál es el objetivo de PA en el
tratamiento de pacientes diabéticos?
En toda diabetes mellitus una presión arterial <130/80 mmHg
En toda diabetes mellitus una presión arterial <1450/80 mmHg
En toda diabetes mellitus una presión arterial <130/100 mmHg
En toda diabetes mellitus una presión arterial <140/90 mmHg
El aumento inadecuado de albuminuria, citocinas y factores de inflamación por un tiempo
prolongado lleva como resultado:
Hipertensión glomerular
Hiperfiltración glomerular
Fibrosis renal
Hipertrofia renal
Las siguientes opciones son características de una proteinuria franca, excepto:
Deterioro progresivo e irreversible de la función renal
Representa la fase avanzada de la ERCD
En pacientes diabéticos puede desarrollar sx nefrótico
Se considera proteinuria franca a la microalbuminuria
En relación a la etiología extracapilar del síndrome nefrítico, seleccione la opción
INCORRECTA
GN proliferativa postinfecciosa
GN con semilunas
Glomerulonefritis rápidamente progresiva
Asociados a enfermedades sistémicas o autoinmunes
Todas son correctas
EN EL SÍNDROME NEFRÍTICO, LA DISMINUCIÓN DEL FILTRADO GLOMERULAR VA A
PRODUCIR:
SELECCIONE UNA:
Oligoanuria.
Hematuria.
Todas son correctas.
Proteinuria.
Ninguna es correcta.
¿CUÁL NO ES CARACTERÍSTICA DEL SÍNDROME NEFRÍTICO?
SELECCIONE UNA:
Edema.
Hematuria.
Retención hidrosalina.
Hiperlipidemia.
Hipertensión.
¿En qué ocasiones se restringe el potasio en un paciente con síndrome nefrítico?
Oligoanuria
Hematuria
Edema
Disnea
Ninguna de las anteriores
De las siguientes opciones cuál es la correcta relacionado al síndrome cardiorrenal tipo 1
Insuficiencia cardiaca aguda que lleva a una LRA​.
Insuficiencia cardiaca crónica que lleva a una LRA
LRA que lleva a insuficiencia cardiaca aguda.
Enfermedad renal crónica que lleva a la insuficiencia cardiaca crónica.
Procesos sistémicos que resultan en insuficiencia cardiaca e insuficiencia renal.
¿Cuál es la causa más frecuente del síndrome cardiorrenal tipo 5?
lupus eritematoso sistémico
diabetes mellitus
sepsis
artritis reumatoide
Seleccione la respuesta correcta. A qué tipo de SCR corresponde la siguiente descripción:
Insuficiencia cardiaca crónica que lleva a la falla renal crónica.
SCR uno
SCR dos
SCR tres
SCR cuatro
SCR cinco
Seleccione la opción correcta sobre la 3era fase en la obstrucción urinaria de presentación
aguda
Se produce dilatación intrapiélica e intracanalicular para disminuir la presión ureteral
La presión ureteral aumenta entre 50-70mmHg
El flujo renal se encuentra aumentado por vasodilatación pre glomerular
En esta fase no puede producirse rotura calicial
No se presenta flujo retrogrado pielovenoso o pielolinfático
16/3/2021 EXAMEN SEGUNDO PARCIAL: Revisión del intento

Área personal / Mis cursos / NEFROLOGÍA / BLOQUE DE CIERRE 2 / EXAMEN SEGUNDO PARCIAL

Comenzado el martes, 16 de marzo de 2021, 13:33


Estado Finalizado
Finalizado en martes, 16 de marzo de 2021, 14:29
Tiempo 55 minutos 52 segundos
empleado

Pregunta 1
Finalizado

Puntúa como 0,25

En la necrosis tubular aguda por nefrotoxicidad se dice que es el efecto del fármaco sobre el:

a. Epitelio tubular renal


b. Ninguna de las anteriores
c. Intersticio renal
d. Todas las anteriores

Pregunta 2
Finalizado

Puntúa como 0,25

¿Qué duración tiene un paciente con Síndrome hepatorrenal tipo I?

a. 2 meses
b. 1 mes
c. 5 años
d. 6 meses - 1 año

Pregunta 3
Finalizado

Puntúa como 0,25

El síndrome de Alport está relacionado con la siguiente clínica, EXCEPTO:

a. Sordera neurosensorial
b. Esplenomegalia
c. Hematuria
d. Proteinuria

https://campusvirtual.ug.edu.ec/mod/quiz/review.php?attempt=1177339&cmid=1122654 1/11
16/3/2021 EXAMEN SEGUNDO PARCIAL: Revisión del intento

Pregunta 4
Finalizado

Puntúa como 0,25

9.- Características de los pacientes que deben suscitar la sospecha de HTA secundaria son las siguientes, excepto:

a. EDEMA DE MIEMBROS SUPERIORES


b. PACIENTES JÓVENES MENORES A 40 AÑOS, CON HTA DE GRADO 2
c. CARACTERÍSTICAS CLÍNICAS DE APNEA OBSTRUCTIVA DEL SUEÑO
d. HTA RESISTENTE

Pregunta 5
Finalizado

Puntúa como 0,25

En la endocarditis bacteriana aguda, están presente los siguientes depósitos inmunitarios, excepto…

a. IgM
b. Fracción C3 del complemento
c. IgG
d. IgA

Pregunta 6
Finalizado

Puntúa como 0,25

Mujer de 50 años, diabética en tratamiento con insulina, con infecciones urinarias y cólicos nefríticos de repetición. Acude a Urgencias por
dolor en fosa renal izquierda de 5 días de evolución, asociado en las últimas 24 horas a fiebre, escalofríos y malestar general. Analítica de
sangre: pla-quetopenia, leucocitosis y disminución de la actividad de la protrombina. Analítica de orina normal. Radiografía de abdomen con
claras imágenes de litiasis. Ecografía renal: dilatación moderada de sistema excretor izquierdo. ¿Cuál es la conducta más adecuada?

a. Realizar urografía intravenosa para intentar filiar la causa.


b. Colocar catéter doble J o practicar nefrostomía percutánea de forma in-mediata con cobertura antibiótica.
c. Hidratar a la paciente bajo observación rigurosa, y repetir ecografía a las 48 horas.
d. Solicitar hemocultivos y urocultivo para establecer la necesidad de an-tibioterapia.

Pregunta 7
Finalizado

Puntúa como 0,25

Según el tiempo de evolución, a que se define como injuria renal aguda

a. Daño renal mayor a 90 días


b. Daño renal menor a 7 días
c. Daño renal menor a 24h
d. Daño renal mayor 7 y menor 90 días

https://campusvirtual.ug.edu.ec/mod/quiz/review.php?attempt=1177339&cmid=1122654 2/11
16/3/2021 EXAMEN SEGUNDO PARCIAL: Revisión del intento

Pregunta 8
Finalizado

Puntúa como 0,25

4.- Cuando existe aumento pasajero de 20mmHg en la presión sistólica que sucede en el consultorio médico. Se denomina como:

a. HTA ENMASCARADA
b. HTA DE BATA BLANCA
c. HTA SISTOLICA ASILADA
d. HTA RESISTENTE

Pregunta 9
Finalizado

Puntúa como 0,25

¿Como subdividió el club internacional de ascitis los criterios diagnósticos?

a. Solo es de 1 tipo: severidad de instauración


b. Solo es de 1 tipo: de acuerdo a su velocidad
c. 2 tipos: de acuerdo a su velocidad y severidad de instauración
d. 3 tipos: de acuerdo a su velocidad, severidad de instauración y edad

Pregunta 10
Finalizado

Puntúa como 0,25

¿El síndrome cardiorrenal tipo 1 a que hace referencia?

a. Disfunción cardiaca aguda induce lesión o disfunción renal aguda.


b. Enfermedad cardiovascular que acompaña a la ERC.
c. Deterioro crónico de la función renal como consecuencia de un deterioro crónico de la función cardiaca.
d. Lesión concomitante renal y cardiaca aguda o crónica secundaria a una enfermedad sistémica aguda o crónica.

Pregunta 11
Finalizado

Puntúa como 0,25

Los tipos de hemodiálisis se clasifican en los siguientes excepto

a. de alta eficacia
b. de alto flujo
c. tradicional
d. convencional

https://campusvirtual.ug.edu.ec/mod/quiz/review.php?attempt=1177339&cmid=1122654 3/11
16/3/2021 EXAMEN SEGUNDO PARCIAL: Revisión del intento

Pregunta 12
Finalizado

Puntúa como 0,25

¿Cúal enunciado no entra en el criterio diagnóstico de nefroesclerosis hipertensiva?

a. Mayores de 55 años de edad


b. Insuficiencia renal de evolución rápida
c. Retinopatía, hipertrofia ventricular izquierda
d. Hipertensión arterial de larga evolución

Pregunta 13
Finalizado

Puntúa como 0,25

¿CUÁL es el gen y la proteína afectados en la PQRAR?

a. HNF-1b - TCF2
b. NPHP1 - Nefrocistina
c. No es una enfermedad genética
d. PKHD1 Fibrocistina
e. PKD 1/PKD 2 - Policistina 1/ Policistina 2

Pregunta 14
Finalizado

Puntúa como 0,25

En el 25% de los casos de síndrome de HELLP puede encontrarse:

a. Trastornos de visión y audición


b. Hiponatremia
c. Epigastralgia
d. Hipertensión

Pregunta 15
Finalizado

Puntúa como 0,25

La Hipertrofia prostática benigna es una causa común de:

a. IRA posrenal
b. IRA prerrenal
c. IRA parenquimatosa
d. IRA renal

https://campusvirtual.ug.edu.ec/mod/quiz/review.php?attempt=1177339&cmid=1122654 4/11
16/3/2021 EXAMEN SEGUNDO PARCIAL: Revisión del intento

Pregunta 16
Finalizado

Puntúa como 0,25

¿A partir de qué estadio de la nefropatía diabética existe macroalbuminuria?

a. Estadio 2
b. Estadio 5
c. Estadio 3
d. Estadio 4

Pregunta 17
Finalizado

Puntúa como 0,25

Señale la incorrecta respecto a la hidronefrosis del embarazo:

a. Se cree que es debido a la peristalsis disminuida por la progesterona


b. Se resuelve en la primera semana después del parto.
c. Es más prominente del lado derecho
d. Se puede visualizar en el segundo trimestre del embarazo

Pregunta 18
Finalizado

Puntúa como 0,25

En ausencia de proteinuria, la preeclampsia también se diagnostica si las mujeres embarazadas tienen hipertensión de reciente comienzo
junto con aparición de cualquiera de los siguientes, excepto:

a. Deterioro de la función hepática (transaminasas &gt; 2 veces el valor normal)


b. Sin síntomas cerebrales o visuales.
c. Trombocitopenia (plaquetas menor 100.000/microL)
d. Insuficiencia renal (creatinina sérica &gt; 1,1 mg/dL o duplicación de la creatinina en suero en mujeres sin enfermedad renal)

Pregunta 19
Finalizado

Puntúa como 0,25

Los siguientes son criterios de AMSTERDAM de aceptación de donante vivo, excepto:

a. Albuminuria mayor 300 mg/24h


b. Ausencia de antecedente de DM
c. Filtrado glomerular mayor 80ml/min/1.73m2
d. IMC menor 35 kg/m2

https://campusvirtual.ug.edu.ec/mod/quiz/review.php?attempt=1177339&cmid=1122654 5/11
16/3/2021 EXAMEN SEGUNDO PARCIAL: Revisión del intento

Pregunta 20
Finalizado

Puntúa como 0,25

¿Cuál es el tratamiento farmacológico para el Hepatorrenal?

a. DIALISIS
b. Vasoconstrictores y albumina
c. Trasplante hepático
d. Derivacion percutánea portosistémica intrahepática

Pregunta 21
Finalizado

Puntúa como 0,25

Paciente de 15 años con diagnóstico de apendicitis aguda desarrolla una peritonitis perforativa, por lo cual desarrolla hipotensión y grave
alteración del estado general. ¿Qué tipo de síndrome cardiorrenal podría desarrollar este paciente principalmente?

a. Síndrome cardiorrenal tipo 3


b. Síndrome cardiorrenal tipo 1
c. Síndrome cardiorrenal tipo 2
d. Síndrome cardiorrenal tipo 5

Pregunta 22
Finalizado

Puntúa como 0,25

La nefritis túbulo-intersticial por nefrotoxicidad es causada por fármacos como Ciclosporina, Litio, AINES:

a. Verdadero
b. Falso
c. Es producida solo por Litio y Ciclosporina
d. Es producida solo por AINES.

Pregunta 23
Finalizado

Puntúa como 0,25

Cuando existe un aumento en el diámetro de los poros de la membrana basal en el síndrome nefrítico que manifestaciones presenta en la
orina.

a. ninguna es correcta
b. hematuria- anuria – proteinuria
c. hematuria-proteinuria cilindros de GR
d. hematuria- oliguria- hiperproteinuria

https://campusvirtual.ug.edu.ec/mod/quiz/review.php?attempt=1177339&cmid=1122654 6/11
16/3/2021 EXAMEN SEGUNDO PARCIAL: Revisión del intento

Pregunta 24
Finalizado

Puntúa como 0,25

Señale. En el Síndrome Hepatorrenal. ¿Cuál es el primer mecanismo vasoconstrictor que se activa después de fallar el aumento del gasto
cardíaco en el paciente con cirrosis?

a. Hipersecreción de vasopresina
b. Sistema nervioso simpático
c. Aumento del gasto cardíaco
d. Sistema Renina – Angiotensina - Aldosterona

Pregunta 25
Finalizado

Puntúa como 0,25

Varón de 31 años, sin antecedentes médicos que acude a Urgencias por oliguria. Presenta PA 160/100 mmHg y edemas en MMII. En la
analítica de sangre se observa Cr 3 mg/dl y U 120 mg/dl. En la analítica de orina existe hematuria de 50 hematíes/campo y proteinuria con
índice prot/Cr de 1 g/g. ¿Qué síndrome clínico presenta este paciente?

a. Síndrome nefrótico.
b. Enfermedad renal crónica.
c. Síndrome nefrítico.
d. Hematuria aislada.

Pregunta 26
Finalizado

Puntúa como 0,25

Hombre de 49 años que consulta por edemas en miembros inferiores de 3 semanas de evolución. En la analítica sanguínea presenta
creatinina 1.3 mg/dL, colesterol total 270 mg/dL y albúmina 2.4 g/dL. En el sedimento de orina presenta 15-20 hematíes por campo y en
orina de 24 horas se detecta proteinuria 3.7 g/día. ¿Cuál es el diagnóstico más probable?

a. Nefropatía membranosa.
b. Glomerulonefritis proliferativa mesangial.
c. Glomeruloesclerosis focal y segmentaria.
d. Glomerulonefritis membranoproliferativa tipo I.
e. Enfermedad de cambios mínimos.

Pregunta 27
Finalizado

Puntúa como 0,25

¿En el manejo del paciente diabético tipo 2 con nefropatía incipiente (microalbuminuria) cuál de las siguientes afirmaciones es cierta?

a. En el diabético tipo 2 con microalbuminuria el objetivo de control glucémico debe ser HbA1c &lt;8,5%.
b. Con el uso de IECAs a dosis bajas no es preciso hacer un control precoz de creatinina y potasio séricos.
c. Hay que evitar el uso de tiazidas en el diabético tipo 2 con microalbuminuria.
d. El abandono del tabaco probablemente influye favorablemente sobre la nefropatía del diabético tipo 2.

https://campusvirtual.ug.edu.ec/mod/quiz/review.php?attempt=1177339&cmid=1122654 7/11
16/3/2021 EXAMEN SEGUNDO PARCIAL: Revisión del intento

Pregunta 28
Finalizado

Puntúa como 0,25

Todas las nefropatías primitivas enumeradas a continuación se manifiestan típicamente como síndrome nefrótico, EXCEPTO una. Señálela:

a. Glomerulonefritis proliferativa focal.


b. Glomerulonefritis membranoproliferativa.
c. Glomerulonefritis segmentaria y focal.
d. Glomerulonefritis de mínimos cambios.
e. Glomerulonefritis membranosa.

Pregunta 29
Finalizado

Puntúa como 0,25

Son sustancias inhibidoras de la formación de litos renales, excepto

a. Sustancia A
b. Nefrocalcina
c. Uropontina
d. Citrato

Pregunta 30
Finalizado

Puntúa como 0,25

¿Cuál de las siguientes afirmaciones es correcta respecto a los criterios de diabetes?

a. Glucemia mayor igual a 100mg/dl dos horas tras la SOG con 75 g de glucosa
b. Paciente sintomático con glucemia de 100 mg/dl
c. Glucemia basal mayor igual a 126 mg/dl
d. HbA1C mayor igual a 65%

Pregunta 31
Finalizado

Puntúa como 0,25

¿En el manejo del diabético tipo 2 con insuficiencia renal moderada (creatinina sérica mayor 2 mg/dl) cuál de las siguientes afirmaciones es
falsa?

a. Las necesidades de insulina suelen disminuir porque, aunque se incrementa la resistencia insulínica predomina la acumulación
insulínica por el fallo renal.
b. Es bastante frecuente que la causa de la insuficiencia renal no sea la nefropatía diabética específica.
c. Habitualmente se requiere tratamiento farmacológico combinado para el control estricto de la PA.
d. Es más importante el uso, en sí mismo, de un nefroprotector como IECAs que el control estricto de la PA.

https://campusvirtual.ug.edu.ec/mod/quiz/review.php?attempt=1177339&cmid=1122654 8/11
16/3/2021 EXAMEN SEGUNDO PARCIAL: Revisión del intento

Pregunta 32
Finalizado

Puntúa como 0,25

¿Cuál de las siguientes circunstancias es una contraindicación absoluta para la litotricia extracorpórea con ondas de choque de la litiasis
renal?:

a. Embarazo.
b. Marcapasos cardiaco.
c. Coagulopatía.
d. Aneurisma aórtico abdominal.

Pregunta 33
Finalizado

Puntúa como 0,25

El Sindrome hepatorrenal tipo 1 tiene un tiempo de evolución de:

a. 4 semanas
b. Menor a 2 semanas
c. 1 año
d. 6 meses

Pregunta 34
Finalizado

Puntúa como 0,25

El tratamiento de mantenimiento con IECA Aumenta el aclaramiento de insulina y el FSR, a la vez que disminuye la proteinuria:

a. Falso
b. Verdadero
c. Disminuye el FSR y la proteinuria.
d. Aumenta el aclaramiento de insulina y disminuye el FSR

Pregunta 35
Finalizado

Puntúa como 0,25

¿Cuál de los siguientes medicamentos son considerados de primera elección para la nefroprotección en ERC?

a. IECA y B bloqueantes.
b. IECA y ARA II
c. ARA II y Diureticos.
d. Diuréticos y Antagonistas de calcio.

https://campusvirtual.ug.edu.ec/mod/quiz/review.php?attempt=1177339&cmid=1122654 9/11
16/3/2021 EXAMEN SEGUNDO PARCIAL: Revisión del intento

Pregunta 36
Finalizado

Puntúa como 0,25

En un paciente con síndrome nefrótico, las medidas generales no específicas, para corregir la proteinuria incluye uno de los siguientes
procedimientos:

a. Dieta hiperproteica.
b. Inhibidores del enzima conversor de la Angiotensina (IECAs).
c. Diuréticos y/o b-bloqueantes.
d. Corticoides por vía sistémica.
e. Calcioantagonistas no dihidropiridínicos.

Pregunta 37
Finalizado

Puntúa como 0,25

En el varón, el diagnóstico diferencial de ITU debe incluir

a. Todas las anteriores


b. Uretritis
c. Prostatitis
d. Cistitis

Pregunta 38
Finalizado

Puntúa como 0,25

Las Glomerulonefritis asociadas a ANCA – C son:

a. Granulomatosis de Wegener y Poliangeitis microscópica


b. Granulomatosis de Wegener y Sx Goodpasture
c. Granulomatosis de Wegener
d. Granulomatosis de Wegener y Sx Churg Strauss

Pregunta 39
Finalizado

Puntúa como 0,25

En la ERCA deben emplearse preferentemente:

a. Diuréticos antialdosterónicos
b. Diuréticos de asa
c. Diuréticos tiazídicos
d. Manitol

https://campusvirtual.ug.edu.ec/mod/quiz/review.php?attempt=1177339&cmid=1122654 10/11
16/3/2021 EXAMEN SEGUNDO PARCIAL: Revisión del intento

Pregunta 40
Finalizado

Puntúa como 0,25

Cuál de los siguientes no es hallazgo del síndrome nefrítico

a. Hematuria
b. Lesión renal aguda
c. Proteinuria &gt;3,5 g/día
d. Edema

◄ TERAPIA RENAL SUSTITUTIVA.

Ir a...

HISTORIA CLINICA ►

https://campusvirtual.ug.edu.ec/mod/quiz/review.php?attempt=1177339&cmid=1122654 11/11
Dom. Cien., ISSN: 2477-8818
Vol. 5, núm. 3, julio, 2019, pp. 434-442

Síndrome nefrótico a revisión de un caso

DOI: 10.23857/dc.v5i3.944
Ciencias de la salud
Artículo de investigación

Síndrome nefrótico a revisión de un caso

Nephrotic syndrome on a case review

Síndrome nefrótica em uma revisão de caso

Melania del Consuelo Alcívar-García I


dramelania@hotmail.com

Recibido: 28 de marzo de 2019 *Aceptado: 17 de junio de 2019 * Publicado: 05 de julio de 2019

I.
Magíster en Gerencia Clínica en Salud Sexual y Reproductiva, Magíster en Gerencia Educativa,
Doctor en Medicina y Cirugía, Docente de la Universidad Técnica de Manabí, Portoviejo,
Ecuador.

http://dominiodelasciencias.com/ojs/index.php/es/index
Dom. Cien., ISSN: 2477-8818
Vol. 5, núm. 3, julio, 2019, pp. 434-442
Síndrome nefrótico a revisión de un caso

El Artículo Forma Parte de los Resultados de un Trabajo de Investigación Institucional del


Octavo Semestre Paralelo “B” de la Facultad Ciencias de la Salud de la Universidad Técnica
de Manabí, Portoviejo, Ecuador.

1 Basurto Arteaga Josseline Irene jb1997-94@hotmail.com


2 Bernardi Zambrano Gianfranco giancoberzam@gmail.com
Valentino
3 Briones Zambrano Carlos Andrés carlozbrionesz@gmail.com
4 Campaña Arteaga Nikole Lucia nilu_ca@hotmail.com
5 Castro Zambrano Ana María anitamaria1208@hotmail.com
6 Casanova Ferrin Andrés Miguel andres-casanova@live.com
7 Cedeño Salazar Anthony Edmundo anthony4story@gmail.com
8 Cusme López Andrea Stefania andy10loc@hotmail.es
9 Intriago Rivera Nixon Ignacio nixon.intriago1997@gmail.com
10 Laaz Salazar Rosa Amelia rositalaaz@gmail.com
11 Licoa Navarro Yosenka Jaritza yosenka96@gmail.com
12 Lizano Macías María Isabel ilizano678@gmail.com
13 Loor Mendoza Paula Valeria loorpaulita@gmail.com
14 Loor Pico Iván Andrés ivanloor023@gmail.com
15 Macías Aránea Natasha Lilibeth natymaciasaranea@gmail.com
16 Maila Balderramo Ariana Margarita ariana15mb@hotmail.com
17 Mendoza García Bianka Isabel bianisabel12@hotmail.com
18 Montes Guerrero Lenin Alexander alexandermontes_96@hotmail.com
19 Palacios Arteaga Juan Carlos jcarlos19971@gmail.com
20 Pazmiño Moya Shaaron Magaly shaaronpaz22@gmail.com
21 Ponce Mero Nicole Stefanía stefaniapm1997@hotmail.com
22 Quiroga Sabando Andrea Isabel isa20011939@gmail.com
23 Rezabala Leones Gema Guadalupe grezabala0032@gmail.com
24 Salas Zambrano Olmedo Enmanuel olmedoenmanuel@gmail.com
25 Saldarriaga Mera Gabriel Eduardo gabrielsaldarriaga15@gmail.com
26 Sánchez Flores Jazmín Alexandra jaz155@hotmail.com
27 Sornoza Saltos Elías Hernán hernan1000@live.com
28 Vélez Intriago Anggie Yamilet anggievi28@gmail.com
29 Zambrano Mendoza José Luis josezambrano95@hotmail.com
30 Zambrano Mera Jordy Gabriel jzammer2011@hotmail.com

435 Vol. 5, núm. 3, julio, 2019, pp.434-442


Melania Consuelo Alcívar García,
Trabajo de Investigación Institucional del Octavo Semestre Paralelo “B” de la Facultad Ciencias de
la Salud de la Universidad Técnica de Manabí, Portoviejo, Ecuador.
Dom. Cien., ISSN: 2477-8818
Vol. 5, núm. 3, julio, 2019, pp. 434-442
Síndrome nefrótico a revisión de un caso

Resumen

Se describe el caso clínico de una paciente de 14 años de edad del sexo femenino que llega a la
casa de salud por presentar cuadro clínico de aproximadamente 1 hora de evolución caracterizado
por nauseas, mareos, vértigo, dificultad para la movilización de miembros inferiores y pérdida de
la conciencia de forma súbita de aproximadamente 3 minutos ; se le realizan exámenes de
laboratorio en la cual se evidencia cierto grado de compromiso renal por medio de bioquímica
sanguínea en la que se revela una elevación de la urea y la creatinina, así como una pérdida de
proteínas a expensas de la albumina y disminución de la globulina.

Palabras clave: Síndrome; nefrótico; urea; creatinina; albumina.

Abstract
The clinical case of a 14-year-old female patient who arrives at the health home is described by
presenting a clinical picture of approximately 1 hour of evolution characterized by nausea,
dizziness, vertigo, difficulty in mobilization of lower limbs and loss of consciousness suddenly of
approximately 3 minutes; Laboratory tests are performed in which a certain degree of renal
involvement is evidenced by means of blood biochemistry in which an elevation of urea and
creatinine is revealed, as well as a loss of proteins at the expense of albumin and decreased globulin.

Key words: Syndrome; nephrotic; urea; creatinine; albumin


Resumo
O caso clínico de uma paciente de 14 anos que chega ao domicílio é descrito por apresentar um
quadro clínico de aproximadamente 1 hora de evolução caracterizado por náusea, tontura,
vertigem, dificuldade de mobilização de membros inferiores e perda de consciência de repente de
aproximadamente 3 minutos; São realizados exames laboratoriais nos quais se evidencia certo grau
de acometimento renal por meio da bioquímica sangüínea, na qual se revela elevação de uréia e
creatinina, além de perda de proteínas à custa da albumina e diminuição globulina

Palavras-chave: Síndrome; nefrótico; uréia; creatinina; albumina

436 Vol. 5, núm. 3, julio, 2019, pp.434-442


Melania Consuelo Alcívar García,
Trabajo de Investigación Institucional del Octavo Semestre Paralelo “B” de la Facultad Ciencias de
la Salud de la Universidad Técnica de Manabí, Portoviejo, Ecuador.
Dom. Cien., ISSN: 2477-8818
Vol. 5, núm. 3, julio, 2019, pp. 434-442
Síndrome nefrótico a revisión de un caso

Introducción
El síndrome nefrótico (SN) es una de las patologías nefrológicas más frecuentes en pediatría, y se
caracteriza por edema generalizado, oliguria, proteinuria masiva, hipoalbuminemia e
hipercolesterolemia. La incidencia comunicada en población anglosajona es de 1-7 casos por 100
mil habitantes menores de 16 años, siendo mayor en poblaciones asiáticas y afroamericanas y en
países latinoamericanos como Colombia y Ecuador se ha comunicado una cifra de 2-7 casos por
100 mil menores de 18 años. (Nedier Pedraza G., Ceballos M L, Cano SCH F . 2008),( Castaño
I, Bolaños L 2005).

Cuando no está relacionado con enfermedades sistémicas, se le denomina síndrome nefrótico


primario (SNP) o idiopático, en contraste con el secundario, en el cual estas manifestaciones
ocurren en el transcurso de algunas otras enfermedades. Entre estas se pueden citar las
enfermedades autoinmunitarias, p. ej., el lupus eritematoso sistémico (LES), la enfermedad mixta
del tejido conectivo; otras enfermedades sistémicas como la púrpura de Schölein-Henoch, la
amiloidosis; nefropatías como la enfermedad por depósito de IgA o de Berger; algunas
enfermedades infecciosas como la glomerulonefritis postinfecciosa o postestreptocócica, sífilis,
lepra, endocarditis infecciosa, infecciones virales, o puede estar asociado con la exposición a ciertas
drogas (antiinflamatorios no esteroideos, penicilamina, sales de oro) o toxinas, como las que dejan
las picaduras de abejas. El SNP corresponde a un 90 % a 95 % de los casos diagnosticados en
niños, según diferentes reportes, mientras que en adultos la proporción de SN secundario es
sustancialmente mayor. (Noguera Valverde, R A, Madrigal Campos, G, Carranza Portocarrero, A.
2007).

El 80 % de los casos de síndrome nefrótico en niños presenta una lesión mínima en la biopsia renal
y buena respuesta a esteroides e inmunosupresores, aunque se reportan diferencias étnicas en
cuanto al tipo de lesión y a la respuesta terapéutica.

437 Vol. 5, núm. 3, julio, 2019, pp.434-442


Melania Consuelo Alcívar García,
Trabajo de Investigación Institucional del Octavo Semestre Paralelo “B” de la Facultad Ciencias de
la Salud de la Universidad Técnica de Manabí, Portoviejo, Ecuador.
Dom. Cien., ISSN: 2477-8818
Vol. 5, núm. 3, julio, 2019, pp. 434-442
Síndrome nefrótico a revisión de un caso

Comunicamos el caso de una paciente que presento cuadro clínico de aproximadamente 1 hora de
evolución caracterizado por nauseas, mareos, vértigo, dificultad para la movilización de miembros
inferiores y pérdida de la conciencia de forma súbita de aproximadamente 3 minutos.

Caso Clínico

Paciente del sexo femenino de 14 años de edad, que llega al área de emergencia del Hospital Verdi
Cevallos Balda presentando sintomatología de nauseas, mareos, vértigo, y dificultad para
movilización de miembros inferiores, familiar refiere pérdida de la conciencia de forma súbita de
aproximadamente 3 minutos. Con antecedente personales de epilepsia hace un año, tratada con
carbamazepina, y antecedentes familiares de epilepsia en su abuela materna y hermano.

Al interrogatorio se encuentra orientada en tiempo y espacio, con un Glasgow de 15/15 colabora a


la exploración.

El examen físico se constata:

Reflejo pupilar: isocóricas, reactivas a la luz

Presión arterial: 105/60 mmhg

Frecuencia cardíaca: 84 lpm

Frecuencia respiratoria: 22 rpm

En el estado General del paciente se encontró como datos relevantes una facie pálida, con edema
palpebral, acompañado de leve dolor en epigastrio a la palpación superficial y profunda además de
edema en miembros inferiores.

Al momento de su llega es diagnosticada con epilepsia más síndrome nefrótico en estudio. Es


valorada por médico especialista en Nefrología el cual envía exámenes de laboratorio, en los cuales
presenta un déficit significativo de hematíes, hemoglobina, volúmenes eritrocitarios bajos, lo cual
da diagnóstico para una anemia microcita e hipocromíca por disminución de VCM y HCM.

438 Vol. 5, núm. 3, julio, 2019, pp.434-442


Melania Consuelo Alcívar García,
Trabajo de Investigación Institucional del Octavo Semestre Paralelo “B” de la Facultad Ciencias de
la Salud de la Universidad Técnica de Manabí, Portoviejo, Ecuador.
Dom. Cien., ISSN: 2477-8818
Vol. 5, núm. 3, julio, 2019, pp. 434-442
Síndrome nefrótico a revisión de un caso

En la bioquímica sanguínea se encuentra una alteración con elevación de la urea y creatinina, lo


cual demuestra un daño en la función renal, acompañado de perdida de proteínas, a mayor expensa
de albumina, pero acompañada de disminución de globulina.

Consiguientemente se encuentra alterado el perfil lipídico.

Debido al daño renal que existe y la pérdida de proteínas se solicita exámenes más específicos de
los cuales resultaron en:

Una pérdida de proteínas en orina de 394.20 mg/dl

Microalbumina Orina Ocasional de 86.99 mg/dl

Microalbumina/creatinina 4740.90 mg/g de Creatinina,

Proteínas en orina de 24 en un Rango de 8.5 gr/24h

Lo cual da diagnóstico de confirmación para síndrome nefrótico en evolución. Se indica un


ultrasonido para visualizar cavidad abdominal en la cual se observa un derrame pleural bilateral de
pequeña cuantía, acompañado de una ligera a moderada cantidad de líquido libre ascítico
intrabdominal.

Tratamiento Aplicado

Cloruro de sodio al 0.9% pasar I.V. 1900ml en 20 horas, FB: 80ml/hora, Ranitidina 50 mg I.V.
cada 8 horas, Carbamazepina 200mg cada 12 horas, Eritropoyetina, Ácido fólico, Hierro

Se realizan exámenes de control, indicando una leve mejoría en los marcadores hematológicos.
Pero sigue existiendo alteración en la bioquímica sanguínea de la función renal.

Se le prescribe ampicilina+sulbactan 1.5gr IV cada 8 horas, prednisona 60mg VO 8 Am,


Espironolactona 25mg VO 16:00 horas, Furosemida 20mg QD, carbamazepina 200mg VO Am
;400mg VO Pm, Omeprazol 20mg VO QD antes del desayuno, Paracetamol 500mg VO cada 8
439 Vol. 5, núm. 3, julio, 2019, pp.434-442
Melania Consuelo Alcívar García,
Trabajo de Investigación Institucional del Octavo Semestre Paralelo “B” de la Facultad Ciencias de
la Salud de la Universidad Técnica de Manabí, Portoviejo, Ecuador.
Dom. Cien., ISSN: 2477-8818
Vol. 5, núm. 3, julio, 2019, pp. 434-442
Síndrome nefrótico a revisión de un caso

horas, ácido acetilsalicílico 8 mg después del almuerzo , eritropoyetina2000 U.I IV SC ,ácido


fólico 1 tableta VO antes de almuerzo, Liquido de mantenimiento VO 500ml QD.

La paciente es referida al Hospital Luis Vaca Ortiz de la Ciudad de Quito. Antes del traslado se le
realiza exámenes de control y se muestra falla renal y sin mejoría.

Comentarios

El síndrome nefrótico (SN) es una de las enfermedades crónicas más frecuentes en la niñez. Su
incidencia varía entre 1 a 3 casos por 100,000 habitantes menores de 16 años. Durante la edad
pediátrica, las primeras causas de enfermedad renal son las alteraciones estructurales de los riñones
y tracto urinario, las cuales pueden progresar a insuficiencia renal crónica. Se presenta con mayor
frecuencia entre los 2 y 8 años y en el sexo masculino. La mayoría de los casos que se inician entre
los dos y diez años de edad corresponden a SN primario o idiopático (SNI), y constituye la
alteración histológica subyacente más frecuente la enfermedad por cambios mínimos (ECM) o
nefrosis lipoidea; más raras son las formas secundarias a enfermedades sistémicas como vasculitis,
lupus eritematoso sistémico, diabetes melllitus, infecciones virales y otras. (Gómez Morejón, A.,
Pérez González, L., & Chaviano Mendoza, O. 2019). (Román E 2014), (López M, Yánez V,
Ramírez AE, Díaz DJ, Rivas K 2017).

El síndrome nefrótico (SN) se caracteriza clínicamente por la presencia de edemas, oliguria,


hipoalbuminemia, hiperlipidemia y proteinuria en el rango nefrótico (>40 mg/m2sc/hora). La
respuesta al tratamiento con esteroides es satisfactoria en un 80-95 % de los niños. Sin embargo,
alrededor de 70 % pueden recaer con frecuencia en el primer año, y algunos ser cortico-
dependientes o corticorresistentes, y por este motivo, requerir otro tipo de medicamento
inmunosupresor. (Behrman RKR. Nelson 2013), (González L, Cantillo J. 2013).

Las manifestaciones clínicas de nuestro paciente (facie pálida, edema palpebral y de miembros
inferiores), junto con los hallazgos de laboratorio (anemia microcitica, hipoalbuminemia con

440 Vol. 5, núm. 3, julio, 2019, pp.434-442


Melania Consuelo Alcívar García,
Trabajo de Investigación Institucional del Octavo Semestre Paralelo “B” de la Facultad Ciencias de
la Salud de la Universidad Técnica de Manabí, Portoviejo, Ecuador.
Dom. Cien., ISSN: 2477-8818
Vol. 5, núm. 3, julio, 2019, pp. 434-442
Síndrome nefrótico a revisión de un caso

Proteinuria de 8.5 gr/24h, elevación de azoados, hiperlipidemia), hacen probable el diagnóstico de


un Síndrome Nefrótico.

En este tipo de casos de síndrome nefrótico tomando en cuenta otras referencias se está de acuerdo
realizar tratamiento general en cuanto al edema, la hiperlipidemia, proteinuria, anticoagulación y
la hipertensión. En cuanto al edema se recomienda la restricción de sal 1,5 a 2 gramos las 24 horas,
reposo en decúbito o sedestación con piernas elevadas, diuréticos de asa vía oral y aumento
progresivo hasta dosis altas. En el caso de que haya hipertensión se recomienda IECA o ARAll
mantener una tensión arterial menor a 125/75 mm Hg; al presentar una hiperlipemia se recomienda
la dieta y el consumo de estatinas.

441 Vol. 5, núm. 3, julio, 2019, pp.434-442


Melania Consuelo Alcívar García,
Trabajo de Investigación Institucional del Octavo Semestre Paralelo “B” de la Facultad Ciencias de
la Salud de la Universidad Técnica de Manabí, Portoviejo, Ecuador.
Dom. Cien., ISSN: 2477-8818
Vol. 5, núm. 3, julio, 2019, pp. 434-442
Síndrome nefrótico a revisión de un caso

Referencias Bibliográficas

Behrman RKR. Nelson. Tratado de Pediatría. 18va. ED. Elsevier; 2013

Castaño I, Bolaños L. (2005). Colombia Médica, 36

Gómez Morejón, A., Pérez González, L., & Chaviano Mendoza, O. (2019). El síndrome nefrótico
en pediatría: un impacto en la infancia. Revista Finlay, 9(1), 20-25. Recuperado de
http://www.revfinlay.sld.cu/index.php/finlay/article/view/683/1735

González L, Cantillo J. (2013). Abordaje diagnóstico de la enfermedad glomerular del adulto.


Transformando una idea compleja en un proceso práctico. Acta Médica Colombiana. 38(2):101-7

López M, Yánez V, Ramírez AE, Díaz DJ, Rivas K. (2017). Estudio epidemiológico y demográfico
de la consulta externa de nefrología pediátrica. Rev Cubana Pediatr; 90(1)

Nedier Pedraza G., Ceballos M L , Cano SCH F.( 2008) Síndrome nefrótico cortico-resistente
secundario a mutación genética, a propósito de 2 casos clínicos. Rev Chil Pediatr 2008; 79(4): 398-
403 Recuperado de https://scielo.conicyt.cl/scielo.php?script=sci_arttext&pid=S0370-
41062008000400008

Román E. Síndrome nefrótico pediátrico. Protoc diagn ter pediatr;1(1):283-301

442 Vol. 5, núm. 3, julio, 2019, pp.434-442


Melania Consuelo Alcívar García,
Trabajo de Investigación Institucional del Octavo Semestre Paralelo “B” de la Facultad Ciencias de
la Salud de la Universidad Técnica de Manabí, Portoviejo, Ecuador.
Spanish Archives of Urology (Printed Ed.)printed version issn 0004-0614
printed version issn 0004-0614

Arch. Esp. Urol. Vol.58 no.7 Sep. 2005

Clinical cases

GIANT INGUINOESCROTAL VESICAL HERNIA WITH


OBSTRUCTIVE UROPATIA
BILATERAL INCIPIENT ASSOCIATED

Nicolás Alberto Cruz Guerra, Manuel Albarrán Fernández1, Máximo Porto Sierra
and Antonio Tarroc Blanco.

Urology and General Surgery Department1.


Zamora Hospital Complex. Zamora. Spain.

Abstract.- OBJECTIVE: Presentation of a case of inguinoscrotal bladder hernia with associated incipient
bilateral obstructive uropathy.
METHOD: We describe the case of a 54-year-old male patient who reported urination in two stages,
requiring scrotal compression to complete it. The physical examination revealed a large left inguinoscrotal
hernia, with a significant pre-compression ultrasound postvoid residue. Serum creatinine = 1.7 mg / dL was
observed. The diagnosis of bladder hernia with mild hydronephrosis was confirmed with voiding
cystourethrography and intravenous urography. Hernia reduction plus associated inguinal hernioplasty was
indicated.
RESULTS: Morphofunctionally satisfactory. Clinical and analytical standardization.
CONCLUSIONS: We highlight the infrequency of bilateral supravesical obstructive uropathy secondary to
bladder hernia. We agree with other authors on the validity of conservative reconstructive surgery for cases
such as the one described.

Key words: Bladder hernia. Hydronephrosis. Surgery.

INTRODUCTION

Inguinoscrotal bladder hernia, also known as scrotal cystocele, is an uncommon pathology whose incidence
becomes even more discrete for those cases associated with obstructive uropathy with impaired kidney
function (with only approximately ten clinical notes published in the last decade) . We present an example of
this entity, in an incipient stage of nephrological repercussion.

CLINICAL CASE
A 54-year-old male patient, obese, with a personal history of diabetes mellitus controlled with glipizide plus
metformin, as well as a significant smoking habit, who attended the emergency department due to difficulties
in completing urination, of several months of evolution, although more marked in the last 20 days; referring
to scrotal compression maneuvers being necessary for this. International Index of Prostate Symptoms (IPSS)
score = 17.

Physical examination revealed a large, uncomplicated left inguinoscotal hernia. The rectal examination did
not show significant findings. The results of the analytical determinations were within normal ranges, except
serum creatinine figures of 1.7 mg / dL. The presence of significant postvoid residue, absent ultrasound after
pressure on the hemiscrotal content indicated bladder catheterization. A serial voiding cystourethrography
was performed, which demonstrated the existence of a large bladder inguinoscrotal hernia (Figure 1).
Intravenous urography revealed displacement of both distal ureters towards the hernial neck, as well as some
proximal ectasia of the same (Figure 2).

Given the aforementioned findings, the patient was scheduled jointly with the General Surgery Service,
undergoing bladder hernia reduction plus inguinal hernioplasty according to the Rutkow-Robbins technique,
with a prolene mesh implant. The postoperative period was uneventful, except for incisional wound seroma,
which was resolved with drainage and local cures.

In a subsequent review, clinicoanalytical normalization was confirmed, as well as the urographic image, both
from a morphological and functional point of view (Figure 3).
DISCUSSION

Inguinoscrotal bladder hernia can occur in up to 10% of men over 50 years of age (1), although it is much
less frequent for the intrasacular cystic component to exceed 50% of the total of said organ, as occurred in
our case. The right predominance (60%) (2) was not verified in this patient, although it did correspond to the
most frequently described anatomical types (inguinal - 75% of cases -, and paraperitoneal - 60% -) (3).

The most determining pathophysiological factors in this entity are infravesical obstructive uropathy and
abdominal parietal weakness at the level of the inguinal canal. Overweight and increases in intra-abdominal
pressure in potentially coughing subjects exert an adjuvant effect (4).

Most incipient bladder hernias behave asymptomatically. When they progress in dimensions, two-stage
urination (aided in the second phase by scrotal manual pressure) is the most frequently referred manifestation
- Mery's sign - (3). Renal failure associated with this entity has also been described in the literature (1) (3),
although with a much lower incidence, occurring in those cases of associated trigonal herniation and
consequent ureteral retraction and angulation that condition obstructive uropathy. Our clinical case offers the
peculiarity of having been detected in an incipient phase of renal function alteration. Other published
findings in relation to bladder hernias have been vesicoureteral reflux (5), lithiasis (6), and even urothelial
neoplasms (7).

Within the diagnostic imaging methods, voiding cysturethrography has been considered the initial test of
choice (3), although some authors have claimed the complementary cost-effectiveness of ultrasound (8). The
planning of an interventional treatment benefits from the previous performance of other techniques such as
intravenous urography, in which the classic Reardon-Lowman triad has been described (distal ureteral lateral
displacement plus asymmetric and small bladder, as well as incompletely visualized bladder base ) (9).
Computerized axial tomography can also provide useful information in order to outline the vesicoureteral
morphology of each case. Flexible cystoscopy gains value in the evaluation of possible hernial contents
already mentioned (2).

There is a broad consensus (1-3) regarding the surgical indication in the treatment of bladder hernias,
involving the reduction of the saccular content and subsequent correction of the parietal defect at the level of
the inguinal canal. We agree with other authors (2) on the validity, whenever feasible, of a conservative
attitude with respect to the herniated bladder portion (except in those cases in which associated '' in situ ''
pathology is present), since - as is our case - the functional results are satisfactory.
Informe de un caso: Insuficiencia renal aguda por envenenamiento masivo de un niño
de dos años causada por picaduras de abejas asesinas

En una aldea a lo largo del río Maroni, en la Guayana Francesa, una niña de 2 años fue
atacada por una colonia de abejas alrededor de las 15:00 horas en el exterior, cerca de su
casa. Al cabo de 2 horas, fue ingresada en un centro sanitario alejado con dolor difuso, diarrea,
vómitos e inflamación cutánea en todo el cuerpo, incluida la cara, con oclusión completa de
los párpados. La exploración clínica no reveló ningún déficit hemodinámico, respiratorio o
neurológico.

La paciente pesaba 10 kg, y se extrajeron aproximadamente 100 aguijones de abeja, es decir,


10 aguijones/kg, la mayoría de los cuales estaban localizados en la cabeza, incluyendo los
párpados y la boca, el cuello, la parte superior del cuerpo y las extremidades superiores.

Inmediatamente se le administraron 20 mg de prednisolona, 5 mg de polaramina, 150 mg de


paracetamol y una solución poliónica a 150 mL/kg/24 horas por infusión intravenosa.
Dieciocho horas después del envenenamiento, seguía anúrica. El médico de primeros auxilios
no tenía recursos para realizar un análisis de sangre y solicitó la evacuación médica en
ambulancia aérea a una unidad de cuidados intensivos (UCI) pediátrica.

A su llegada a la UCI, la presión arterial era de 100/60 mmHg, la frecuencia cardíaca de 110
latidos/minuto, la temperatura de 37°C, la saturación de oxígeno del 100% y su estado de
conciencia no estaba alterado. Parecía tener dolor y permanecía anúrica.

Siguió recibiendo 20 mg/día de prednisolona, 5 mg/día de dexclorfeniramina, 600 mg/día de


paracetamol e hidratación con una solución poliónica durante los primeros 7 días. También
introdujimos 10 mg/día de furosemida durante 3 días.

Sus síntomas gastrointestinales cesaron 36 horas después del envenenamiento, su diuresis


se normalizó a 5 mL/kg/hora en 72 horas, y su edema facial persistió durante la primera
semana.

El análisis bioquímico (Tabla 1) mostró una insuficiencia renal funcional aguda que se agravó
hasta el cuarto día, junto con una hiperpotasemia que alcanzó su punto máximo a la hora 30.
Al mismo tiempo, el aumento de la creatina-cinasa (CK) y de las transaminasas (aspartato-
transferasa y alanina-transaminasa) indicaba una rabdomiólisis grave.

Tabla 1: Tabla resumen de los resultados de la bioquímica sanguínea

Tiempo
después 19 30 12 15
3 días 4 días 5 días 7 días 9 días
de las horas horas días días
picaduras
Na135-
145 129 130 130 133 133 142 140 141 137
mmol/L
Na
296.56 296.56 296.56 305.75 305.75 326.44 321.85 324.14 314.95
mg/dl
K 3.5–5
7.4 8.1 5.6 4.8 3.9 4.6 4.6 4.8 4.8
mmol/L
K
28.94 31.67 21.9 18.77 15.25 17.99 17.99 18.77 18.77
mg/dl
K
corregido
- 6.7 - - - - - - -
(pH =
7,16)
Urea 1.8–
6.0 15.8 18.7 26.8 28.9 23.9 14.5 2.2 3.1 3.1
mmol/L
Urea
94.91 112.33 160.98 173.6 143.56 87.1 13.22 18.62 18.62
mg/dl
Creatinina
30–70 135 169 255 257 125 74 33 34 34
µmol/L
Creatinina
1.53 1.91 2.88 2.9 1.41 0.84 0.37 0.38 0.38
mg/dl
Creatinina
cinasa
7,206 >20,000 >20,000 >20,000 19,620 2,815 797 124 77
29–168
U/L
ALT 7–35
112 228 374 513 498 271 157 77 33
U/L
AST 9–45
- 3,175 1,776 1,440 510 87 40 33 28
U/L

Su hiperpotasemia mejoró con sulfonato de poliestireno y salbutamol, y la función renal


empezó a mejorar el cuarto día sin diálisis. Los valores normales de creatinina sérica y CK se
alcanzaron los días 9 y 12, respectivamente.

Desde el punto de vista hematológico, la trombocitopenia y la anemia por dilución se


produjeron con la hidratación masiva y mejoraron con la disminución de la velocidad de
infusión y la reanudación de la diuresis.

Finalmente, nuestra paciente salió del hospital sin secuelas tras 18 días de vigilancia clínica
y seguimiento biológico, incluyendo los 6 primeros días en la UCI.

DISCUSIÓN

En la Guayana Francesa, situada en la Zona de Convergencia Intertropical, todas las abejas


locales se han africanizado desde la llegada de las "abejas asesinas" en 1975.

El fenómeno de la hibridación no modificó la composición de su veneno, que es un cóctel


complejo de 102 proteínas y péptidos. Hasta la fecha, no se ha comercializado ningún
antiveneno, a pesar de la investigación activa en la fase de ensayos clínicos con adultos.

El principal problema de las "abejas asesinas" son sus ataques en forma de enjambre con
cientos de individuos. Este comportamiento específico se explica por la liberación de una
feromona (2-heptanona) en la piel de su objetivo, además de la inyectada a través del veneno,
emitiendo así una potente señal de alarma.

Los niños, por su corta edad y bajo peso, corren un alto riesgo de sufrir consecuencias
peligrosas por el ataque de un enjambre de abejas. Su capacidad para huir, protegerse o
alertar es tanto más reducida cuanto menor sea su edad. Además, cuanto menor sea su peso,
más grave será el envenenamiento.
Así, en 2018, Schmidt definió una escala de morbilidad y mortalidad según el peso de la
víctima y el número de picaduras, así como una dosis letal media de 19 picaduras/kg. Nuestra
paciente recibió 10 picaduras/kg. Según Schmidt, se trata de un nivel de envenenamiento
"muy alto".

En los niños, al igual que en los adultos, este tipo de envenenamiento puede dar lugar a una
reacción sistémica bifásica con, al principio, una reacción inmediata de tipo anafiláctico
(afectación del tejido cutáneo-mucoso, compromiso respiratorio, reducción de la PA o
síntomas asociados, y síntomas gastrointestinales persistentes), seguida de una reacción
tóxica (rabdomiólisis), que puede resultar mortal debido a la insuficiencia renal aguda (IRA)
en un plazo de 72 horas (Tabla 2).

Los mecanismos fisiopatológicos de este FRA incluyen, por un lado, la hipovolemia prerrenal
resultante de la disminución de la resistencia vascular debida a la anafilaxia, así como las
lesiones tubulares debidas a la rabdomiólisis y la potencial nefrotoxicidad directa de la melitina
(aproximadamente el 50% del peso seco del veneno de abeja).

Tabla 2: Puntos clave del envenenamiento masivo por abejas asesinas (más de 1 picadura/kg)
Reacción sistémica
Inmediata Tardía
• Anafilaxia: • Rabdomiólisis
Afectación del tejido
• Insuficiencia renal aguda
cutáneo-mucoso,
Compromiso respiratorio
Diagnóstico Reducción de la PA o
síntomas asociados,
Síntomas gastrointestinales
persistentes.

• Epinefrina IM, 10 µg/kg,


repetida cada 10-15
Tratamiento recomendado minutos hasta que haya
respuesta, • Hidratación salina
por las sociedades
isotónica.
científicas • Relleno vascular,
• Oxigenoterapia si es
necesario.

En nuestro caso, la combinación inmediata de signos clínicos (edema facial, diarrea y vómitos)
permite diagnosticar una anafilaxia, según los criterios planteados por Sampson17 y basados
en las guías americanas y europeas.

Esta reacción grave sin exposición previa fue causada, por un lado, por la acción directa de
las aminas biógenas del veneno y, por otro, por la degranulación de los mastocitos causada
por la melitina y el "péptido de degranulación de los mastocitos" del veneno.

De acuerdo con las directrices americanas y europeas, el tratamiento precoz mediante la


inyección intramuscular de epinefrina, a una dosis de 10 µg/kg para un niño (repetida cada
10-15 minutos hasta que haya respuesta),30 puede bloquear la activación de los mastocitos,
reducir la cascada de mediadores de la anafilaxia y aumentar la resistencia vascular.
Sin embargo, la epinefrina se infrautiliza habitualmente; según Worm, sólo alrededor del 20%
de los casos de anafilaxia europeos reciben epinefrina, a diferencia de los corticosteroides y
los antihistamínicos, que no constituyen un tratamiento de urgencia.

Nuestra paciente no recibió epinefrina. Se anurizó a la hora 18, y su primer análisis de sangre
muestra un FRA que se produce antes del pico de CK.

Tal y como se recomienda, la reacción anafiláctica debe ser monitorizada durante al menos 6
horas tras la desaparición de los síntomas. En el caso de un envenenamiento de más de una
picadura de abeja/kg, este periodo de monitorización también permitirá detectar la segunda
fase de la reacción.

De hecho, la melitina y la fosfolipasa A2 actúan alterando la integridad de las membranas


celulares, causando generalmente rabdomiólisis grave, complicada por el FRA. Según
Szugye, el contenido intracelular de las células musculares muertas que se libera al espacio
extracelular y circula es responsable del daño renal debido a los siguientes mecanismos
fisiopatológicos en primer lugar, la activación del sistema renina-angiotensina-aldosterona,
que reduce el flujo sanguíneo renal; en segundo lugar, la mioglobina (una proteína hemo que
se une al oxígeno) se libera en cantidades abrumadoramente elevadas y precipita en el filtrado
glomerular, causando daños en los túbulos renales; y en tercer lugar, el ácido úrico se libera
y forma depósitos de cristales que provocan la destrucción tubular. En nuestro caso, el nivel
de CK en sangre se mantuvo muy alto durante los primeros 4 días (> 20.000 U/L). Actualmente
no existen guías para el manejo de la rabdomiólisis pediátrica, pero, según la revisión de
Szugye, el suero salino isotónico es el más utilizado para la hidratación.

Así pues, nuestra paciente sufrió inicialmente un FRA por una anafilaxia mal tratada, que
posteriormente se agravó con la rabdomiólisis. Sufrió un error de tratamiento muy común al
no recibir epinefrina inmediatamente. La inyección inmediata de epinefrina, manteniendo la
función renal durante la fase anafiláctica inicial, habría evitado la acumulación de mioglobina
y ácido úrico en la sangre debido a la rabdomiólisis. De este modo, los túbulos renales podrían
haberse preservado.

CONCLUSIÓN

En la Guayana Francesa, los pueblos remotos sólo son accesibles por avión o canoa. En este
contexto, los médicos aislados deben ser conscientes de los signos de anafilaxia. En tales
situaciones, deben administrar epinefrina junto con un relleno vascular lo antes posible para
detener rápidamente una reacción tóxica en cadena potencialmente mortal.

A continuación, durante las primeras 24 horas, la vigilancia clínica y la monitorización biológica


en una UCI son esenciales para la detección precoz del shock hipovolémico y la dificultad
respiratoria.

Por último, aunque la reacción inicial parezca estar clínicamente resuelta, la toxicidad directa
del veneno a una dosis superior a 1 picadura/kg puede provocar un FRA por rabdomiólisis.
Esto justifica una vigilancia médica atenta durante al menos 72 horas.
ROBERT CÓRDOVA – MARLON CÓRDOVA
Hipertensión arterial
Generalidades

• Enfermedad con cuadro nosológico propio. Factor de riesgo cardiovascular.


• Etiología desconocida en el 90% de los casos.Secundaria en hasta el 10% de los
casos.Afecta hasta el 25% de la población adultaEnfermedad Cardiovascular 2a. causa
de mortalidad anual en el mundo.Hipertensos 20% de la población general y 50% de la
población entre 65 y 74 años.

Clasificación

• Presión arterial óptima: <120/80 mm de Hg


• Presión arterial normal: 120-129/80-84 mm de Hg
• Presión arterial normal alta: 130-139/ 85-89 mm de Hg
• Hipertensión arterial:
a) Etapa 1: 140-159/ 90-99 mm de Hg
b) Etapa 2: 160-179/ 100-109 mm de Hg
c) Etapa 3: >180/ >110 mm de Hg

Clasificación etiológica.

• Primaria o esencial.
• Secundaria.
a) Renal.
b) Endocrinas.
c) Cardiovasculares.
d) Sistema nervioso central.
e) Medicamentos.
f) Tóxicos.
g) Gestacional.
h) Ambientales.

Fisiopatología

• Anormalidades del sistema nervioso simpático.Disfunción endotelial-.Alteraciones


metabólicas.Alteración de la relación presión/diuresis.Sistema renina-angiotensina-
aldosterona.

Diagnóstico.Fundamentalmente clínico.Establecer si la hipertensión es primaria o


secundaria.Establecer la repercusión orgánica.Hipertensión de bata blanca.
Cuadro clínico.Inespecífico.Varía según edad, sexo, tipo de HTA y tiempo de evolución.Puede
ser asintomática o presentarse con manifestaciones clínicas de alguna de sus complicaciones
Complicaciones.

• El exceso de morbilidad y mortalidad producida por la HTA, se relaciona directamente


con los niveles de presión arterial sistólica y diastólica.
• Neurológicas.
• Cardiovasculares.
• Renales

Tratamiento.
Enfatizar con el paciente sobre la importancia de ser constante en su tratamiento,
proporcionando siempre información exhaustiva sobre la enfermedad, llevando controles
clínicos frecuentes y solicitar colaboración de otros profesionales de la salud.
Objetivo primordial: evitar complicaciones y prevenir la morbilidad asociada
Tratamiento no farmacológico.

• Modificaciones en la dieta.
a) Restricción de sal.
b) Control de sobrepeso.
c) Potasio en la dieta.
d) Calcio y magnesio en la dieta.
e) Restricción de grasas saturadas.

Fármacos.

• Diuréticos. .
• Beta bloqueadores. .
• Alfa bloqueadores.
• Calcioantagonistas.
• Inhibidores de ECA.
• Inhibidores de receptores de ATII
• Inhibidores adrenérgicos periféricos
• Agonistas centrales.
• Vasodilatadores

NEFROPATÍA HIPERTENSIVA
Benigna, Maligna, Silente-Asintomático-Evoluciona a Crónico
Producido por: Silente-Asintomático-Evoluciona a Crónico lo que produce Endurecimiento de
las arterias y arteriolas de los riñones, formando Hipertrofia=Nefroesclerosis y Deteniendo la
función normal del riñón.
FACTORES
-Raza Negra -Bajo Peso al Nacer -Polimorfismo del APOL 1
EPIDEMIOLOGÍA
En el Ecuador presenta una alta prevalencia y morbimortalidad, Prevalencia 35-40% en la
población en general, Predominio en pacientes mayores de 60 años en ambos géneros, HTA
segundo lugar en la enfermedad renal crónica terminal, 4 de 10 adultos en el mundo padece de
HTA y aumenta con la edad.
RESTO DEL MUNDO
EEUU 31-46% CANADA 32-46% INDIA 29-43% CHINA 25-50%, COREA DEL SUR 28-
48%
PATOLOGÍA
afectación vascular -glomerular -tubulointersticial
Hipertrofia: Engrosamiento fibroblástica intima estrechamiento de la luz vascular, Respuesta es
adaptativa de la hipertensión arterial crónica sobre la intima.
Hialinosis arteriolar: Depósitos hialinos en puntos donde la musculatura lisa es deficiente,
Favorece la exudación de proteínas plasmáticas.
GLOMERULOESCLEROSIS: Pueden mostrar tanto la esclerosis global focal como la
esclerosis focal segmentaria
La esclerosis global: Refleja lesión isquémica = pérdida de las nefronas, todo el glomérulo puede
estar comprometido, Frecuencia raza negra.
Esclerosis focal y segmentaria: Respuesta compensatoria a la pérdida de nefronas, Se asocia a
glomérulos aumentados de tamaño, Alteraciones hemodinámicas resultado del metabolismo
anormal del óxido nítrico.
TÚBULO-INTERSTICIO: Compromiso vascular y glomerular se asocia a menudo con nefritis
intersticial severa. Condiciones hipóxicas favorecen la transdiferenciación de las células tubulares
en miofibroblastos, promoviendo la fibrogénesis
FIBROSIS INTERSTICIAL Y ATROFIA TUBULAR
Estenosis de la arteria renal principal puede inducir atrofia tubular y una afluencia de células
inflamatorias. Está expuesto a presión arterial alta, pero sin isquemia, desarrolla cambios
tubulointersticiales.
FISIOPATOLOGIA
Existen 3 factores con fuerte asociación entre ellos
• El tiempo de evolución, así como el grado y el control de HTA.
• A mayor intensidad y precocidad de la hipertensión, existe mayor posibilidad de lesión
cardiovascular y renal
• La prevalencia de estas complicaciones se incrementa de manera
• Importante cada 10 años y con presión arterial ≥ a 160/90 mmHg
Aparece la micro albuminuria, para luego comenzar la caída de la tasa de filtración glomerular y
la capacidad de depuración de azoados.
FACTORES DE PROGRESION DE ENFERMEDAD RENAL EN HIPERTENSOS
Progresión de la enfermedad renal se agrupan 3 mecanismos principales
Las alteraciones vasculares en los glomérulos dañados: producen disminución de la perfusión
glomerular e isquemia tubular, favoreciendo la atrofia e inflamación del túbulo y el intersticio.
La proteinuria producida por el aumento de la presión de filtración: es reabsorbida
por los túbulos y causa inflamación intersticial, aumento de la matriz extracelular y atrofia tubular.
La derivación del ultra filtrado glomerular al intersticio peri glomerular y peritubular :
Presenta lesiones de tipo GEFS--injuria glomerular segmentaria a la esclerosis global,
degeneración tubular y fibrosis intersticial.
MANIFESTACIONES CLINICAS
❖ La hipertensión está presente durante > 10 años, con evidencia de periodos de PA acelerada
o mal controlada
❖ aumentos lentamente progresivos en el (BUN)
❖ concentración de creatinina en plasma,
❖ proteinuria leve.
Las características que sugieren el diagnóstico de Nefroesclerosis hipertensiva

• Hipertensión que precede a la disfunción renal.


Mayores de 55 años de edad
• Raza negra con su GEN APOLO1
Sexo masculino
• Hipertensión de larga duración o persistente
Proteinuria menos de 0.5 g / dl
• Hipertensión diagnosticada antes de la aparición de proteinuria
Un resultado TFGe de menos de 60 mililitros por minuto (mL/min)
• Hipertrofia del ventrículo izquierdo
Hallazgos de biopsia compatibles con el diagnóstico

EXAMEN FISICO
Aparecer signos y síntomas de enfermedad renal crónica
• anorexia, náuseas, vómitos, prurito, somnolencia y confusión
• Daños orgánicos secundarios a la hipertensión
• Evidencia de daño en el órgano diana relacionado con la hipertensión
• Las hemorragias o exudados son característicos de la hipertensión acelerada, y el papiledema
DIAGNOSTICO
❖ Principalmente clínico, y se apoya en la ecografía y en los resultados de las pruebas de
laboratorio de rutina.
❖ La ecografía-La biopsia renal
❖ Análisis de sangre de rutina puede indicar un deterioro de la función rena
HIPERURICEMIA
• Hallazgo en la nefroesclerosis benigna-reflejar la reducción del flujo sanguíneo renal.
• Análisis de orina suele ser benigno y el sedimento revela pocas células o cilindros
PROTEINURIA
• La excreción de proteínas suele estar levemente elevada menos de 1 g / día puede llegar a
10g/día
• Glomérulos menos afectados han sido sometidos a hipertrofia compensatoria con una presión
intraglomerular más alta.
INCIDENCIA DE INSUFICIENCIA RENAL
• la tasa de progresión generalmente es lenta y con hipertensión primaria (hipertensión
"esencial”) desarrollan una enfermedad renal progresiva

TRATAMIENTO
Reducción de la albuminuria como un factor clave para retardar al máximo la progresión de la
ERC
1) Disminuir la PA (A).
2) retardar la progresión de la enfermedad renal (A)
3) reducir el riesgo de ECV (B).
Tratamientos de la ERC como parte de una estrategia multi intervención
❖ Modificación de hábitos de vida en hipertensos con ERC
❖ Estadios 1 y 2 de la ERC se puede aplicar la dieta DASH
❖ hipertensos con ERC la restricción de sal
❖ Restricción moderada de sodio (2.000 mg) o de sal (5 g) por día
❖ Se prohibirá el uso de sales sustitutas
❖ Estadios 3 y 4 se bajarán las proteínas a 0,8-1,0 g/kg peso ideal/día
❖ Restringir el fósforo a 0,8-1,0 g/día para mejor manejo del hiperparatiroidismo secundario,
❖ Fármacos antihipertensivos. Los preferidos son los lECA y ARA II
❖ SRAA para disminuir la hipertensión intraglomerular y la proteinuria,
IECA O ARA II
Captopril: 25 mg cada 8 hr Enalapril: 5- 10 mg cada 12 horas Telmisartán: 40 80 mg/d
Quinapril: 4 mg/d Lisinopril: 10-20 mg/ d, Perindopril: 4-8 mg/d, Losartán: 50-100 mg/d
Valsartán: 80-180 mg/d
Reporte de un caso

Diagnóstico del complejo de esclerosis tuberosa en un


paciente derivado por hipertensión no controlada y disfunción
renal: un caso que resalta la importancia de una adecuada
evaluación diagnóstica de los pacientes hipertensos
Pantelis A. Sarafidisa, Athanasios Bikosa, b, Charalampos Loutradisa, Stergios TzikasC,
Efstratios VakirlisD, Eugenia Avdelidoumi, Dimitrios ZafeiriouF, Aikaterini Papagiannia,
y Vasileios VassilikosC

Presentamos el caso de una mujer de 39 años con hipertensión ESC, Sociedad Europea de Hipertensión / Sociedad Europea de
arterial resistente y disfunción renal. La paciente fue hospitalizada 3 Cardiología; FLAIR, recuperación de inversión atenuada por fluido;
meses antes por disnea en el Departamento de Cardiología, donde LV, ventrículo izquierdo; FEVI: fracción de inyección del ventrículo
se le diagnosticó insuficiencia cardíaca (fracción de inyección del izquierdo; mTOR, diana mecanicista de la rapamicina; RVSP: presión
ventrículo izquierdo: 25-30%), hipertensión pulmonar, enfermedad sistólica del ventrículo derecho; T2W, ponderado en T2; TSC,
renal crónica (creatinina sérica: 1,58 mg / dl), e hipertensión complejo de esclerosis tuberosa
resistente y dado de alta con tratamiento óptimo de insuficiencia
cardíaca. En el momento de la presentación en nuestra clínica,
además de hipertensión no controlada durante más de 10 años y

T
antecedentes de preeclampsia y pérdida fetal, la paciente tenía
obesidad (IMC: 38 kg / m2) y fibromas faciales. Los primeros pasos trastorno autosómico dominante, estimado en 1 / 6000–
diagnósticos propuestos por las guías de la Sociedad Europea de El complejo
10000 de esclerosis
nacimientos uberosa
[1,2]. Es (CET)
causada por es un
inactivar
Hipertensión / Sociedad Europea de Cardiología (ESH / ESC) para mutaciones en TSC1 (cromosoma 9q34) o TSC2 (genes del cromosoma
identificar otros daños en órganos diana y causas de hipertensión 16p13.3), que codifican la hamartina y la tuberina, proteínas que
secundaria revelaron nefropatía hipertensiva proteinúrica típica, suprimen la vía de señalización del objetivo mecanicista de la
retinopatía hipertensiva y síndrome de apnea del sueño. Además, rapamicina (mTOR) [3]. Pérdida deTSC La función genética da como
una ecografía renal mostró múltiples angiomiolipomas renales resultado un crecimiento celular descontrolado y tumores benignos
bilaterales, confirmados por una resonancia magnética. Tras la en varios órganos (cerebro, riñones, pulmones, corazón, retina y piel)
consulta con los Servicios de Neurología y Dermatología, se confirmó [3,4]. El diagnóstico de CET se basa en 11 criterios clínicos mayores y
el diagnóstico de complejo de esclerosis tuberosa, basado en la nueve menores (se requiere la presencia de dos mayores o uno mayor
presencia de seis criterios principales. Durante los siguientes 10 y dos menores) y / o pruebas genéticas [2]. La afectación renal es
meses, ajustes cuidadosos en el tratamiento antihipertensivo de la frecuente en TSC; las lesiones más frecuentes son los
paciente, el refuerzo de las intervenciones en el estilo de vida y un angiomiolipomas [5]. El TSC rara vez afecta a vasos de tamaño grande
mejor cumplimiento le permitieron reducir su peso corporal, y mediano, incluidas la aorta y las arterias renales [6,7]. La
controlar la presión arterial, mejorar su corazón (fracción de hipertensión en el CET se considera el resultado de una enfermedad
inyección del ventrículo izquierdo:> 40%) y lesión renal (aclaramiento renal y se asocia con el tamaño y la cantidad de
de creatinina en orina: 125 ml / min, proteína en orina: 178 mg / 24
días) y triglicéridos séricos (153 mg / dl). Estas mejoras permitieron el
inicio de everolimus, necesario para un ligero aumento del tamaño
Journal of Hypertension 2017, 35: 2109–2114
de los angiomiolipomas (3,46 cm) en las exploraciones repetidas. aDepartamento de Nefrología, BTercer Departamento de Cardiología, Hospital Hippokration,

Universidad Aristóteles de Thessaloniki, Thessaloniki, CUnidad de Hemodiálisis Pieria, Katerini,


DPrimer Departamento de Dermatología, Hospital Hippokration, Universidad Aristóteles de

Tesalónica, miDepartamento de Neurología, Hospital Hippokration y FPrimer Departamento de


Pediatría, Hospital Hippokration, Universidad Aristóteles de Salónica, Salónica, Grecia

Palabras clave: everolimus, nefropatía hipertensiva


proteinúrica, hipertensión resistente, complejo de esclerosis Correspondencia a Pantelis A. Sarafidis, MD, MSc, PhD, Departamento de Nefrología, Hospital
Hippokration, Universidad Aristóteles de Thessaloniki, Konstantinoupoleos 49, GR54642
tuberosa Thessaloniki, Grecia. Tel: +30 2313 312930; fax: +30 2313 312930; correo electrónico:
psarafidis11@yahoo.gr
Abreviaturas: BP, presión arterial; ERC, enfermedad renal crónica;
Recibió 14 de abril de 2017 Revisado 1 de mayo de 2017 Aceptado 3 de mayo de 2017
CKD-EPI, CKD Epidemiology Collaboration; CPAP, presión positiva
J Hypertens 35: 2109–2114 Copyright 2017 Wolters Kluwer Health, Inc. Todos los derechos
continua en las vías respiratorias; CrCl, aclaramiento urinario de reservado.
creatinina; eGFR: tasa de filtración glomerular estimada; ESH / DOI: 10.1097 / HJH.0000000000001423

Revista de hipertensión www.jhypertension.com 2109


Copyright © 2017 Wolters Kluwer Health, Inc. Todos los derechos reservados.
Sarafidis et al.

angiomiolipomas [8]. Se notificaron algunos casos de hipertensión su tratamiento antihipertensivo, que revirtió sus síntomas de
renovascular en niños, como resultado de la presión de grandes disnea y redujo los niveles de PA a 150-160 / 85-100 mmHg.
angiomiolipomas o aneurismas aórticos [6]. A continuación, Fue dada de alta a los 7 días con carvedilol, enalapril,
presentamos el caso de un paciente con CET fulminante, que eplerenona, furosemida e ivabradina (tabla 1). Debido al
permaneció sin diagnosticar durante muchas décadas e hipertensión aumento de creatinina sérica (1,5 mg / dl) y antecedentes de
resistente de larga duración, en quien se identificó CET durante los hipertensión, fue remitida a nuestra clínica para una
primeros pasos diagnósticos de hipertensión resistente. evaluación adicional.
En el momento de la presentación en nuestra clínica, el paciente no
refirió síntomas agudos. Además de insuficiencia cardíaca e
REPORTE DE UN CASO
hipertensión pulmonar, su historial también incluyó hipertensión no
Una mujer de 39 años fue remitida a nuestra clínica de nefropatía controlada durante más de una década, preeclampsia con pérdida
hipertensiva / diabética por hipertensión resistente y deterioro de la fetal espontánea durante su primer embarazo (a los 28 años) y
función renal. Su historial reciente comenzó antes de las 12 semanas, segundo embarazo normal. Fumó cinco cigarrillos al día durante 20
cuando visitó el Departamento de Emergencias por falta de aire. Su años hasta su ingreso. Mencionó síntomas frecuentes de fatiga
saturación de oxígeno fue del 91% en aire ambiente y la presión diurna, ronquidos fuertes y apneas del sueño. Recordó haber usado
arterial (PA) fue de 190/110 mmHg. La auscultación torácica reveló varios antihipertensivos antes de la admisión (entre los que se
estertores húmedos bilaterales leves. Un ECG de 12 derivaciones encuentran irbesartan / hidroclorotiazida, betaxolol y metildopa). Su
mostró ritmo sinusal, frecuencia cardíaca de 65 lpm y depresión del padre murió a los 66 años por una enfermedad de las arterias
segmento ST en V5-V6. Ingresó en el Departamento de Cardiología, coronarias. En el examen clínico, era obesa (peso 121 kg, altura 1,72
donde las pruebas estándar no mostraron isquemia coronaria m, IMC 41 kg / m2) con masa muscular preservada. La PA en el
(troponina-I: 0,02metrog / l; hemoglobina: 13,9 g / dl; creatinina: 1,45 consultorio estaba en 170/100 mmHg, y el paciente informó un efecto
mg / dl; sodio: 139 mEq / l; potasio: 3,9 mEq / l; aspartato de "bata blanca". Su inspección reveló fibromas faciales, por los que
aminotransferasa: 17 U / l; alanina aminotransferasa: 13U / l; había sido examinada desde pequeña por varios dermatólogos. El
creatinfosfoquinasa: 181U / l). Un ecocardiograma transtorácico examen del corazón y el abdomen fue normal, la auscultación
mostró alteración de la fracción de eyección del ventrículo izquierdo pulmonar reveló roncus musicales, los pulsos periféricos fueron
(VI) (25-30%), hipertrofia concéntrica del VI (índice de masa del VI: normales pero había edema en la parte inferior de la pierna. El
147,2 g / m2), regurgitación mitral moderada y sobrecarga de la paciente recibió educación detallada sobre las medidas del estilo de
circulación pulmonar (presión sistólica del ventrículo derecho: 60 vida para la hipertensión y se le indicó que realizara mediciones de la
mmHg). El paciente recibió diuréticos intravenosos, moxifloxacino y PA en el hogar. Se añadió amlodipino 10 mg. Más lejos
modificaciones de

TABLA 1. Resultados de las pruebas de laboratorio y tratamiento farmacológico al alta del paciente del Servicio de Cardiología y durante nuestro seguimiento
(el 11/2015 a la paciente se le prescribió amlodipino 10 mg, que nunca usó)

6/2015 11/2015 02/2016 05/2016 10/2016 4/2017

Parámetro
Peso corporal (kg) 121 121 110 116 112 112
Creatinina sérica (mg / dl) 1,45 1,58 1,43 1,27 1.01 0,97
eGFR (ml / min / 1,73 m2) 45 41 46 53 70 73
Urea sérica (mg / dl) 40 58 37 40 28 20
Hemoglobina (mg / dl) 13,9 13,2 13,5 13,4 13,2 12,4
Glucosa en ayunas (mg / dl) - 94 87 83 85 82
Potasio sérico (meq / l) 3,5 4.4 4,7 4.2 4.0 4.2
Sodio sérico (meq / l) 139 143 142 141 141 142
Calcio sérico (mg / dl) 8.8 8.7 9 9 8.8 9.1
Fosfato sérico (mg / dl) Ácido - 3.1 3,5 3.3 3,0 3,0
úrico (mg / dl) 7.3 9.5 9.5 7.7 - 4.5
Triglicéridos (mg / dl) 140 218 - 183 153 256
LDL (mg / dl) 75 99 - 95 102 81
Parathormone (ng) - 69 - - 46 -
Aclaramiento de creatinina (ml / min) - 148 - - 125 -
Potasio en orina (mg / 24 h) Sodio en - 54 - - 49 -
orina (mg / 24 h) - 154 - - 77 -
Excreción de albúmina en orina (g / 24h) - 857 - 787 178 -
Tratamiento médico
Eplerenona 25 mg 11 11 Detener 11 11 11
Furosemida 40 mg 11 11 1/2 2 1þ1/2 1þ1/2 1þ1/2
Carvedilol 12,5 mg 12 12 12 12 12 12
Enalapril 20 mg 11 11 Detener

La ivabradina 5mg 12 12 12 12 12 12
OlmesartánþHCTZ (20/25 mg) 11 11 11 11
Amlodipino 5 mg 12 12 12
Everolimus 5 mg 11 11

eGFR: tasa de filtración glomerular estimada; HCTZ, hidroclorotiazida.

2110 www.jhypertension.com Volumen 35 Número 10 de octubre de 2017

Copyright © 2017 Wolters Kluwer Health, Inc. Todos los derechos reservados.
Diagnóstico del complejo de esclerosis tuberosa durante la evaluación de la hipertensión

Se ordenaron investigaciones para: identificar otros posibles Uso de CPAP. También informó un aumento de la actividad física y una
daños en órganos diana, identificar el estadio y la causa de la reducción de peso de 11 kg. El examen físico mostró reducción del
enfermedad renal crónica (ERC), excluir otras causas edema de la parte inferior de la pierna. El paciente presentó medidas
secundarias de hipertensión e investigar la posibilidad de de PA domiciliaria con PA reducida a 140-156 / 80-90mmHg. Las
síndrome de sueñopapnea. pruebas de laboratorio estándar se muestran en la Tabla 1. Una nueva
En el seguimiento (2 semanas), el paciente devolvió el ecografía cardíaca mostró mejoras en la función cardíaca [fracción de
informe oftalmológico que sugería un estrechamiento inyección del VI (FEVI)> 40%] y el cardiólogo tratante interrumpió la
arteriolar generalizado leve y una tortuosidad vascular administración de eplerenona. Sin embargo, el paciente informó
(retinopatía de Keith-Wegener-Baker grado 2). La ecografía haber interrumpido temprano el amlodipino prescrito debido a un
triplex de arterias carótidas y vertebrales no mostró lesiones edema en la pierna "intolerable" y a la "ingestión de muchas píldoras".
ateroscleróticas. Un estudio de sueño durante la noche arrojó Debido a un control inadecuado de la PA, se sustituyó enalapril por
52 episodios de apnea / h que condujeron al diagnóstico de olmesartán 20 mg / hidroclorotiazida 25 mg una vez al día. A los 6
síndrome de apnea del sueño, para el cual se prescribió meses, el paciente tuvo un aumento de peso de 6 kg y recidiva de
terapia con presión positiva continua en las vías respiratorias edema en las piernas. La presión arterial doméstica se mantuvo en
(CPAP). Las pruebas de laboratorio estándar se presentan en torno a 150/90 mmHg. Las pruebas de laboratorio mostraron
la Tabla 1. El valor de creatinina reflejó la tasa de filtración disminución de creatinina (1,27 mg / dl), ácido úrico (7,8 mg / dl). 7
glomerular estimada a 41 ml / min por 1,73 m2 utilizando la mg / dl) y triglicéridos (183 mg / dl), pero proteinuria persistente (787
ecuación de Colaboración en Epidemiología de la ERC [9]. Sin mg / 24 h). Se le indicó nuevamente al paciente que reanudara la
embargo, una recolección de orina de 24 h concomitante actividad física y redujera la ingesta de sodio. Se aumentó la
mostró hiperfitración renal (aclaramiento de creatinina: furosemida a 60 mg al día, se restableció la eplerenona y el paciente
148ml / min), sodio y potasio a 154 y 54 mg / 24 h, y proteína a acordó comenzar con amlodipino 5 mg dos veces al día (Tabla 1).
857 mg / 24 h, hallazgos que se corresponden con
hipertensión y obesidad de larga duración. La gammagrafía Durante los meses siguientes, se le hizo un seguimiento estrecho
renal mostró perfusión y función levemente reducidas, para asegurar el cumplimiento de los consejos de medicación y estilo
igualmente distribuidas en ambos riñones (50%), y ausencia de vida, con normalización de los valores de PA domiciliaria. A los 11
de isquemia renal. La ecografía renal mostró riñón de tamaño meses, informó una mejora adicional en la calidad del sueño y una
normal y diferenciación corticomedular, pero múltiples disminución de peso de 4 kg. Su PA se controló con niveles domésticos
'masas' bilaterales, parecidas a angiomiolipomas, de hasta 2,6 medios inferiores a 130/80 mmHg. Los análisis de laboratorio
cm. A raíz de nuestra solicitud de investigaciones previas, el mostraron creatinina a 1,01 mg / dl, triglicéridos normales (153 mg /
paciente presentó resultados de biopsia cutánea de las dl) y proteína en orina a 178 mg / 24 h, como era de esperar por la
marcas faciales, describiendo 'lesiones papilomatosas disminución de la PA (tabla 1). Una nueva ecografía renal mostró que
cutáneas, los angiomiolipomas renales aumentaron ligeramente. Una
En ese momento, el diagnóstico de CET se consideró altamente posible. Una consulta de neurología resonancia magnética confirmó el aumento de los tres
no reveló signos o síntomas neurológicos, mientras que una consulta de dermatología confirmó angiomiolipomas más grandes del riñón derecho (3,5, 2,5 y 2,1 cm,
angiofibromas faciales y reveló además fibromas ungueales, máculas hipomelanóticas y parche de respectivamente) y el desarrollo de cinco lesiones más pequeñas. Las
shagreen en el cuello, parte inferior y superior de la espalda (fig. 1a yb). De acuerdo con las guías actuales lesiones del riñón izquierdo se mantuvieron sin cambios. Con estos
para lesiones renales asociadas a CET [10], el paciente fue sometido a RM retroperitoneal, mostrando tres hallazgos, el cumplimiento del paciente y la mejora de los niveles de
angiomiolipomas en el riñón derecho de 18-32 mm y tres angiomiolipomas en el lóbulo inferior del riñón triglicéridos y proteínas en orina, se inició con everolimus 5 mg q. D.
izquierdo de hasta 12 mm de tamaño (fig. 1c). Una resonancia magnética cerebral mostró nódulos y focos se decidió detener la progresión de los angiomiolipomas.
subependimarios, junto con múltiples lesiones corticales y subcorticales con márgenes inespecíficos Aproximadamente 17 meses después de su primera visita, tiene PA
hiperintensivos en las imágenes ponderadas en T2 y en recuperación de inversión atenuada por líquido y controlada y función renal estable (Tabla 1). No tuvo efectos
menos lesiones de pequeño tamaño en la protuberancia. compatible con hipertensión de larga duración secundarios por eleverolimus, cuyos niveles se encontraron
(fig. 1d). Por lo tanto, el paciente cumplió con seis de los 11 criterios clínicos principales, lo que llevó a un repetidamente dentro de las recomendaciones (5-15metrog / l).
diagnóstico definitivo de CET. Una revisión de los tratamientos disponibles para el daño orgánico

relacionado con el CET, llevó a la decisión de no comenzar aún el tratamiento con everolimus, debido al
DISCUSIÓN
tamaño de los angiomiolipomas marginales y las múltiples comorbilidades (incluida la obesidad, la

hipertrigliceridemia y la macroalbuminuria). El paciente recibió nuevamente instrucciones detalladas para El CET es un trastorno genético caracterizado por tumores
reducir la ingesta dietética de sodio, seguir el ejercicio aeróbico y lograr la pérdida de peso. Una benignos en varios órganos y una amplia variabilidad fenotípica
derivación de su hija asintomática de 7 años al Departamento de Pediatría confirmó la ausencia de CET en [4]. Las manifestaciones renales son el segundo hallazgo más
la niña. llevó a la decisión de no iniciar todavía el tratamiento con everolimus, debido al tamaño marginal común, con angiomiolipomas en el 80% y quistes renales en el
de los angiomiolipomas y las múltiples comorbilidades (incluida la obesidad, la hipertrigliceridemia y la 50% de los pacientes [11]. Las complicaciones renales
macroalbuminuria). El paciente recibió nuevamente instrucciones detalladas para reducir la ingesta (insuficiencia renal, hemorragia o enfermedad metastásica)
dietética de sodio, seguir el ejercicio aeróbico y lograr la pérdida de peso. Una derivación de su hija afectan la calidad de vida y aumentan la morbilidad y la
asintomática de 7 años al Departamento de Pediatría confirmó la ausencia de CET en la niña. llevó a la mortalidad [12]. Tradicionalmente, el CET se identificaba en niños
decisión de no iniciar todavía el tratamiento con everolimus, debido al tamaño marginal de los con la tríada de Vogt: angiofibromas faciales, retraso mental y
angiomiolipomas y las múltiples comorbilidades (incluida la obesidad, la hipertrigliceridemia y la convulsiones. Sin embargo, esto ocurre sólo en el 29% de los
macroalbuminuria). El paciente recibió nuevamente instrucciones detalladas para reducir la ingesta pacientes, mientras que el 6% carece de los tres síntomas [13]. En
dietética de sodio, seguir el ejercicio aeróbico y lograr la pérdida de peso. Una derivación de su hija una cohorte de 243 pacientes, la edad del diagnóstico varió desde
asintomática de 7 años al Departamento de Pediatría confirmó la ausencia de CET en la niña. el nacimiento hasta los 73 años, porque el 39% de ellos informó
En la visita de los 3 meses, el paciente informó una mejora síntomas de CET o signos que no se habían detectado [14].
significativa en la calidad del sueño y menos episodios de apnea con

Revista de hipertensión www.jhypertension.com 2111


Copyright © 2017 Wolters Kluwer Health, Inc. Todos los derechos reservados.
Sarafidis et al.

FIGURA 1 (a) Fibromas unguales; (b) múltiples angiofibromas faciales y una mancha de piel de serpiente en el cuello; (c) resonancia magnética retroperitoneal que muestra angiomiolipomas renales
(flechas); (d) RM cerebral que muestra nódulos y focos subependimarios, múltiples lesiones corticales y subcorticales con márgenes inespecíficos hiperintensivos en T2W y FLAIR
imágenes ponderadas y menos lesiones de pequeño tamaño en la protuberancia.

Nuestro paciente había consultado repetidamente a dermatólogos por Nuestra paciente también tuvo hipertensión no controlada durante
fibromas faciales y tenía una biopsia de piel altamente sugestiva. muchos años, incluida preeclampsia durante su primer embarazo, y
de TSC a los 20 años, pero los resultados pasaron por alto la atención de ans. hallazgos que sugieren una posible causa secundaria (edad <30 años en el
Physici momento del diagnóstico, obesidad severa, trastornos del sueño).

2112 www.jhypertension.com Volumen 35 Número 10 de octubre de 2017

Copyright © 2017 Wolters Kluwer Health, Inc. Todos los derechos reservados.
Diagnóstico del complejo de esclerosis tuberosa durante la evaluación de la hipertensión

[16,17] pero nunca se evaluó por posible hipertensión secundaria o daño a órganos Las opciones incluyen cirugía con preservación de nefronas,
diana. De hecho, el paciente ni siquiera tenía el primer paso de las pruebas de embolización selectiva cuando el tamaño o la ubicación de los
diagnóstico de rutina para la hipertensión de nueva aparición, ni un seguimiento angiomiolipomas impiden la cirugía con preservación de nefronas,
estándar. Este caso destaca lecciones importantes sobre el abordaje diagnóstico de los nefrectomía completa cuando no se puede realizar una nefrectomía
pacientes hipertensos. De acuerdo con las guías de la ESH / ESC, se debe realizar un parcial o ablación por radiofrecuencia y crioablación cuando otras
conjunto estándar simple de pruebas hematológicas y bioquímicas (incluida una tira opciones son imposibles [8,24]. Hasta ahora, ningún ensayo clínico
reactiva de proteínas en la orina) y un ECG de 12 derivaciones en todas las personas que aleatorizado ha comparado estas diferentes intervenciones.
primero presentan un aumento de la PA. Sobre la base de la historia, el examen físico y Recientemente se puso a disposición un fármaco para los
las pruebas básicas, se deben realizar investigaciones adicionales para identificar el angiomiolipomas relacionados con el CET y se sugiere que sea la
daño del órgano diana y la hipertensión secundaria, incluida la proteína en orina de 24 opción de primera línea [2,10]. Everolimus es un inhibidor de mTOR,
horas, sodio y potasio, ecocardiograma y fundoscopia, mientras que la evaluación que se dirige a la causa del CET. Se une al receptor de la proteína de
adicional es un dominio de la especialista [18]. Además, los pacientes con hipertensión unión a FK-12, que interactúa directamente con la proteína del
verdaderamente resistente también deben ser evaluados cuidadosamente en busca de complejo mTOR-1, e inhibe la señalización de la vía de la mTOR debido
causas secundarias y daño a órganos diana [16,19]. Si nuestro paciente tuviera los pasos a la ausencia de hamartina y / o tuberina [12]. La eficacia de
de diagnóstico estándar (en la presentación o en el seguimiento de rutina de la everolimus para los angiomiolipomas renales asociados a CET se
hipertensión), podría surgir evidencia de creatinina elevada, proteína de tira reactiva e evaluó en el ensayo EXIST-2 [25], en el que 118 pacientes con CET o
hipertrofia del VI en el ECG, lo que desencadenaría una mayor investigación; sin linfangioleiomiomatosis esporádica y al menos un angiomiolipomas
embargo, esto no sucedió hasta que el paciente tuvo una emergencia hipertensiva con renales con un diámetro de al menos 3 cm se asignaron al azar a
edema agudo de pulmón que requirió hospitalización. eso desencadenaría una mayor everolimus o placebo. El criterio de valoración principal fue la tasa de
investigación; sin embargo, esto no sucedió hasta que el paciente tuvo una emergencia respuesta de angiomiolipomas renales, definida como una reducción
hipertensiva con edema agudo de pulmón que requirió hospitalización. eso de al menos un 50% en el volumen de todas las lesiones diana
desencadenaría una mayor investigación; sin embargo, esto no sucedió hasta que el evaluadas con resonancia magnética, y se logró en
paciente tuvo una emergencia hipertensiva con edema agudo de pulmón que requirió 41,8% de los pacientes tratados con everolimus pero ninguno con
hospitalización. placebo (P <0,0001).
En conclusión, el CET es un trastorno genético grave que presenta
En nuestras manos, una rápida implementación del segundo paso diversos síntomas y signos, de los que los médicos deben estar
diagnóstico de las Guías de la ESH / ESC reveló nefropatía hipertensiva atentos para realizar un diagnóstico precoz. Paralelamente, nunca
proteinúrica y retinopatía hipertensiva, sumadas a la cardiopatía debe omitirse el sencillo algoritmo diagnóstico propuesto para la
hipertensiva. Posteriormente, la resonancia magnética cerebral reveló evaluación de pacientes hipertensos; en nuestro paciente una rápida
lesiones compatibles con hipertensión no controlada de larga implementación de algunas pruebas estándar condujo al diagnóstico
duración. Además, los síntomas obvios del paciente y el IMC guiaron de CET a través de una sola ecografía renal, pero también de
un estudio del sueño y revelaron el síndrome de apnea del sueño, que nefropatía proteinúrica severa, retinopatía, miocardiopatía y síndrome
puede contribuir a la hipertensión resistente. de apnea del sueño, todos asociados con un aumento extremo de la
[20]. Más importante aún, una simple ecografía renal llevó a sospechar morbilidad y mortalidad cardiovascular en una mujer de 39 años y
el diagnóstico de CET perdido hace mucho tiempo. La ecografía renal todo revirtió en un grado importante a los pocos meses de un control
es una modalidad de imagenología no invasiva, fácilmente disponible adecuado de la PA.
y barata, que proporciona un amplio espectro de información
morfológica y establece el diagnóstico de muchas anomalías renales AGRADECIMIENTOS
(desde riñón atrófico hasta riñón hidronefrótico y desde cicatrices
El artículo actual no fue respaldado por ninguna fuente y
hasta enfermedad renal poliquística). La exploración Doppler de las
representa un esfuerzo original de los autores.
arterias renales puede ayudar a diagnosticar la hipertensión
renovascular, pero también a detectar la lesión renal en la Conflictos de interés
hipertensión de larga duración a través del índice de resistencia renal
No existen conflictos de intereses.
[21]. Así, puede contribuir a una mejor evaluación de los pacientes
hipertensos al identificar las causas secundarias que se originan en el REFERENCIAS
riñón y detectar la lesión renal. Por estas razones, los expertos 1. Bourneville DM. Sclerose tubereuse des circonvolutions cerebrales.
propusieron recientemente la ecografía renal color-Doppler como una Arch Neurol 1880; 1: 81–91.
prueba de rutina para pacientes hipertensos recién diagnosticados. 2. Northrup H, Krueger DA. Actualización de los criterios diagnósticos del complejo de
esclerosis tuberosa: recomendaciones de la Conferencia Internacional de Consenso
[21]. Nuestro caso de diagnóstico de CET en un paciente adulto
sobre el Complejo de Esclerosis Tuberosa de 2012.Pediatr Neurol 2013; 49: 243-254.
hipertenso respalda aún más esto, ya que, a pesar de la rareza del
CET, el diagnóstico precoz podría salvarle la vida. 3. Crino PB, Nathanson KL, Henske EP. El complejo de esclerosis tuberosa.
En pacientes con manifestaciones renales de CET, se requieren N Engl J Med 2006; 355: 1345-1356.
evaluaciones anuales de la función renal y el control de la PA, y la medición 4. Curatolo P, Bombardieri R, Jozwiak S. Esclerosis tuberosa. Lanceta 2008; 372:
657–668.
precisa de la PA es crucial, utilizando criterios específicos de edad para
5. Rakowski SK, Winterkorn EB, Paul E, Steele DJ, Halpern EF, Thiele EA.
niños [2]. La vigilancia de los angiomiolipomas renales en el CET incluye Manifestaciones renales del complejo de esclerosis tuberosa: incidencia,
resonancia magnética al menos una vez cada 3 años. [2,10]. En los pronóstico y factores predictivos.Riñón Int 2006; 70: 1777-1782.
angiomiolipomas asociados a CET se podría seguir previamente un enfoque 6. Salerno AE, Marsenic O, Meyers KE, Kaplan BS, Hellinger JC. Afectación
vascular en la esclerosis tuberosa.Pediatr Nephrol 2010; 25: 1555–
de observar y esperar. La resección quirúrgica o la embolia se eligió en
1561.
situaciones de alto riesgo para prevenir la hemorragia [22], ya que el riesgo 7. Wong H, Hadi M, Khoury T, Geary D, Rubin B, Filler G. Manejo de la
acumulado de hemorragia es del 18% para las mujeres y del 8% para los hipertensión severa en un niño con anomalías vasculares importantes
hombres [23]. Quirúrgico relacionadas con la esclerosis tuberosa. J hipertensos 2006; 24: 597–599.

Revista de hipertensión www.jhypertension.com 2113


Copyright © 2017 Wolters Kluwer Health, Inc. Todos los derechos reservados.
Sarafidis et al.

8. Eijkemans MJ, van der Wal W, Reijnders LJ, Roes KC, vanWaalwijk van Doorn- 17. Rimoldi SF, Scherrer U, Messerli FH. Hipertensión arterial secundaria:
Khosrovani SB, Pelletier C, et al. Seguimiento a largo plazo que evalúa los ¿cuándo, quién y cómo evaluar?Eur Heart J 2014; 35: 1245-1254.
patrones de tratamiento, la morbilidad y la mortalidad del angiomiolipoma 18. Mancia G, Fagard R, Narkiewicz K, Redon J, Zanchetti A, BohmM, et al.
renal: un estudio observacional en pacientes con complejo de esclerosis 2013 ESH / ESC Directrices para el manejo de la hipertensión arterial: el
tuberosa en los Países Bajos. Soy J Riñón Dis 2015; 66: 638–645. Grupo de Trabajo para el manejo de la hipertensión arterial de la Sociedad
9. Levey AS, Stevens LA, Schmid CH, Zhang YL, Castro AF 3rd, Feldman HI, et al. Europea de Hipertensión (ESH) y de la Sociedad Europea de Cardiología (ESC).
Una nueva ecuación para estimar la tasa de filtración glomerular. Ann Intern J hipertensos 2013; 31: 1281-1357.
Med 2009; 150: 604–612. 19. Sarafidis PA, Georgianos PI, Zebekakis PE. Epidemiología comparada de la
10. Krueger DA, Northrup H. Vigilancia y tratamiento del complejo de esclerosis hipertensión arterial resistente en la enfermedad renal crónica y la población
tuberosa: recomendaciones de la Conferencia de consenso internacional hipertensa en general.Semin Nephrol 2014; 34: 483–491.
sobre el complejo de esclerosis tuberosa de 2012. Pediatr Neurol 2013; 49: 20. Parati G, Ochoa JE, Bilo G, Mattaliano P, Salvi P, Kario K, Lombardi C. Síndrome
255-265. de apnea obstructiva del sueño como causa de hipertensión resistente.
11. Dixon BP, Hulbert JC, Bissler JJ. Enfermedad renal compleja de esclerosis Hipertensos Res 2014; 37: 601–613.
tuberosa.Nephron Exp Nephrol 2011; 118: e15 – e20. 21. Tsioufis C, Andrikou I, PruijmM, Ponte B, Sarafidis PA, Koureas A, et al.
12. Budde K, Gaedeke J. Angiomiolipomas asociados al complejo de esclerosis ¿Debería ser la ecografía Doppler color renal una prueba de rutina en pacientes
tuberosa: se centran en la inhibición de mTOR. Soy J Riñón Dis 2012; 59: hipertensos recién diagnosticados? J hipertensos 2017; (en prensa).
276-283. 22. Siroky BJ, Yin H, Bissler JJ. Conocimientos clínicos y moleculares sobre la enfermedad
13. Schwartz RA, Fernandez G, Kotulska K, Jozwiak S. Complejo de esclerosis renal compleja de la esclerosis tuberosa.Pediatr Nephrol 2011; 26: 839–852.
tuberosa: avances en el diagnóstico, la genética y el tratamiento. J Am Acad 23. Bissler JJ, Kingswood JC. Angiomiolipomas renales.Riñón Int 2004; 66: 924–934.
Dermatol 2007; 57: 189-202.
14. Staley BA, Vail EA, Thiele EA. Complejo de esclerosis tuberosa: desafíos diagnósticos, 24. Staehler M, Sauter M, Helck A, Linsenmaier U, Weber L, Mayer K, Fischereder
síntomas de presentación y signos que comúnmente se pasan por alto. M. Resección del angiomiolipoma con conservación de nefrona después del
Pediatría 2011; 127: e117 – e125. pretratamiento con sirolimus en pacientes con esclerosis tuberosa. Int Urol
15. Seibert D, Hong CH, Takeuchi F, Olsen C, Hathaway O, Moss J, Darling TN. Nephrol 2012; 44: 1657-1661.
Reconocimiento de la esclerosis tuberosa en mujeres adultas: presentación 25. Bissler JJ, Kingswood JC, Radzikowska E, Zonnenberg BA, Frost M, Belousova E,
tardía con consecuencias potencialmente mortales.Ann Intern Med 2011; et al. Everolimus para el angiomiolipoma asociado con el complejo de
154: 806–813; W-294. esclerosis tuberosa o linfangioleiomiomatosis esporádica (EXIST-2): un
16. Sarafidis PA. Epidemiología de la hipertensión resistente.J Clin Hypertens ensayo multicéntrico, aleatorizado, doble ciego y controlado con placebo.
(Greenwich) 2011; 13: 523-528. Lanceta 2013; 381: 817–824.

2114 www.jhypertension.com Volumen 35 Número 10 de octubre de 2017

Copyright © 2017 Wolters Kluwer Health, Inc. Todos los derechos reservados.
UNIVERSIDAD DE GUAYAQUIL
FACULTAD DE CIENCIAS MEDICAS
CARRERA DE MEDICINA

NEFROLOGÍA

NOVENO SEMESTRE

DOCENTE:

DR. SERRANO FIGUEROA LUIS ALBERTO

GRUPO: 5

NEFROPATIA DIABETICA

ESTUDIANTES:

NATHALY ANNABELLA VACA MOREIRA

LISBETH EMILY YANEZ ENCALADA


NEFROPATÍA DIABÉTICA
Toda lesión renal por causa de la diabetes
Son lesiones microangiopáticas: cuando esta presente la proteinuria ya suele estar presenta la
retinopatía y la neuropatía. Esta presente desde que se diagnosticó la diabetes .

ESTADIO 1 :
- 0 a 5 años .
-Hiperfiltrado glomerular( 130ml/min).
- Aumento del flujo sanguíneo a nivel renal
-Hipertrofia renal ( aumento de la silueta renal en la ecografía).
ESTADIO 2 :
-Albuminuria moderada intermitente , antes llamada microalbuminuria.
- Algunos autores no distinguen este estadio.
ESTADIO 3 :
-Nefropatia incipiente : albumina de grado moderado .
- (30-300 mg/24h, 30-300 mg/g en cociente albúmina/creatinina en micción aislada) persistente
en reposo.
-Para confirmar la presencia de albuminuria debemos detectarla en 2-3 muestras recogidas en un
periodo de 3-6 meses.
- Es consecuencia del daño glomerular producido tanto por el depósito de glucosa, como por la
hiperfiltración secundaria a la activación del SRAA.
-Esta fase dura entre 10-15 años
-HTA por activación del SRAA
-El FG ira descendiendo hasta llegar a ND establecida.
ESTADIO IV

• Nefropatia establecida
• Proteinuria franca : mayor de 300 mg en 24 h
• Puede llegar aparecer SX nefrótico (>3,5 g/1,73/ 24 h).
• HTA constante
• Deterioro progresivo de la función renal.
ESTADIO V
-IRC terminal.
-25 a 30 años después de la aparición de la DM.
ANATOMIA PATOLÓGICA
GLOMERULOESCLEROSIS DIFUSA GLOMERULOESCLEROSIS NODULAR
• (+) MBG. ▪ Aparecen en las áreas de mayor cumulo de
• Acumulación de matriz mesangial matriz mesangial. Depósitos hialinos
( expansión del mesangio). nodulares , PAS (+).

• Patron mas frecuente y el primero en No es patognomónica


aparecer
Puede aparecer también en enfermedad de
cadenas ligeras.
Nódulos de kimmestiel wilson

LESIONES TUBULOINTERSTICIALES:
Las células de Armani-Ebstein, PAS+, aparecen a nivel tubular y son patognomónicas. Que se
producen por depósitos de glucógeno en las células epiteliales tubulares a nivel de la pars recta
del tubulo contorneado proximal y asa de Henle).
La patogénesis de la ND , puede estar explicada por la glicación no enzimática de proteínas .
Siendo esta uno de los principales mecanismos de lesión glomerular.
▪ Los productos terminales de la glicación avanzada pueden unirse a los grupos aminos de otras
proteínas .

Su acumulación en el colágeno y otras proteínas de la matriz como la elastina, pueden :


▪ Disminuir la adhesión y replicación de células endoteliales
▪ Producir vasoconstricción por células mesangiales
▪ Incrementar la adhesión de lipoproteínas y complejos inmunes a monocitos y macrófagos.

MECANISMOS DE DAÑO RENAL EN LA DIABETES.


1. Hiperglicemia
2. Producción de AGEs
3. Activacion de los sistemas vasoactivos : SRAA y el sistema de las endotelinas.
4. NOX: especies de oxigeno reactivo derivadas de NADPH oxidasas.

PRINCIPALES CAMBIOS HISTOLÓGICOS:


- Expansión mesangial por la hiperglicemia o por la producción aumentada de la matriz o
de la glicación de las proteínas de la matriz.
- Engrosamiento de la MBG
- Esclerosis por Hipertensión intraglomerular por : dilatación de la arteria renal aferente o
por lesión isquémica de la arteria renal aferente , inducida por el estrechamiento hialino
de la misma.

OTRAS LESIONES GLOMERULARES:

• Llamadas exudativas: gota capsular y hialinosis glomerular.


• Gota capsular: Deposito homogéneo, hialino en la capsula de Bowman, suele ser
redondeado o elongada y es altamente sugestivo de ND.
• Hialinosis glomerular: resulta de la extravasación de componentes plasmáticos que se
acumulan en segmentos periféricos del penacho capilar
MICROSCOPIA ELECTRÓNICA:

• Incremento de la matriz extracelular


• MBG engrosada difusamente de aspecto fibrilar y parece laminada
INMUNOFLUORESCENCIA:

• Depositos lineales de IgG y albuina en las paredes capilares glomerulaes y en menor


medida basales tubulars y capsula de Bowman.
TRATAMIENTO
Control de la tensión arterial: se usa IECA y ARAII como primera elección, se debe empezar
por un IECA de 24 horas de acción, reservando el ARAII en caso de intolerancia o efectos
adversos. Como segundo y tercer hipotensor se aconseja un calcio antagonista y un diurético.
Control de la proteinuria: recomienda aumentar la dosis de inhibidor del SRAA (IECA o
ARAII) en dosis máximas para lograr el control de proteinuria.
Control glucémico: en los casos de ND dependerá de la FG, si la FG es menor de 30 ml/minuto
se deberá utilizar insulina subcutánea para lograr el control glucémico, estando contraindicados
la mayoría de los antidiabéticos orales.
Dislipidemia: las estatinas no precisan ajuste de dosis en general en la IR leve-moderada, Los
fibratos se pueden utilizar ajustando la dosis en IR, excepto cuando el FG es menor de 30
ml/minuto.
Hiperuricemia: Se recomienda alopurinol 100 mg inicialmente y aumentar la dosis según
respuesta y tolerancia.
BIBLIOGRAFÍA :

• Manual AMIR nefrología 14va Edición.


• Afectación renal en diabetes y otras enfermedades metabólicas: hiperuricemia, oxalosis
(https://kidneypathology.com/Diabetes_y_otras.html). Nefropatía diabética.
UNIVERSIDAD DE GUAYAQUIL
CÁTEDRA DE NEFROLOGÍA
DR. LUIS SERRANO

GRUPO 5

NEFROPROTECCIÓN

INTEGRANTES
SHEYLA PALMA
CRISTINA PALMA
DANIXA MOSQUERA
Nefroprotección

Conjunto de medidas conducidas a:


▪ Impedir el desarrollo de la enfermedad renal en las poblaciones de
riesgo.
▪ Retardar el daño renal en el sujeto que la presenta.
▪ Limitar las complicaciones en el paciente con tratamiento de diálisis

Primarias Secundarias. Terciarias.

• Acciones dirigidas a • Diagnóstico precoz en • Limitar el daño renal, y


impedir el desarrollo de poblaciones de riesgo. cardiovascular en
la enfermedad renal a • Medidas empleadas pacientes que estén en
través de la promoción para retardar. diálisis.
de la salud y manejo de
factores de riesgo.

http://aps.isss.gob.sv/profesional/publicaciones/Medidas%20nefroprotectoras%20para%20cuidado%20renal
ERC y clasificación de acuerdo a la TFG

La ERC se define como la


disminución de la función renal,
expresada por un FG inferior a
60ml/min/1,73 m2, o como la
presencia de daño renal de forma
persistente durante al menos tres
meses.

http://aps.isss.gob.sv/profesional/publicaciones/Medidas%20nefroprotectoras%20para%20cuidado%20renal
Causas de ERC

http://aps.isss.gob.sv/profesional/publicaciones/Medidas%20nefroprotectoras%20para%20cuidado%20renal
Epidemiología

http://aps.isss.gob.sv/profesional/publicaciones/Medidas%20nefroprotectoras%20para%20cuidado%20renal
Estrategias de mejora de la ERC

• Bajo peso al • Desnutrición, • IECA • Educación en

Nefroprotección
Modificación de la comorbilidad.

Prevención de las complicaciones urémicas.

Preparación para la terapia sustitutiva.


nacer. anemia. • Antagonistas unidades de
• Enfermedad Enfermedad de los atención.
cardiovascular. renal ósea, receptores de • Inicio
• Enfermedad trastornos angiotensina. oportuno del
vascular electrolíticos, • Control de la tto.
periférica. agua, presión arterial • Trasplante
equilibrio renal en
• Restricción
ácido base. prediálisis.
proteica.
• Uso precavido
de medios de
contraste.
• Identificación
de tóxicos.

http://aps.isss.gob.sv/profesional/publicaciones/Medidas%20nefroprotectoras%20para%20cuidado%20renal
Objetivos de la
Nefroprotección

http://aps.isss.gob.sv/profesional/publicaciones/Medidas%20nefroprotectoras%20para%20cuidado%20renal
Factores que podrían intervenir en la evolución de la historia natural de la ERC

http://aps.isss.gob.sv/profesional/publicaciones/Medidas%20nefroprotectoras%20para%20cuidado%20renal
Recomendaciones para la prescripción de medicamentos.

• Acumulación y efectos adversos


• Medir función renal antes y despues de la preescripción de medicamentos
• Leve deterioro de función renal en personas de edad avanzada
• Otros factores de riesgo (DM, IC, deshidratación, hipovolemia)

✓ Evitar la hiperpotasemia asociada a fármacos (diurético ahorrador de K con otro que retenga K
(IECA, ARA II))
✓ Dar los fármacos necesarios durante el tiempo necesario.
✓ Si se require un inicio rápido del efecto de un medicamento utilizer la dosis inicial habitual
para lograr un nivel adecuado y ajustar la dosis de mantenimiento
✓ Fármacos uso habitual que con o sin ajuste pueden causar nefrotoxicidad (aminoglucósidos,
inmunosupresores, contrastes radiológicos)

http://aps.isss.gob.sv/profesional/publicaciones/Medidas%20nefroprotectoras%20para%20cuidado%20renal
Nefroprotección en diabetes mellitus

Scully, T. Diabetes en cifras. Nature 485, S2-S3 (2012). https://doi.org/10.1038/485S2a


Estrategias de Nefroprotección en nefropatía diabética

Nefropatía diabética

▪ Complicación microvascular y se presenta tanto en la diabetes mellitus tipo 1


(T1DM) como en la diabetes mellitus tipo 2 (T2DM).

El trastorno se presenta con:


▪ Presión sanguínea elevada
▪ Disminución progresiva de la tasa de filtración glomerular (TFG)
▪ Albuminuria persistente (mayor de 300 mg / d) en al menos dos visitas con un
intervalo de 3 a 6 meses.

▪ Tratamiento temprano puede retrasar o prevenir la progresión del trastorno

Varghese RT, Jialal I. Nefropatía diabética. [Actualizado el 19 de noviembre de 2020]. En: StatPearls [Internet]. Treasure Island (FL): StatPearls Publishing; 2020 Ene. Disponible en:
https://www.ncbi.nlm.nih.gov/books/NBK534200/
Factores de progresión de la ERC en pacientes diabéticos

▪ Seguimiento estricto con creatinina


sérica, TFG y microalbuminuria.
▪ Cada 6 meses para los estadios 1 y
2, cada 4 meses para el estadio 3, y
cada 2 a 3 meses para el estadio 4.

Hormonales

• Hiperglucemia e insulinopenia.
• Aumento de glucagón y hormona del crecimiento

Metabólicos.

• Presencia de PGA.
• Aumento de concentraciones de ácido orgánico.

Otros

• Ingesta proteínica elevada.


• Expansión del volumen extracelular.

http://aps.isss.gob.sv/profesional/publicaciones/Medidas%20nefroprotectoras%20para%20cuidado%20renal
Estrategias de Nefroprotección en nefropatía diabética

Tratamiento

Prevención primaria.
• Control de la hiperglucemia: dieta,
medicamentos.
Prevención secundaria.
• Control de factores hemodinámicos y
metabólicos.
Estrategias de Nefroprotección en nefropatía diabética

http://aps.isss.gob.sv/profesional/publicaciones/Medidas%20nefroprotectoras%20para%20cuidado%20renal
Estrategias de Nefroprotección en nefropatía diabética.
ARA II

Losartán se asoció a una menor tasa de


insuficiencia renal y menos muerte por
enfermedad cardiovascular asociada a
enfermedad renal.

http://aps.isss.gob.sv/profesional/publicaciones/Medidas%20nefroprotectoras%20para%20cuidado%20renal
Estrategias de Nefroprotección en nefropatía diabética.
IECA

Estudio ESCAPE

Controlaron de forma muy


estricta la presión arterial, si
se controla ésta se disminuye
el riesgo de progresión de
daño renal.

http://aps.isss.gob.sv/profesional/publicaciones/Medidas%20nefroprotectoras%20para%20cuidado%20renal
Estrategias de Nefroprotección en nefropatía diabética.
IECA

Si damos un manejo multifaltorial:


• Control de proteinuria
• De la presión
• De la glicemia
• De colesterol
Es la única manera de frenar el riesgo
de daño renal.

http://aps.isss.gob.sv/profesional/publicaciones/Medidas%20nefroprotectoras%20para%20cuidado%20renal
Estrategias de Nefroprotección en nefropatía diabética.
IECA

▪ Una combinación de verapamilo y


trandolapril (IECA) se va a lograr
una reducción de la
microalbuminuria.
▪ No es el verapamilo el que lo logra.
▪ IECA es el tto de elección para
frenar la proteinuria en pcts
diabéticos

http://aps.isss.gob.sv/profesional/publicaciones/Medidas%20nefroprotectoras%20para%20cuidado%20renal
Estrategias de Nefroprotección en nefropatía diabética.
ARA II

ESTUDIO ROADMAP

▪ Estudio en adultos donde evaluaron un


antagonista de receptores de
angiotensina, éste era olmersartán y se
observo que en los pacientes diabéticos
la microalbuminuria se puede
controlar.
▪ Menos progresión a daño renal.

http://aps.isss.gob.sv/profesional/publicaciones/Medidas%20nefroprotectoras%20para%20cuidado%20renal
Estrategias de Nefroprotección en nefropatía diabética.
SRAA

ESTUDIO AVOID CON ALISKIREN


▪ Es el inhibidor de la renina
▪ Mostro de una manera importante la reducción
de microalbuminuria pero no de la presión
arterial.

http://aps.isss.gob.sv/profesional/publicaciones/Medidas%20nefroprotectoras%20para%20cuidado%20renal
Estrategias de Nefroprotección en nefropatía diabética.
Combinación IECA y ARA II

▪ Telmisartan a otros ramipril y a otros una


combinación de telvisartan y ramipril.
▪ Ahora se bloquean los dos, los resultados
son que la pcte se incremente la creatinina,
que ingrese a diálisis o que fallezca.
▪ La combinación aumenta el riesgo de
daño renal, incrementa la probabilidad de
ingresar a diálisis.
▪ Si disminuyó la proteinuria.

http://aps.isss.gob.sv/profesional/publicaciones/Medidas%20nefroprotectoras%20para%20cuidado%20renal
Estrategias de Nefroprotección en nefropatía diabética.
INHIBIDORES DE LA SGLT2

http://aps.isss.gob.sv/profesional/publicaciones/Medidas%20nefroprotectoras%20para%20cuidado%20renal
Efectos antihiperglucemiante de la inhibición del SGLT2 con empagliflozina

Eficacia
antihiperglucemiante.
• De glucemia. Eficacia reducida cuando la
• De glucosa plasmática de ayuno. TFG cae por debajo de 45
• Hemoglobina glucosilada (0,78 en ml/min
monoterapia y 0,82 en
combinación).

Preserva la función de la Disminuye el peso corporal


célula Beta. entre 1,6-3,2 kg.

http://aps.isss.gob.sv/profesional/publicaciones/Medidas%20nefroprotectoras%20para%20cuidado%20renal
Uso de inhibidores del SGLT2 y su impacto cardiovascular.

Tratamiento de 12
Tratamiento durante 8
semanas, las dosis de
semanas: efectos
empaglifozina de 10 y
positivos sobre la
de 25 mg disminuyen la
rigidez arterial
PAS de 4 a 5 mmHg.

http://aps.isss.gob.sv/profesional/publicaciones/Medidas%20nefroprotectoras%20para%20cuidado%20renal
Estrategias de Nefroprotección en nefropatía diabética

▪ Alto efecto nefroprotector.


EMPAGLIFOZINA ▪ Primer antihiperglucemiante que ha
logrado una reducción importante de
los eventos cardiovasculares y renales

MECANISMOS POTENCIALES
✓ Efectos diuréticos,
✓ Efectos sobre la vasculatura
periférica
✓ Mejora el metabolismo cardiaco
✓ Supresión del SNS
✓ Reducción en arritmias

EMPA-REG OUTCOME
Nefroprotección en nefropatía hipertensiva.

DEFINICIÓN

▪ Es la Enfermedad Renal
Crónica causada por
Hipertensión Arterial,
casi siempre esencial,
usualmente
acompañada de
Proteinuria.

Fisiopatología de
nefropatía hipertensiva
Objetivos en nefropatía hipertensiva

La medida más importante en Nefroprotección en Hipertensión Arterial es:

CONTROLAR LA PRESION ARTERIAL CON CUALQUIER ANTIHIPERTENSIVO

Objetivos:
✓ TA menor de 125/75 si la proteinuria es mayor a 1gr
✓ TA menor a 130/80 si la proteinuria es menor a 1gr

OTRAS MEDIDAS??

Disminución de la Proteinuria.
Disminución de la Hipertensión Intraglomerular
Tratamiento en nefropatía hipertensiva

IECA, ARA II en pacientes con Proteinuria mayor que 500


mg/día

Control similar de la PA con otros antihipertensivos, los


IECAs y ARA II han demostrado ser eficaces en Retardar la
Progresión de la ERC Proteinúrica

EFECTO ANTIFIBRÓTICO Y EN LA PERMEABILIDAD


SELECTIVA GLOMERULAR que contribuye al
enlentecimiento en la progresión de la ERC
MEDICAMENTOS NEFROTOXICOS

Factores de riesgo de nefrotoxicidad por


Causa común de lesión medicamentos.
aguda
Edad avanzada >60 años Diabetes

Insuficiencia renal basal Enfermedad cardiovascular


Deshidratación sepsis
Considerar la
nefrotoxicidad de los Hipovolemia Exposición a más de un
fármacos fármaco nefrotóxico

En pacientes de riesgo
se debe conocer la
función renal
MECANISMOS PATOGENICOS DE LA NEFROTOXICIDAD

De la capacidad del riñón de regular la


ALTERACION DE LA AINES presión intraglomerular
HEMODINAMICA IECA,ARA II De la TFG
INTRAGLOMERULAR

• Incremento de su concentración tubular


• Secundario a la reabsorción de liquido
• Afectando al túbulo proximal
TOXICIDAD DE
CELULAS TUBULARES

aminoglucósidos, anfotericina B, antirretrovirales


(adefovir, cidofovir y tenofovir), medios de
contraste
MECANISMOS PATOGENICOS DE LA NEFROTOXICIDAD
DESARROLLO DE PROCESOS INFLAMATORIOS

Glomerulonefritis Nefritis intersticial aguda Nefritis intersticial crónica

• Mecanismos de tipo • Reacción de tipo • Ciclosporina, tacrolimus, litio,


inmunológico tras la idiosincrásica AINES
administración de • Como reacción alérgica a un
determinados fármacos. medicamento
• Sales de oro, hidralazina, • Antibióticos, antivirales,
interferon, litio, fenitoína y diuréticos, AINES, fenitoína
pamidronato
MECANISMOS PATOGENICOS DE LA NEFROTOXICIDAD

• Precipitación de cristales insolubles en el interior de la luz tubular.


NEFROPATIA • Se obstruye el flujo de orina y produce la alteración renal
CRISTALINA • Depende de la concentración del fármaco en la orina y del pH
• Antibióticos, antivirales, metrotexato

• La lesión del músculo esquelético a nivel de los miocitos puede ocasionar una lesión
renal secundaria
RABDOMIOLISIS • Por acción de la mioglobina, obstrucción tubular y alteraciones de la filtración
glomerular
Estatinas

• Lesión renal secundaria mediada por reacción inmune o por toxicidad directa sobre el
MICROANGIOPA endotelio
TIA
TROMBOTICA • Antiagregantes plaquetarios (clopidrogrel)
• Ciclosporina - Mitomicina C - Quinina
MEDIDAS PREVENTIVAS
Tabla 3. grupos farmacológicos que requieren ajuste
Mantener una ingesta de líquidos de dosis
adecuada Hipoglucemiantes Digoxina
Antibióticos Analgésicos

Evaluar la función renal basal del Antifúngicos Antineoplásicos


paciente y ajustar la dosis de Antivirales Hipolipemiantes
medicamento
Antihipertensivos Antiagregantes plaquetarios

Antipsicóticos Antidepresivos
Monitorizar la actividad renal del
paciente Antigotosos Anticonvulsivantes

Evitar el uso de combinaciones de


fármacos nefrotóxicos
HIPOGLUCEMIANTES
METFORMINA

Contraindicada si FG < 60 ml/min

Monitorizar anualmente la función renal por el riesgo de acidosis láctica

Evaluar la función renal antes de iniciar el tratamiento y en pacientes con factores


de riesgo de deterioro de la función renal

Considerar su uso como razonablemente seguro si FG > 45 ml/min/1,73 m2.

Reducir la dosis al 50% si FG 30-45 ml/min/1,73 m2.

No utilizarla si FG < 30 ml/min/1,73 m2.

Suspender temporalmente su administración ante circunstancias que pongan en


riesgo la función renal
HIPOGLUCEMIANTES

SULFONILUREAS

Se recomienda el uso de gliclazida, glipizida o gliquidona

En caso de utilizarlas, se recomienda el uso de gliclazida, glipizida o


gliquidona

Limitar el uso de glimepirida, ajustando la dosis, a pacientes con FG >


60 ml/min/1,73 m2 .

No se recomienda utilizar glibenclamida.


HIPOGLUCEMIANTES
INSULINA

● La ERC se asocia a resistencia a la insulina


● Siendo posible incluso la interrupción del tratamiento insulínico en algunos pacientes con ERC
avanzada
● El tratamiento con insulina en pacientes con ERC requiere una monitorización estrecha de la
glucemia.

Si FG > 60 ml/min/1,73 m2 : no se precisa ajuste de la dosis.


En general, no se
requieren ajustes de dosis
en FG > 60 ml/min/1,73
Si FG 20- 60 ml/min/1,73 m2 : reducir la dosis aproximadamente m2
un 25%.

Si FG < 20 ml/min/1,73 m2 : reducir la dosis un 50%.


ANTIHIPERTENSIVOS
DIURÉTICOS
Diuréticos tiazídicos: se pueden utilizar a dosis convencionales si FG > 30 ml/min/1,73 m2

Diuréticos de asa (furosemida, torasemida): no requieren ajuste de dosis en ningún estadio de IR.

Ahorradores de potasio (amiloride, triamtereno): riesgo aumentado de hiperpotasemia.

Antagonistas de la aldosterona (espironolactona, eplerenona): ambos están contraindicados si


FG < 30 ml/min

Espironolactona: en IR leve la dosis habitual es de 25 mg/día

Eplerenona: si FG 30-60 ml/min iniciar el tratamiento con 25 mg en días alternos

Se recomienda la monitorización periódica de la creatinina y el potasio sérico y ajustar las dosis en


función de los mismos
ANTIHIPERTENSIVOS

BETABLOQUEANTES

Recomendaciones en IR grave:

Los de eliminación renal Atenolol: dosis de 50 mg/día o 100 mg en días


Los de metabolismo como acebutolol, alternos.
hepático como atenolol, bisoprolol,
carvedilol, metoprolol, celiprolol, nadolol, Bisoprolol: no sobrepasar la dosis diaria de 10 mg.
propranolol y labetalol nebivolol y sotalol, en
no requieren ajuste de general no requieren
dosis. ajuste de dosis en IR leve Nevibolol: no se recomienda su uso.
a moderada.
HIPOLIPEMIANTES
ESTATINAS

SIMVASTATINA,
ATORVASTATINA LOVASTATINA Y PRAVASTATINA ROSUVASTATINA, PITAVASTATINA
FLUVASTATINA

Si FG < 30 ml/min, Está contraindicada en Utilizar con precaución


considerar IR grave a cualquier ya que los datos
cuidadosamente la dosis. En IR moderada disponibles en IR de
No requiere ajuste de En IR moderada-grave
administración de (FG < 60 ml/min) se cualquier grado son
dosis en ningún se recomiendan dosis
dosis superiores a 10 recomienda una dosis muy limitados. La dosis
estadio de la IR. iniciales de 10 mg/día.
mg/día, 20 mg/día y de inicio de 5 mg y está de 4 mg no está
40 mg/día contraindicada la dosis recomendada en los
respectivamente. de 40 mg. pacientes con IR grave.
ANTICOAGULANTES
DABIGATRÁN:

ANTICOAGULANTES • Evaluar la función renal antes de iniciar el tratamiento con el fin de


ORALES. excluir a pacientes de FG < 30 ml/min, en los que está
contraindicado.
Acenocumar RIVAROXABÁN:
ol no
requiere • Reducir la dosis a 15 mg/24 h si FG 15-50 ml/min. Contraindicado si
FG < 15 ml/min
ajuste de
dosis. APIXABÁN:

• Reducir la dosis a 2,5 mg/12 h si FG < 30 ml/min. Contraindicado si


FG < 15 ml/min

HEPARINAS DE BAJO PESO MOLECULAR (HBPM)

• En general, es necesario ajustar la dosis si FG < 30 ml/min, siendo


apropiada la reducción de dosis (enoxaparina: 20 mg/día, nadroparina:
reducir la dosis un 25-33%)
ANTIGOTOSOS E HIPOURICEMIANTES

ALOPURINOL: COLCHICINA: FEBUXOSTAT:


No requiere ajuste de
En la ERC se recomienda Si FG < 50 ml/min,
dosis si FG > 30 ml/min. Si
iniciar con una dosis disminuir la dosis a la
FG < 30 ml/min, se
máxima de 100 mg/día e mitad y/o incrementar los
produce cierta
incrementar solo si la intervalos entre las tomas.
acumulación, pero no se
respuesta no es Contraindicado si FG < 30
ha evaluado bien la
satisfactoria. ml/min.
eficacia y la seguridad.
En ERC grave, puede ser
aconsejable utilizar menos
de 100 mg/día o usar
dosis únicas de 100 mg a
intervalos mayores de un
día.
ANTIBIOTICOS

Amoxicilina y amoxi-clavulánico: dosis


Amoxicilina-clavulánico a dosis alta
habitual hasta FG 30 ml/min. Si FG
(1.000/62,5 mg): dosis habitual hasta Cefixima: si FG < 20 ml/min reducir la
10-30 ml/min: máximo 500 mg/12h.
FG 30 ml/min. Uso no recomendado dosis a la mitad.
Si FG < 10 ml/min: máximo 500
si FG < 30 ml/min.
mg/24h.

Ciprofloxacino: si FG 30-60
Cefuroxima-axetilo: si FG < 20
Claritromicina: si FG < 30 ml/min, la ml/min/1,73 m2 , dosis de 250-500
ml/min, administrar una única dosis
dosis debe reducirse a la mitad. mg/12 h. Si FG < 30 ml/min/1,73 m2 ,
al día.
dosis de 250- 500 mg/ 24 h.

• Fluconazol: dosis de mantenimiento del 50% si FG < 50 ml/min.


ANTIFÚNGICOS • Itraconazol: hay datos limitados de su uso en la ERC

• Aciclovir requiere ajuste si FG < 25 ml/min.


• Valaciclovir requiere ajuste si FG < 50 ml/min.
ANTIVIRALES • Famciclovir requiere ajuste si FG < 60 ml/min.
ANALGÉSICOS Y ANTIINFLAMATORIOS

Evitar el uso
• Se debe evitar el uso innecesario de AINE en ERC. innecesario de
AINE.

• En caso de ser necesarios, seleccionar preferiblemente AINE de vida media corta y AINE por el riesgo
durante pocos días.
• Todos los AINE están contraindicados en ERC grave (y diclofenaco también en ERC
de deterioro de la
moderada). función renal
• En caso de utilizar ibuprofeno en ERC leve-moderada, se recomienda disminuir la dosis

PARACETAMOL.

• Se puede utilizar a dosis de 500-650


mg (como alternativa a AINE) • Codeína: se debe usar con precaución en la ERC, y evitarla en la ERC grave. Si
ANALGÉSICOS
OPIOIDES
• Si FG < 50 ml/min: 500-650 mg/6 h. FG 10-50 ml/min, reducir la dosis al 75%.
• Si FG < 10 ml/min: 500-650 mg/8 h. • Tramadol liberación inmediata: si FG < 30 ml/min prolongar el intervalo de
dosificación a 12 h. Contraindicado su uso si FG < 10 ml/min.
• Tramadol liberación prolongada: si FG < 30 ml/min no se recomienda su uso.
Contraindicado si FG < 10 ml/min.
• Morfina de liberación inmediata: relativamente segura si se titula la dosis
progresivamente. Evitar su uso si FG < 30 ml/min.
• Morfina de liberación prolongada: evitar su uso en la ERC.
ESTRATEGIAS DE PROTECCION RENAL

IDENTIFICACION DE RIESGO PERIOPERATORIO

• Identificar a paciente en riesgo de desarrollar DRA


• La ERC es el >factor de riesgo para DRA

FACTORES CLÍNICOS FACTORES SOCIODERMOGRAFICOS


• diabetes mellitus • Edad >60 años
• hipertensión • Exposición a quicos o
• enfermedad cardiovascular condiciones medioambientales
• obesidad neurotóxicas
• enfermedades autoinmunes • Bajo nivel socioeconómico
• enfermedades sistémicas
• exposición a nefrotóxicos
ESTRATEGIAS DE PROTECCION RENAL

OPTIMIZACIÓN HEMODINAMICA

• Mantener el volumen intravascular


• Gasto cardiaco
• PAM

Periodos de hipotensión tan cortos como 5 y 10 min por debajo


de presiones arteriales medias de 55 mmHg, cursan con DRA

Los pacientes con DRA inducido por hipotensión pueden


requerir una PAM de 70-80 mmHg más que la usual de 65
mmHg para prevenir DRA.

Tener protocolos de atención para el manejo del paciente con


inestabilidad hemodinámica más que el tipo de monitorización
que se utilice
ESTRATEGIAS DE PROTECCION RENAL

PREVENCION DEL DAÑO RENAL

Administración de Utilización de Realizar estudio


menor medio de contraste hemodinámico un
volumen de medio isotónico o día antes de la
de contraste hipotónico cirugía del paciente

Iniciar
Mantenerlo
administración de
euvolemico e
liquido 24 horas
hidratado
antes del estudio
ESTRATEGIAS DE PROTECCION RENAL

PROTECCION DEL RIÑON FRENTE A DROGAS NEFROTOXICAS

Mantener euvolemia, Aminuglucósidos cada


Infusión prolongada de
gasto cardiaco y 24 horas, vancomicina
aciclovir y sulfas
presión arterial media en infusión continua

Suspender los
medicamentos Dosis altas con
Evitar el uso
antihipertensivos en el intervalos grandes en
concomitante de otros
postoperatorio la administración de
nefrotóxicos
inmediato de estos colistina
pacientes

Evitar el uso de agentes


antiinflamatorios no
Monitorizar los niveles esteroideos así́ como los
séricos inhibidores IECAS y
bloqueadores de
receptores de angiotensina
Medios de contraste

Son solutos no reabsorbibles de alta, baja


o isoosmolalidad. osmolaridad
Diatriazoato,
alrededor de 1000–2500 iotalamato,
Primera generación mOsm / kg (en metrizoato,
comparación con el ioxitalamato.
plasma (290 mOsm / kg)
Todos los medios de contraste son más
viscosos que el plasma sanguíneo. Ioxaglato,
iopromuro,
alrededor 400–800 iopamidol,
Segunda generación
mOsm/kg iohexol,
ioversol
Su viscosidad aumenta exponencialmente yodixanol y el
con su concentración molar y también Tercera generación 300 mosm/L iotrolan
depende de la concentración de yodo.
Nefropatia inducida por medios de contraste

Tercera causa de IRA en pacientes


hospitalizados

Se menciona al daño renal inducido por Se menciona al daño renal inducido por
contraste a la manifestación por elevación contraste a la manifestación por elevación
de las cifras de creatinina a más del 25% a de las cifras de creatinina al incremento de
tres días de la exposición del Medio. 1mg/dl en 48 hrs.
Nefropatia inducida por medios de contraste

Factores de riesgo : La Sociedad Europea de Paciente Medio de contraste


Radiología Urogenital los divide en factores Edad mayor a 60 años Administración del medio de contraste vía
asociados con el paciente y factores asociados con el intraarterial o intrarrenal.
medio de contraste. Administración del medio de contraste de
Deshidratación
alta osmolaridad.

Enfermedad renal crónica preexistente o


Administración de un volumen mayor de 4
pacientes con una tasa de filtración
cc/kg o más de 350 cc.
glomerular (TFG) <60 mL/min/1.73m2

Infarto agudo de miocardio reciente (<24 Realización de múltiples estudios en


horas) menos de 72 horas.

Uso concomitante de fármacos


nefrotóxicos

Hipoalbuminemia

Trasplante renal
Fisiopatología

Nefrotoxicidad directa del medio de Efectos hemodinámicos indirectos,


contraste al epitelio de las células caracterizados por una lesión
tubulares isquémica
Nefrotoxicidad directa del medio de contraste al epitelio de las células tubulares

pérdida de la polaridad de la célula lo que resulta en un


transporte anormal de iones
1.- Redistribución de la
bomba Na+/K+ ATPasa
aumenta de la entrega de Na+ y Cl- a las asas de
Henle y túbulos distales

2.- obstrucción tubular


Efectos hemodinámicos indirectos, caracterizados por una lesión isquémica

Medio de Disminución de
vasoconstricción
contraste la perfusión
Prevención de nefropatía inducida por contraste
Medidas no farmacológicas
• Medio de contraste
• Hemodiálisis y hemofiltración ?
• Suspender fármacos nefrotóxicos no esenciales

Medidas farmacológicas
• Expansión de volumen
• N-acetilcisteína
• Estatinas
• Ácido ascórbico, Teofilina, Dopamina ?
Medidas no farmacológicas
Medidas no farmacológicas

Estudios indican que la hemodiálisis profiláctica no tuvo ningún


➢ Hemodiálisis y hemofiltración
beneficio; por el contrario, la hemofiltración por 24 h
posprocedimiento sí redujo la incidencia de NIC. Sin embargo,
según las guías KDIGO, no son recomendados en la práctica
clínica.

➢ Suspender fármacos nefrotóxicos no esenciales

Muchos de los medicamentos utilizados de rutina tienen


excreción renal y potencial de nefrotoxicidad, entre los cuales se
destacan los antiinflamatorios no esteroideos/inhibidoresde la
ciclooxigenasa 2, aminoglucósidos, diuréticos de asa
Medidas farmacológicas
Medidas farmacológicas

Estatinas
• Efectos pleiotrópicos se encuentran la limpieza de especies
reactiva de oxigeno (ROS)
• Efectos antiinflamatorios y antitrombóticos.

• Estimulan la producción de óxido nítrico y han demostrado


efectos antiapoptóticos de células tubulares renales

• Inducen disminución en el número de receptores de


angiotensina y disminuyen la síntesis de endotelina
1,modulando así la hipoperfusión renal.
Caso clínico

Female patient name is Rashida khan 63 year old religion by Muslim lived in the yawatmal. She is a house wife but she
used to do work in her own farm and lived in joined family her husband and son is the breadwinner of the family, rashida
done her education in class 8 Th and her monthly family income is around 8000 per month. The source of health care that
is government hospital in yavatmal. She was admitted in the AVBRH with the chief complaint of breathing difficulty,
uremia, fever, nausea and vomiting, swelling on both legs, lethargy weakness since 10 days before she came to the
hospital she is admitted in yavatmal, she can suddenly unconscious in the house so her relative take her in yavatmal
hospital then doctor freshly diagnose as aDM 2 and she also having the past history of HTN. She doesn’t have any other
medical history in past and she done her family planning (tubal ligation) other than she not having any type of surgical
history.

Indian Journal of Forensic Medicine & Toxicology, January-March 2021, Vol. 15, No. 1
Nursing Asessment In physical examination the patient showing a poor condition, presence of distress, undernourished,
body build thin, activity is dull, conscious and oriented to things. Showing vitals like heart rate of 94 beats per minute,
breathing rate of 16 breaths per minute, saturation of 96%, blood pressure that is increase, 140/90 mmHg and she
having a fever- 100 F and her body is warm. She having pain in both the lower leg, swelling present on both leg, she
had a hemodialysis catheter present over neck. The other physical examination not showing any abnormalities.
Thesome raising and decrease laboratory finding showing below:
· HB%- 10.0 g/dl
· WBC- 15000cu/mm
· Blood Urea – 56.0mgdl
· Bun-26.17 mg/dl
· Creatinine- 5.0 mg/dl
· Sodium – 132 mmo/l
· Potassium – 4.0 mmo/l
DRUGS:
Pantoprazole 40mg- IV-OD
Piperacillin /tazobactam 4.5g - IV - BID
Furosemide - 50mg -IV -TID
Ondansetron - 4g- IV -BD
Insulin - 15mg P/O -BD
Cyanocobalamin - 1amp IV -OD
Nitrofurantoin - 100mg P/O –BD
Hemodialysis: she started her heamodialysis cycle as came to hospital

Indian Journal of Forensic Medicine & Toxicology, January-March 2021, Vol. 15, No. 1
Discusión
• Present case report showing that the patient those having the history HTN
and it could not control because of not aware about disease condition,
adherence of medication, not control over diet and sedentary life style
which lead to the DM 2 and this Chronic Kidney Disease of the patient.
Then treatment include the drug therapies, control sign and symptoms,
Hemodialysis, kidney transplant, patient counseling, prevent complication,
life style modification and taking regular follow up.With the help of drug
patient need to hemodialysis for recovering the condition of kidney and
managing the sigh and symptoms in this disease condition, also reducing
the complication which may lead to further. There is need to patient
education about her condition and treatment modalities.

Indian Journal of Forensic Medicine & Toxicology, January-March 2021, Vol. 15, No. 1
• All over the world the people present with some degree of kidney disease that is 1 in
10 people. The CKD can develop mostly in elder people but it can develop at any age
and there so many condition leads to develop CKD. It can be very common with
people increasing their age, nearby 1 % function of the kidney can affect by the age
over the 40 year. Such as condition like hypertension, diabetes mellitus type 2, heart
diseases can start affecting on the kidney after ageing as compare to young people. In
developing countries over the 600 million population they do not afford the surgeries
like renal replacement so it can be leads to increase the mortality rate in kidney
patients. Although 80 % of people who get the renal replacement therapy can be
alive in some developed countries. As we see in our country there is less than 10 % of
people we get to need the renal replacement therapy and they live with the get new
kidney, the WKD can spreading massage about the importance of the health of
kidney.8 Studies shown that there is the one in five men and one in four women that
age of 65-75 year, the elder people come in risk factor to getting the kidney injury as
compare to younger people. The health of kidney is very necessary because if the
kidney is damaged so it can lead to heart attack, stroke and other life threatening
condition. In treatment of CKD some cases are required dialysis and renal
replacement therapy. The reducing the risk factor and life modification and early
detection and treatment that is the way to reducing the mortality and morbidity rate
in kidney diseases.

Indian Journal of Forensic Medicine & Toxicology, January-March 2021, Vol. 15, No. 1
GRACIAS
BIBLIOGRAFÍA

http://aps.isss.gob.sv/profesional/publicaciones/Medidas%20nefropro
tectoras%20para%20cuidado%20renal
CASO CLÍNICO:
Indian Journal of Forensic Medicine & Toxicology, January-March 2021, Vol. 15, No. 1
RESUMEN DE UROPATIA OBSTRUCTIVA

Es un acontecimiento que provoca daño funcional y/o parenquimatoso renal secundario a la obstrucción del tracto urinario en
cualquier sector de su longitud siendo su origen mecánico o funcional. Las entidades que conducen a la misma varían según la edad y
el género del paciente de estos cuadros de micción disfuncional. Aunque existen factores intrínsecos como la herencia, edad y el sexo
asociado de la formación de litos y factores extrínsecos como la geografía, el clima, la ingesta de alimentos o bebidas (dieta), la
ocupación e inclusive los efectos de fármacos y enfermedades como la mielitis aguda que producen incremento de la presión arterial
con disminución del filtrado glomerular que conducirían potencialmente a la insuficiencia renal aguda como crónica sino es tratado a
tiempo.

Donde fisiopatológicamente, son varios los mecanismos que llevan al compromiso de la función renal en este disturbio donde hay
cambios glomerulares, vasculares y hemodinámicos; como aumento de la presión intraluminal, dilatación ureteral y en consecuencia
peristalsis ureteral inefectiva y oposición a la presión neta de ultra filtración glomerular, vasoconstricción intrarenal con la
consiguiente reducción en el flujo sanguíneo glomerular, liberación de sustancias quimiotácticas que atraen monocitos y macrófagos
que liberan localmente proteasas y radicales libres. Durante una uro-obstrucción puede además desencadenarse hipertensión arterial
(vasocontricción-hipervolemia), hiperkalemia, acidosis metabólica (resistencia a la aldosterona) y diabetes insípida (resistencia a la
vasopresina).

Su histopatología hay evidencia de lesión tubular después de la obstrucción hay presencia de varias moléculas inflamatorias que
median estos cambios intersticiales, con activación de los bucles paracrinos y autocrinos; en el caso de una uropatía obstructiva del
tracto urinario inferior podemos categorizar: en congénita que se da en infancia cuya etiología se da por estenosis meatal valvas
uretrales posteriores, afectación de las raíces S2 a S4 (espina bífida o mielomeningocele), reflujo vesiculouretreales (estasis vesicales
y renales) y adquirida que presenta disfunción neurogénica, elongación y acodadura del uréter como en la mujer que se da en el
embarazo, en presencia de tumores ginecológicos y en el hombre se da en el cáncer de próstata, en la hipertrofia de próstata y en la
estenosis de uretra. De acuerdo a la naturaleza de la obstrucción podemos evidenciar infección. En el proceso obstructivo hay
cambios como aumento de la presión en los túbulos para vencer la resistencia, al disminuir la tasa de filtrado glomerular, entra a la
fase de compensación donde hay aumento sanguíneo renal este proceso de vasodilatación dura 90 minutos, en el momento de la caída
de flujo renal y elevación de presión ureteral dentro de las 24 horas, se manifiesta la caída de ambas funciones.

La clasificación según la causa que puede ser mecánica (congénita, inflamatoria, traumática, tumoral) y funcional (vaciamiento
deficiente como en la aperistalsis y mala contracción vesical o vejiga neurogénica, por reflujo y en casos de polidipsia psicógena). En
la clasificación según la localización puede afectar al cáliz e infundíbulo, en las uniones uretero pélvico, uretero lumbar, uretero
pélvico, uretero vesical, en el cuello vesical y en la uretra

Según la presentación clínica de la patología tenemos una fase aguda y otra crónica, de acuerdo al grado de obstrucción es completa e
incompleta, y dependiendo su localización es infravesical o supravesical.

En la afectación del tracto superior puede darse comúnmente en un estadio agudo por la obstrucción donde hay oliguria o anuria con
evidencia de 3 fases de compensación: la primera fase se da en los primeros 90 minutos donde hay aumento del flujo sanguíneo renal
que corresponde de 50 a 70 milímetro de mercurio, en la segunda fase se evidencia a partir de los 90 minutos a 5 horas donde hay
disminución del flujo sanguíneo y de la presión de filtración glomerular, pero con aumento la presión uretral.

En la fase crónica hay evidencia no solo de engrosamiento o colapso de la pared vascular sino de esclerosis global que producirá
como efecto isquemia crónica dado por el aplanamiento de papilas, dilatación y luego atrofia de los túbulos, disminución del espesor
medular, cambios en el glomérulos, y la presencia de la proteína de Tamm-Horstal; periodo en el cual ya hay se connotado la
disminución del flujo sanguíneo renal, la reducción del índice de filtración glomerular, la disminución de la capacidad de
concentración urinaria y la disminución de la depuración de hidrogeniones.

Entre las manifestaciones clínicas hay dolor en el flanco con irradiación por estenosis del área afectada, fuerza y calibre disminuido
de orina, goteos terminales, hematuria, disuria, escalofríos, nausea, debilidad, palidez, turbiedad de la orina, tenesmo y anuria;

El diagnostico se da por medio de imagen como urotac, eco, radiografía simple de abdomen, urografía excretora, cistografía
retrograda, urografía retrograda, entre las más comunes y en ocasiones maniobras semiológicas.

El tratamiento es disminuir la presión intravesical de manera que se eviten los daños locales del aparato urinario superior la cual
puede ser por medio de nefrotomía o ureterotomía, uso de sondas vesicales, cateteres doble J y litotricia cuyo procedimiento utiliza
ondas de choque para desintegrar cálculos de gran tamaño en el riñón y partes del uréter para que puedan ser eliminados por estas
vías. Y en caso de infecciones el uso de antibióticos.

El pronóstico dependerá del lugar, la intensidad y duración de la obstrucción. Sobre todo, la rapidez y eficacia con que se establece
un drenaje completo en donde haya o presencia o ausencia de infección y valorando si es uni o bilateral la presentación clínica en el
paciente.
UNIVERSIDAD DE GUAYAQUIL
FACULTAD DE CIENCIAS MÉDICAS
CARRERA DE MEDICINA

CÁTEDRA DE NEFROLOGÍA
DR. SERRANO FIGUEROA LUIS ALBERTO
GRUPO #5

TEMA:
INSUFICIENCIA RENAL AGUDA

SUBGRUPO #8
INTEGRANTES:
• PÉREZ GOYA IRANIA ABIGAIL
• POMA ORTIZ DIANA ELISA
• PUPIALES HERNÁNDEZ PABLO ANDRÉS
• QUINAPALLO BRAVO KEVIN ANDRÉS

NOVENO SEMESTRE
CICLO I
2021-2022
INSUFICIENCIA RENAL AGUDA
Se la define como un deterioro brusco de la función renal originando la acumulación de urea y creatinina
y otros desechos que en circunstancias normales son eliminados por los riñones, instaurada en horas,
días y semanas (hasta 3 meses).
EPIDEMIOLOGÍA
La IRA en Latinoamérica tiene una incidencia que varía entre los 2.000 y los 15.000 pacientes/millón
de habitantes/año de acuerdo con las diferentes publicaciones. El 70% es de origen prerrenales, el 25%
es de origen prerrenal y el 5% es de origen postrenal. Es más frecuente en hombres que en mujeres
en relación de 2:1.
CLASIFICACIÓN
Clasificación Anatómica
Prerrenal Renal Postrenal u obstructiva
• Disminución de TFG • Si la causa que provoca la • Una obstrucción al flujo
• Incapacidad para excretar los hipoperfusión renal se urinario repercute en las
productos nitrogenados prolonga o es severa, funciones normales del riñón y
• Si se corrige, recupera la desencadena un daño (NTA) y puede llegar a provocar anuria
funcion renal normal, al requiere días o semanas para si es bilateral
contrario se inicia NTA o IRA recuperar la función • Es reversible y retorna la
establecida • Causas de IRA intrínseca: función renal normal al
inmunológicas, vasculitis, NIA, corregirse la causa o facilitar
fármacos, enfermedad la salida de orina mediante
ateroebólica sondaje, cateterización o
nefrostomía

Clasificación Según el Volumen de Orina

• IRA oligúrica, si la diuresis es <400ml/24h, se inicia dentro de las primeras horas siguientes a
la lesión que produce la NTA, puede durar de 1 a 2 semanas.
• IRA no oligúrica, si la diuresis es >400ml/24h, la urea y creatinina aumentan a pesar de que
existe disminución en el volumen urinario a 400ml/día.

DIAGNÓSTICO DE IRA
DIAGNÓSTICO DIFERENCIAL
Las NTA de origen hemodinámico-isquémico se diferencian del FRA prerrenal en que la hipoperfusión
renal es suficientemente grave como para dañar las células del epitelio tubular, y el FRA no se resuelve
con la reposición de volumen eficaz.
Las causas que conducen a la azotemia prerrenal-pérdida de volumen, bajo gasto cardíaco,
redistribución interna de fluidos, y vasodilatación extrema pueden al perpetuarse desembocar en una
NTA isquémica.
Puede desarrollarse NTA nefrotóxica ante la exposición a muy diversos fármacos, tóxicos y compuestos
endógenos, ya que el riñón es especialmente vulnerable por la riqueza de su vascularización y su
capacidad de concentración de los tóxicos en el túbulo. Está demostrada la acción sinérgica entre la
hipoperfusión renal y la exposición a ciertos nefrotóxicos, como los aminoglucósidos, la vancomicina,
la ciclosporina, la anfotericina o los contrastes yodados.
Las causas que conllevan a un FRA obstructivo difieren según la edad. En la infancia predominan las
anomalías anatómicas, en la edad adulta, la litiasis y en pacientes mayores las causas son la
hiperplasia prostática y las neoplasias pelvianas: vejiga, próstata, útero y recto. El FRA obstructivo es
más prevalente en pacientes de edad avanzada. El origen de la obstrucción puede ser intraluminal,
intramural o extraluminal.
TRATAMIENTO
El FRA, desde el punto de vista terapéutico, y con la excepción de las técnicas de diálisis, es en muchas
ocasiones frustrante para el nefrólogo. La complejidad de su fisiopatología limita las opciones
terapéuticas y obliga, frecuentemente, a esperar a que la función renal se recupere espontáneamente.
Además, a pesar de la mejora de las técnicas dialíticas, la mortalidad del FRA sigue siendo muy
elevada. Los fármacos vasoactivos, frecuentemente eficaces en animales de experimentación, no han
demostrado ser útiles para prevenir la aparición de FRA ante determinadas agresiones tóxicas o
isquémicas. Existen datos contradictorios acerca del efecto de la dopamina en el tratamiento o
prevención del FRA.
Aspectos prácticos del tratamiento
Ante un paciente con una disminución de la diuresis y/o un aumento progresivo y rápido de las
concentraciones séricas de productos nitrogenados. La primera, que debe ser automática, consiste en
la supresión inmediata de todos los fármacos potencialmente nefrotóxicos que el paciente estaba
recibiendo.
La dieta es un elemento fundamental en el tratamiento conservador del FRA. La elección de una dieta
adecuada tiene innegables ventajas. En los pacientes con FRA, e independientemente de la vía de
nutrición empleada, el aporte de fluidos no debe superar en más de 500 ml las pérdidas totales del
organismo.
En segundo lugar, las dietas que se administren deben constar del menor componente electrolítico
posible, de forma que se puedan hacer los ajustes necesarios en todo momento.
En tercer lugar, es muy importante recordar que, salvo en raras excepciones, los pacientes con FRA
no deben recibir suplementos de potasio. Lo más probable es que muestren una tendencia excesiva a
la hiperpotasemia.
Tratamiento del FRA establecido mediante técnicas especiales

Antes del desarrollo de las técnicas de diálisis y ultrafiltración, la mayoría de los pacientes con FRA
morían de uremia, hipercaliemia, sobrecarga de volumen o diátesis hemorrágica. En la actualidad, se
dispone de tres grupos de técnicas de depuración extrarrenal (TDE) para el tratamiento del FRA: la
diálisis peritoneal (DP), la hemodiálisis intermitente (HDI) y la hemofiltración (HF) o hemodiafiltración
(HDF) continuas.
BIBLIOGRAFÍA

1. Gainza, F. (2020, 2 junio). Insuficiencia Renal Aguda | Nefrología al día. Nefrología al día.
https://www.nefrologiaaldia.org/es-articulo-insuficiencia-renal-aguda-317

2. Diaz, M; Briones, J; Carrillo, R & Moreno, A. (2017). Insuficiencia renal aguda (IRA)
clasificación, fisiopatología, histopatología, cuadro clínico diagnóstico y tratamiento una
versión lógica. Revista Mexicana de Anestesiología, 40(4), 280-287.
https://www.medigraphic.com/pdfs/rma/cma-2017/cma174e.pdf

3. Lombi, F., Varela, C. F., Martínez, R., Greloni, G., Campolo Girard, V., & Rosa Diez, G.
(2017). Lesión renal aguda en Latinoamérica en la era del big data. Nefrología, 37(5), 461-
464. https://doi.org/10.1016/j.nefro.2017.03.010

4. Tenorio Cañamás, M. T., Galeano Álvarez, C., Rodríguez Mendiola, N., & Liaño García,
F. (2010). Diagnóstico diferencial de la insuficiencia renal aguda. Nefrología, 3(2), 16-32.

5. L. Hernando Avendaño. Nefrología clínica. Editorial Medica Panamericana; 3º edición.


Capítulo 17.

6. Geoffroy S, Fremery A, Lambert Y, Marty C, Elenga N. Case report: Acute kidney failure
due to massive envenomation of a two-year-old child caused by killer bee stings. Am J
Trop Med Hyg [Internet]. 2021; Disponible en: http://dx.doi.org/10.4269/ajtmh.20-1276
(Caso clínico)
UNIVERSIDAD DE GUAYAQUIL
FACULTAD DE CIENCIAS MÉDICAS
CARRERA DE MEDICINA

TEMA: SINDROME NEFROTICO

ESTUDIANTE:
JOSELYN ZULLAY AREVALO MORALES

CATEDRA:
NEFROLOGÍA

NIVEL:
9NO SEMESTRE

GRUPO:
5

2021-2022 CI
SÍNDROME NEFRÓTICO
ETIOLOGÍA Y CUADRO CLÍNICO

Etiología del síndrome nefrótico: Glomerulonefritis primarias

Nefropatías con cambios mínimos, glomeruloesclerosis focal, nefropatía membranosa, Glomeruloesclerosis


mesangiocapilar

Etiología del síndrome nefrótico: Enfermedades Glomerulares secundarias

LES, DM II, Amiloidosis, AINE, Mercurio, Captopril, entre otras infecciones como la sifilis, hepatitis B y
C, Paludismo.

FISIOPATOLOGÍA

Alteración de la barrera de filtración glomerular, que condiciona la pérdida de proteínas por la orina
(proteinuria) y, como consecuencia, la hipoalbuminemia y el resto de las alteraciones del síndrome nefrótico
generando una serie de signos y síntomas hasta patologías que ponen en riesgo la vida del paciente.

Hipoproteinemia. - (Menor 6g/dl) con hipoalbuminemia (menor de 3g/dl). Existe aumento de α-2 y β-
globulinas. Descenso de albúmina, α-1 y gammaglobulinas (IgG). Existe tendencia a la infección por esta
hipogammaglobulinemia con déficit de IgG y factor B del complemento y por tendencia a la linfopenia. La
IgA y la IgM pueden estar aumentadas. Suelen aparecer infecciones por gérmenes capsulados, siendo
característica y frecuente la peritonitis espontánea por neumococo.

Edema. - Son blandos y dejan fóvea. Aparecen en párpados y zonas declives (miembros inferiores).
Pueden acompañarse de derrame pleural, pericárdico o peritoneal, o incluso llegar al grado de anasarca.
Puede ser causada por la hipoalbuminuria que causa la activación del SRAA o por daño asociado al túbulo
colector cortical.

Hiperlipidemia mixta. - Consiste en hipercolesterolemia (>250 mg/dl), con aumento de LDL y VLDL y
en ocasiones descenso de HDL, e hipertrigliceridemia. La hiperlipidemia condiciona la aparición de
aterosclerosis

Hipercoagulabilidad. -. Las trombosis arteriales son más frecuentes en las arterias femoral y pulmonar.

Hipovolemia e insuficiencia renal aguda. - Episodios de hipovolemia, por hipoalbuminemia grave o


diuréticos, que genera necrosis tubular aguda de origen isquémico

PROTEINURIA. - Barrera de filtración glomerular, formada por células endoteliales glomerulares


fenestradas, la membrana basal del glomérulo (MBG) y los pedicelos de los podocitos (células epiteliales
viscerales glomerulares), representa una gran restricción al paso transcapilar de proteínas en relación con
su tamaño, forma y carga eléctrica. En la limitación por tamaño tiene un papel primordial la existencia de
poros entre los pedicelos de los podocitos, cerrados por una membrana delgada (slit diaphragm), que
evitarían el paso de macromoléculas de tamaño superior a 150 kDa. Diversas proteínas como la nefrina y la
podocina, junto con proteínas del citoesqueleto, son constituyentes fundamentales de estos poros. (Rozman,
2016)

La hipoproteinemia se la puede expresar como una disminución significativa de varias proteínas en la


sangre, entre estas se pueden encontrar a:

• Gammaglobulinas
• Antitrombina III y diversos factores de la coagulación
• Numerosas proteínas transportadoras, ocasionando déficits nutricionales. Globulina ligadora de
tiroxina, proteína transportadora del cortisol, proteína transportadora de vitamina D3, transferrina,
ceruloplasmina y proteína transportadora de cinc.
• Albúmina

Bibliografía
AMIR. (2019). AMIR Manual de Nefrologia . España: ACADEMIA DE ESTUDIOS MIR.

Rozman, F. (2016). Medicina interna de Farreras Rozman. España: ELSEVIER.


SINDROME
NEFRÓTICO
JOSELYN AREVALO M.
ANA B ANEGAS
SINDROME NEFRÓTICO

Definición

Patognomónico de enfermedad
glomerular

Aumento de la permeabilidad
glomerular

Proteinuria masiva e hipoalbuminemia,


edema, hiperlipemia y lipiduria
SINDROME NEFRÓTICO

Etiología
SINDROME NEFRÓTICO

Etiología
SINDROME
NEFRÓTICO
SINDROME NEFRÓTICO
Conclusión Aumento del perfil
lipídico Ateroesclerosis

Proteinuria
Inmunodepresión
Trombos
Protein Hipocalcemia
Edema
uria Hiperparatiroidismo
Hipotiroidismo
Anemia

SRAA
SINDROME
NEFRÓTICO
EVOLUCI ON CLINICA
INICIAL
ANA BANEGAS
DIAGNOSTICO

• Se debe intentar filiar l a


caus a del s índrome
nefrótico para poder
iniciar un tratamiento
específico.

• Conocer la gr avedad y l as
reperc usiones del
síndrome nefrótico .

• La historia clínic a y la
exploración físic a
cuidados as permi ten
orientar en muc has
ocasiones el diagnóstico
etiológico.
Si tras la esta
evaluación inicial no se
descubre la causa del
síndrome nefrótico,
estaría indicada la
biopsia renal, por las
implicaciones tanto
pronósticas como
terapéuticas que
representa.
DATOS ANALITICOS

Se define SN ante una proteinuria superior a 3,5 g/24 h en el


adulto o más de 40 mg/m2 /24 h en el niño, junto con un valor
sérico de albúmina inferior a 3 g/dl.

El filtrado glomerular es generalmente normal, aunque en edades


avanzadas puede aparecer insuficiencia renal oligúrica,
determinada por la hipovolemia, que se produce ante la pérdida
brusca e importante de proteínas, y favorecida por el edema en el
intersticio renal.

Diversas alteraciones en factores de la coagulación, fibrinógeno y


plaquetas son responsables, junto a la hipovolemia, de problemas
trombóticos durante los períodos de actividad del SN.
MEDIDAS HIGIENICO DIETETICAS
• Restriccion de la ingesta de sodio (2 a
4gr/dias)
• Restriccion de la ingesta de agua en
caso de anasarca
TRATAMIENTO • Medidas posturales
• Vendas elasticas de compression

DIURETICOS
• Del ASA en dosis crecientes
Independientemente de la EDEMA DE SX • Asociar tiacidas si la respuesta a los
causa del síndrome nefrótico NEFROTICO diureticos del ASA es insuficiente
y de su tratamiento específico No suele tener especial • Vigilar cuidadosamente los valores
si lo hubiere, existe un trascendencia clínica. Sólo sericos del K
tratamiento general mediante ocasionalmente puede ser • Vigilar la aparicion de alcalosis
medidas higiénicas, dietéticas grave y acompañarse de metabolica
y farmacológicas, que derrame pleural y
pretende disminuir la pericárdico. MEDIDAS PARA DISMINUIR LA
morbimortalidad de las PROTEINURIA
complicaciones asociadas a
este síndrome. ANASARCA REFRACTARIA E
INCAPACITANTE
• Considerar la administracion IV de
diureticos
• Hemofiltracion continua o ultrafiltracion
mediante maquinas de hemodialisis.
HIPOPROTEINEMIA

No se recomiendan dietas hiperproteicas, ya que no mejoran la hipoalbuminemia y, además, inducen hiperfiltración


glomerular, pudiendo agravarse la lesión renal.

Las dietas hipoproteicas tienen un efecto favorable sobre los lípidos y sobre la proteinuria, pero tienen un riesgo
elevado de favorecer la malnutrición a medio o largo plazo, por lo que no se prescriben habitualmente, salvo que
se garantice un seguimiento muy estrecho del balance nitrogenado.

Otras medidas muy útiles para disminuir de forma no específica la proteinuria

Antiinflamatorios no Inhibidores de la enzima Antagonistas de los Inmunoadsorción


esteroideos conversora de angiotensina receptores de la con columnas de
Disminuyen la Eficaces para disminuir la angiotensina II proteína A
proteinuria en mayor proteinuria asociada a las El efecto Permite disminuir de
cuantía que la explicada enfermedades glomerulares, tanto antiproteinúrico es forma transitoria la
por los en nefropatía menos intenso que el proteinuria en
efectos hemodinámicos diabética, como en otras obtenido con los IECA. varios tipos de
sobre la vascularización nefropatías glomerulares, incluso enfermedades
glomerular. en pacientes normotensos. glomerulares.
• Las dietas pobres en colesterol son aconsejables, aunque de limitada
eficacia. Igualmente se recomienda ejercicio razonable y reducción de
peso si el paciente es obeso.
• En la mayoría de los pacientes es necesario el tratamiento
TRATAMIENTO DE LA HIPERLIPEMIA farmacológico, siendo de elección en monoterapia los inhibidores de
la hidroximetilglutaril-coenzima A-reductasa.
• Se debe disminuir su dosis en al menos un 50%, ya que su fracción
libre aumenta en situaciones de hipoalbuminemia, favoreciéndose la
aparición de efectos secundarios a veces graves.

Ante cualquier episodio de trombosis se debe iniciar


tratamiento con heparina, seguida de anticoagulación oral
prolongada TRATAMIENTO DE LAS
COMPLICACIONES
Se han comunicado algunos casos de trombosisagudas TROMBÓTICAS
bilaterales de la vena renal tratadas con éxito mediante
trombólisis con estreptocinasa ourocinasa
➢ Entre un 70-80% de casos presentan un curso
clínico caracterizado por repetidas remisiones y
recaídas espontáneas o desencadenadas por
procesos infecciosos y/o alérgicos.

➢ En la edad infantil, el 20-25% de los casos no


presentan recaídas, el 20-25% tienen recaídas
infrecuentes, y el resto evolucionan
➢ como corticodependientes con recaídas frecuentes.
PRONÓSTICO ➢ La morbilidad está en relación con procesos
infecciosos, peritonitis, sepsis, celulitis o neumonía,
junto con problemas derivados de complicaciones
trombóticas.

➢ La mortalidad es excepcional en la edad pediátrica,


menor del 2% entre 5-10 años de seguimiento y
secundaria a procesos infecciosos.

➢ En el adulto varía según la edad entre el 6-10% y


en el anciano es del 35-40% durante los tres
primeros años de evolución.
Caso Clínico

80-year-old patient, long-standing hypertensive, well controlled and type 2 diabetic for 25 years,
on treatment with oral antidiabetic drugs due to an allergic reaction to various types of insulin,
with poor metabolic control and non-proliferative diabetic retinopathy

Among other personal antecedents, he presented asymptomatic hyperuricemia without


treatment, iron deficiency anemia in treatment with oral iron, normal functioning multinodular
goiter and complete arrhythmia due to atrial fibrillation of several years and in treatment with
oral anticoagulation, with several episodes of heart failure.

He continues treatment with apixaban, metformin / vildagliptin, glycazide, furosemide, diltiazem,


losartan, omeprazole, simvastatin and ferroglycine sulfate.

The patient is admitted to the Internal Medicine service (MI) in July 2014 due to a clinical picture
compatible with viral gastroenteritis and oliguria. On examination, she is normotensive and
presents bilateral perimalleolar pitting edema. Laboratory tests (Table 1) show a deterioration in
renal function (plasma creatinine [Crp]: 3.4 mg / dl) which is classified as prerenal, partially
improving with fluid therapy up to Crp 1.7 mg / dl.
It should be noted that prior to admission the patient had normal renal function (Crp: 0.7 mg / dl;
estimated glomerular filtration rate [Modification of Diet in Renal Disease, MDRD-4] > 60 ml / min
/ 1.73 m2) without urinary changes (elemental and sediment [EyS]: normal; albuminuria / Cr
ratio: 2.5 mg / g).

In the rest of the complementary tests (PC) on admission, hypertriglyceridemia (227 mg / dl),
hypoproteinemia (5.2 g / dl) and hypoalbuminemia (2.8 g / dl), iron deficiency anemia
(hemoglobin [Hb]: 10 , 7 g / dl; MCV: 76 fL; ferritin: 39 ng / ml) and proteinogram without
monoclonal band. The EyS showed: proteins ++++, signs of hemoglobin, 1-3 red blood cells /
field and 4-8 leukocytes / field, with a urine sodium of 34 mmol / l. Renal Doppler ultrasound
showed no abnormalities. After clinical and analytical improvement, the patient is discharged

Fifteen days later, he went back to the emergency room due to general syndrome, dyspnea,
edema and oliguria in the last week, with diarrhea and rectal bleeding. Upon re-examination, she
admits habitual ibuprofen consumption (600 mg every 8-12 hours for the last three months) and is
admitted to Nephrology. On physical examination, she presented a blood pressure of 168/76
mmHg, jugular engorgement, lung auscultation with rhonchi, basal wheezing and crackles, and
pitting edema to the knees.
In PC, Crp: 4.9 mg / dl, proteins: 4.7 g / dl, albumin: 2.3 g / dl, cholesterol: 242 mg / dl,
triglycerides: 208 mg / dl, Hb: 11, 7 g / dl, leukocytes: 8700 /, lymphocytes: 14.2%, segmented:
69.2%, eosinophils: 5.1%, EyS: proteins ++, Hb ++, 15-20 red cells / field, 80- 100 leukocytes /
fields, moderate bacteriuria, urine sodium: 28 mEq / L, proteinuria in 24 hours > 10 g, negative
Bence Jones proteinuria, normal immunoglobulins and complement, ANA, ANCA, anti-GBM,
rheumatoid factor, anti-PLA2R and negative viral serologies

During admission, a progressive worsening of renal function is observed up to Crp 6 mg / dl,


remaining in anuria and anasarca that does not respond to intravenous diuretic in high doses, for
which it is necessary to start hemodialysis.

In summary, sudden onset nephrotic syndrome and rapidly progressive renal failure in a diabetic
patient. Renal biopsy was performed.
EVOLUTION
Renal biopsy showed diffuse segmental
glomerulosclerosis with formation of PAS-positive
eosinophilic nodules (Figure 1). The arterioles
showed frank PAS-positive hyalinosis and the
interstitium showed marked fibrosis with diffuse
inflammatory infiltrates formed by mature
lymphocytes and numerous eosinophils that
produced images of tubulitis (Figure 2).
Immunofluorescence was weak positive for
mesangial IgM, with no other findings. Congo red
was negative.

Therefore, the histological diagnosis was diabetic


nephropathy (diffuse nodular glomerulosclerosis
with arteriolar hyalinosis) and tubulo-interstitial
nephropathy with eosinophilia suggestive of drug
toxicity
With the diagnosis of immunoallergic interstitial nephritis due
to non-steroidal anti-inflammatory drugs (NSAIDs)
superimposed on diabetic nephropathy, treatment with oral
prednisone at a dose of mg / kg of body weight was started.
After ten days of treatment, there is a progressive increase in
diuresis and recovery of normal renal function (Crp: 0.6 mg /
dl; eGFR by MDRD-4:> 60 ml / min / 1.73 m2) which allowed
cessation of renal replacement therapy (Figure 3).

During admission, he presented progressive anemia to an Hb


of 8 g / dl, without platelet penia and with normal lactate
dehydrogenase, so it was ruled out that it was of hemolytic
origin, following the same course as the deterioration of renal
function, which suggested which was related to the acute renal
failure itself. Rectorrhagia was self-limited in two days, with
disappearance of diarrhea. In the previous admission to MI, a
non-contrast abdominal computed tomography had been
performed with no evidence of pathology, with a negative stool
occult blood determination and a normal coagulation study, so
the study was not expanded. Improvement in anemia was
noted (Hb: 11.6 g / dl) after blood transfusion, treatment with
intravenous iron and erythropoietin.
Antihypertensive treatment was adjusted, restarting angiotensin II receptor antagonist (ARB II),
calcium antagonist and alpha-blocker, with good control. In addition, insulin therapy was
modified with better glycemic control and oral anticoagulation was reintroduced without
complications and treatment lipid-lowering treatment.

After two months of oral prednisone in a descending regimen, there was a progressive
improvement in proteinuria, with the protein / creatinine ratio in urine being 756 mg / g in the
last check-up.

https://www.revistanefrologia.com/en-estadisticas-X188897001544566X
HERNIA VESICAL INGUINOESCROTAL GIGANTE CON UROPATIA
OBSTRUCTIVA
INCIPIENTE BILATERAL ASOCIADO
Paciente varón de 54 años, obeso, con antecedentes personales de diabetes mellitus
controlada con glipizida más metformina, así como hábito tabáquico importante, que acude al
servicio de urgencias por presentar dificultad para completar la micción, de varios meses de
evolución, aunque más acentuada en los últimos 20 días; refiriendo ser necesarias para ello
maniobras de compresión escrotal. Puntuación del Indice Internacional de Sintomatología
Prostática (IPSS)= 17.

En la exploración física se objetivó una gran hernia inguinoescotal izquierda, no complicada. El


tacto rectal no evidenció hallazgos significativos. Los resultados de las determinaciones
analíticas se situaron dentro de los rangos normales, excepto cifras de creatinina sérica de 1,7
mg/ dL. La presencia de resíduo postmiccional significativo, ausente ecográficamente tras
presión sobre el contenido hemiescrotal hizo indicar la cateterización vesical. Se realizó
cistouretrografía miccional seriada, que demostró la existencia de una gran hernia vesical
ínguinoescrotal (Figura 1). La urografía intravenosa permitió evidenciar desplazamiento de
ambos uréteres distales hacia el cuello herniario, así como cierta ectasia proximal de los
mismos (Figura 2).
Exámenes
complementarios
CASO CLÍNICO
Ante los hallazgos citados se programó al
paciente de forma conjunta con el Servicio
de Cirugía General, practicándose reducción
de la hernia vesical más hernioplastia
inguinal según técnica de Rutkow-Robbins,
con implante de malla de prolene. El
postoperatorio cursó sin incidencias, salvo
seroma de herida incisional, resuelto con
drenaje del mismo y curas locales.

En revisión posterior se comprobó


normalización clinicoanalítica, así como de la
imagen urográfica, tanto desde un punto de
vista morfológico como funcional (Figura 3).
Discusión
La hernia vesical ínguinoescrotal puede presentarse hasta en un 10 % de varones mayores de 50 años,
si bien es mucho menos frecuente que el componente cístico intrasacular supere el 50 % del total de
dicho órgano, tal y como ocurrió en nuestro caso. El predominio derecho (60 %) (2) no se verificó en
este paciente, aunque sí se correspondió con los tipos anatómicos más frecuentemente descritos
(inguinal - 75 % de casos -, y paraperitoneal - 60 % -).

Los factores fisiopatogénicos más determinantes en esta entidad son la uropatía obstructiva infravesical
y la debilidad parietal abdominal a nivel del canal inguinal. El sobrepeso e incrementos en la presión
intraabdominal en sujetos potencialmente tusígenos ejercen un efecto adyuvante.

La mayor parte de hernias vesicales incipientes se comportan de forma asintomática. Cuando progresan
en dimensiones, la micción en dos tiempos (ayudada en la segunda fase mediante la presión manual
escrotal) es la manifestación referida con más frecuencia - signo de Mery - . La insuficiencia renal
asociada a esta entidad también ha sido descrita en la literatura, aunque con mucha menor incidencia,
presentándose en aquellos casos de herniación trigonal asociada y consecuente tiraje y angulación
ureteral que condicionan una uropatía obstructiva. Nuestro caso clínico ofrece la particularidad de
haberse detectado en una fase incipiente de alteración de la función renal. Otros hallazgos publicados
en relación con las hernias vesicales han sido reflujo vésicoureteral, litiasis , e incluso neoplasias
uroteliales.
https://scielo.isciii.es/scielo.php?script=sci_arttext&pid=S0004-06142005000700012
EVALUACIÓN
Total de puntos1/5

NOMBRE COMPLETO *
Linda Paredes C

Paciente femenino de 63 años acude a la emergencia por presentar palpitaciones y


dificultad para respirar de rápida instauración. APP interés: HTA esencial de 18 años
de evolución tratada con Losartan y Clortalidona. T* 37,8 C PA 200/140 mmHg FC
120 lpm FR 28 rpm. Exploración física: edema de MMII bilateral, ingurgitación
yugular, a la auscultación cardiaca ritmo de galope. Hemograma normal, EGO:
Proteinuria ++ Sangre (+). Una vez estabilizada la paciente se le realiza USG renal
¿Qué esperamos encontrar? *
0/1

Perdida de la diferenciación cortico/medular


Riñones disminuidos de tamaño
Riñones aumentados de tamaño
Riñones dentro del tamaño normal

Paciente masculino de 18 años con APP de DMT1 de 5 años de evolución se realiza


chequeo de rutina en área de oftalmología y nefrología. Examen de fondo de ojo sin
hallazgos de interés, Cr 1.3 mg/dl y BUN 20 mg /dl. TFG 140 ml/min y en la USG
renal se aprecia silueta renal aumentada ¿En que estadio de la nefropatía diabética
se encuentra el paciente? *
0/1

Estadio lll, Nefropatía incipiente


Estadio lV, Nefropatía establecida
Estadio l, conducta expectante
Estadio V, enfermedad renal crónica

Paciente masculino de 67 años con APP de HTA de 23 años de evolución con pobre
adherencia al tratamiento, acude a emergencias por presentar disnea rápidamente
progresiva. T* 37,8 PA 150/90 FR 29 rpm FC 120 lpm. Al examen físico se aprecia
tiraje intercostal y supraesternal, edema en extremidades de predominio distal, a la
auscultación cardiaca ritmo de galope y en tórax disminución de ruidos respiratorios
en bases. Rx muestra imagen radiopaca que borra ángulos costofrénicos en ambos
hemitórax. Hemograma Hb 10 g/dl y plaquetas 50.000. Cr 2.3 mg/dl BUN 180 mg/dl
EGO: Proteinuria ++++ Sangre (+). Se le realiza USG renal ¿Qué esperamos
encontrar? *
1/1

Riñones dentro del tamaño normal


Riñones aumentados de tamaño
Perdida de la diferenciación cortico/medular
Conserva la diferenciación cortica/medular

Paciente de 55 años con APP de Diabetes Mellitus ll de 12 años de evolución


controlado con Metformina/Glibenclamida, como parte de medidas profilácticas ud
solicita 2 exámenes de albumina en orina/24h en los últimos 4 meses obteniendo: 250
mg/24h y 300 mg/24h respectivamente. Frente a estos resultados ¿Cuál sería su
diagnóstico y tratamiento? *
0/1
Nefropatía diabética establecida, iniciar tratamiento antiproteinúrico con IECA/ARAll
Insuficiencia renal aguda, iniciar terapia renal sustitutiva
Insuficiencia renal crónica terminal, iniciar terapia renal sutitutiva
Nefropatía diabética incipiente, iniciar tratamiento antiproteinúrico con IECA/ARAll

Paciente femenina de 45 años con APP DMT1 de 20 años de evolución y artritis


reumatoidea diagnosticada hace 5 años, acude a la consulta por presentar fatiga y
edema generalizado, que afecta su calidad de vida. Al examen físico se evidencia
edema peri orbitario y en extremidades inferiores que deja fóvea, líneas de Mhuercke
en la lámina ungueal y acropaquías, deformidad de dedos en cuello de cisne. Uso de
insulina SC y Naproxeno. Analítica sanguínea destacan albumina 2 g/dl Cr 1.7 g/dl
BUN 40 mg/dl. EGO: Proteínas ++++ Sangre (-) Glucosa (+). Ante estos hallazgos
¿Cuál sería su proceder? *
0/1

Estimar la albumina en orina de 24 horas


Ajustar dosis de insulina y agendar otra consulta en 3 meses
Biopsia renal para buscar depósitos amiloideos
TAC abdominopélvico para evaluar dimensiones renales

Linda
1. Paciente de sexo masculino de 50 años que debuta con HTA, edemas moderados, hematuria y
proteinuria de 3 g/24 horas, se sospecha de un síndrome nefrítico por lo cual se realizó un examen
serológico como dato relevante ANCA positivo. Luego al realizarse una biopsia no se encontraron
anticuerpos. ¿Cuál de las siguientes patologías NO ENTRARIA EN SUS DIFERENCIALES?

Granulomatosis de Wegener

Síndrome de Chug - Strauss

Poliangitis microscópica

Glomerulonefritis post- infecciosa

2. Cuál de los siguientes medicamentos usted le prescribiría a un paciente con diabetes mellitus
tipo 1?

Metformina

Sulfonilureas

Ninguna

Tiazolidindiona

3. De las siguientes combinaciones de fármacos selecciones cual NO está recomendada en el


tratamiento de HTA.

Bloqueante B + diuréticos

Bloqueante B + antagonista de Ca

IECA + Diureticos

ARA II + IECA

4. Llega a su consulta paciente de sexo femenino de 30 años por presentar una orina de un color
extraño (ver imagen), a su vez presenta una presión arterial de 125/83 mmHg. No tiene ningún
otro dato de relevancia; en el EGO se le reporta eritrocitos incontables de morfología normal.
Según su criterio ¿Cuál es el origen de la hematuria?

Síndrome nefrítico

Síndrome nefrótico

Nefropatía hipertensiva

Origen urogenital
5. En relación a las pruebas serológicas que es lo más lógico que espera encontrar positivo:

ANCA

ASTO

ANA

Anti GMB

6. Según la historia clínica en que patología se podría sospechar

Enfermedad de cambios mínimos

Nefropatía membranosa

ninguna de las anteriores

Glomerulonefritis post-infecciosa

7. En relación a las pruebas serológicas que es lo más lógico que espere encontrar con relación a
los factores de complemento:

C3 y C4 disminuidos

C3 disminuido y C4 normal

C3 normal y C4 muy disminuido

C3 y C4 normales

Paciente masculino de 9 años acude al centro de salud local por presentar edema en miembros
inferiores y cara de carácter matutino según relata la madre. PA 110/80 mmHg, T 37,2 C, FC 83
lpm, FR 14 rpm. Exploración física evidencia edema facial con predominio periorbitario y en
extremidades. En los análisis sanguíneos destacan Creatinina sérica 0,7 mg/dl, Urea 16 mg/dl
albumina sérica 2.0 g/dl. Se realiza EGO. ¿Cuál sería el siguiente examen por solicitar en este
paciente?

Ecografia abdominopelvica

Urocultivo y antibiograma

Orina en 24 horas

Perfil hepático
Paciente femenina de 85 años acude a consulta por presentar desde hace dos meses fatiga y
ganancia de peso. APP Artritis reumatoidea, ulcera péptica e Hipertensión. Consume de manera
crónica hidroclorotiazida y paracetamol. No refiere tabaquismo, alcoholismo ni uso de drogas
ilegales. Pa 130/70 mmHg, FC 80 lpm FR 14 rpm. Edema generalizado, palpación de hígado 5 cm
por debajo de reborde costal, EGO: Proteinuria ++++. USG Renal con crecimiento bilateral. Se indica
biopsia renal. ¿Cuál es el hallazgo patológico más probable que se observa en la biopsia?

Formación de semilunas

Depósitos de amiloide

Microscopia normal

Jorobas supetiliales

Paciente masculino de 32 años con reciente diagnóstico de linfoma de Hodgkin acude a consulta
por fatiga y edema generalizado. Analítica: Na 145 mEq/L, K 3.8 mEq/L, creatinina 0.9 mg/dl,
albumina sérica 2.0 g/dl, Bilirrubina total 0.9 mg/dl. EGO: Proteinuria ++++, Sangre (-), Signos
vitales normales. ¿Cuál de las siguientes enfermedades glomerulares es más probable que
presente el paciente?

Amiloidosis

Glomerulonefritis focal y segmentaria

Enfermedad de cambios mínimos

Glomerulonefritis membranopoliferativa

Paciente masculino de 22 años acude a urgencias por presentar orina oscura notada hoy en la
mañana APP de interés Amigdalectomía a los 9 años, Infección de vías respiratorias hace 4 días T
37,1 C, PA 145/92 mmHg, FC 80 lpm, FR 14 rpm. Exploración física sin dermatosis y articulaciones
normales. Hemograma con leve leucocitosis. EGO. Glucosa -, Proteina +, Cetonas -, Esterasa -.
Nitritos -, leucocitos 3-6/campo, eritrocitos 30-50/campo, cilindros eritrocitarios. Na +138 mEq, K
4,5 Me1/L, HCO3 22 mEq/L, BUN 18 mg/dl, creatinina 1,4 mg/dl. Niveles de complemento serico
normales, se solicitaron estudios serológicos con resultados pendientes. ¿Cuál es el diagnóstico más
probable?

SINDROME DE GOODPASTURE

GLOMERULONEFRITIS POST INFECCIOSA

NEFRITIS LUPICA

NEFROPATIA POR IgA


Paciente de 45 años acude al servicio de urgencias por presentar disnea, fatiga, disminución del
apetito y aumento de peso en las últimas 2 semanas. Síntomas iniciaron con disnea de esfuerzo. En
los últimos días refiere despertar en las noches con falta de aire y dificultad para abrir los ojos por
la mañana por edema facial. Sin APP de interés. No refiere abuso de alcohol tabaco o drogas. No
consume medicamentos. PA 200/120 mmHg, FC 100 lpm. Al examen físico se evidencia Anasarca,
distensión yugular al estar sentado, auscultación pulmonar con estertores basales bilaterales. EGO:
proteínas + Nitritos -, Esterasa leucocitaria, Eritrocitos >50/campo, cilindros eritrocitarios. ¿Cuál
sería el mecanismo más probable del edema de éste paciente?

Obstrucción linfática

Incremento de la presión hidrostática

Disminución de la presión oncotica

Incremento de permeabilidad capilar

También podría gustarte